DM pour les 302

This commit is contained in:
Bertrand Benjamin 2018-04-19 19:32:51 +03:00
parent ff836fb45b
commit c531792ff4
35 changed files with 8022 additions and 0 deletions

View File

@ -0,0 +1,278 @@
\documentclass[a4paper,12pt]{article}
\usepackage{myXsim}
\usepackage{tkz-fct}
\usepackage{wrapfig}
\title{DM de Paques}
\tribe{302}
\date{Jeudi 3 mai 2018}
\sujet{01}
%\geometry{left=10mm,right=10mm, bottom= 10mm, top=10mm}
\xsimsetup{
solution/print = false
}
\begin{document}
\maketitle
\bigskip
{\Large \textbf{Nom - Prénom:}}
\begin{exercise}[subtitle={Battle of the year}]
% theme: Fonction linéaire, Fonction affine
% require: tkz-fct
Taraina veut inscrire ses 21 élèves à un entrainement pour l'évènement \textbf{Battle of the year}.
Deux tarifs lui sont proposés:
\begin{itemize}
\item Tarif Individuel: 200 \euro par danseur inscrit.
\item Tarif Groupe : Paiement d'un forfait de 619 \euro pour le groupe puis 149 \euro par danseur inscrit.
\end{itemize}
\medskip
\begin{enumerate}
\item Complète le tableau suivant :
\medskip
\begin{tabularx}{0.7\linewidth}{|c|*{3}{>{\centering \arraybackslash}X|}}\hline
Nombre d'inscriptions & 0 & 10 & 25\\
\hline
Prix au tarif Individuel en \euro & & & \\
\hline
Prix au tarif Groupe en \euro & & & \\
\hline
\end{tabularx}
\begin{solution}
\begin{tabularx}{0.7\linewidth}{|c|*{3}{>{\centering \arraybackslash}X|}}\hline
Nombre d'inscriptions & 0 & 10 & 25\\
\hline
Prix au tarif Individuel en \euro & 0 & 2000 & 5000 \\
\hline
Prix au tarif Groupe en \euro & 619 & 2109 & 4344\\
\hline
\end{tabularx}
\end{solution}
\medskip
\item Pour chacun des tarifs, exprimer le prix en fonction du nombre de danseurs inscrits.
\begin{solution}
$x$ représente ici le nombre d'élèves inscrits.
\begin{itemize}
\item Tarif Individuel: $f: x \mapsto 200x$
\item Tarif Groupe: $g: x \mapsto 149x + 619$
\end{itemize}
\end{solution}
\item Tracer sur le graphique suivant, les courbes représentants les 2 tarifs proposés.
% On force que le graphique soit légèrement plus grand que 12 cm
\begin{tikzpicture}[yscale=0.5]
\tkzInit[xmin=0,xmax=26,
ymin=0,ymax=5200,
xstep=2,ystep=200]
\tkzAxeX[thick, poslabel=right,label=]
\tkzAxeY[thick, poslabel=above,label=]
\tkzDrawX[label={\textit{Danseurs inscrits}},below= -12pt]
\tkzDrawY[label={\textit{Prix}}, below=-10pt]
\tkzGrid
\tkzFct[domain=0:26, color=blue, very thick]{200*\x}
\tkzFct[domain=0:26, color=red, very thick]{149*\x+619}
\end{tikzpicture}
\item Pour quel nombre d'inscriptions paye-t-on le même prix quel que soit le tarif choisi?
\begin{solution}
$12.137254901960784$ % est entre 10 et 20 par contruction des paramètres
\end{solution}
\end{enumerate}
\end{exercise}
\begin{exercise}[subtitle={Parcours}]
% exo de geometrie comme au brevet blanc.
Une commune souhaite aménager des parcours de santé sur son territoire. On fait deux propositions au conseil municipale, schématisés ci-dessous:
\begin{itemize}
\item Le parcours ACDA
\item Le parcours AEFA
\end{itemize}
Ils souhaitent faire un parcours dont la longueur s'approche le plus possible de 107m.
Peux-tu les aider à choisir le parcours? Justifie
\textbf{Attention: La figure proposée au conseil municipale n'est pas à l'échelle, mais les codages et les dimension données sont correctes.}
\begin{minipage}{0.6\textwidth}
\includegraphics[scale = 0.4]{./fig/parcours}
\end{minipage}
\begin{minipage}{0.4\textwidth}
\begin{itemize}
\item $AC = 14m$
\item $CD = 48m$
\item $AE' = 19.1m$
\item $AE = 38.2m$
\item $AF = 15.3m$
\item $E'F' = 25.95m$
\item $(E'F') // (EF)$
\item L'angle $\widehat{EAF}$ vaut $30^o$
\end{itemize}
\end{minipage}
\begin{solution}
\begin{itemize}
\item Parcours ACDA:
D'après la figure, on voit que le triangle $ACD$ est rectangle en $C$ donc d'après le théorème de Pythagore, on a
\begin{align*}
AD^2 &= AC^2 + DC^2 \\
AD^2 &= 14^2 + 48^2 \\
AD^2 &= 196 + 2304 \\
AD^2 &= 2500 \\
AD &= \sqrt{2500} = 50m
\end{align*}
Donc le parcours ACDA mesure
\begin{align*}
AD + AC + CD = 50 + 14 + 48 = 112m
\end{align*}
\item Parcours AEFA:
D'après les données, on sait que $(EF) // (E'F')$. On voit aussi que $A$, $E'$ et $E$ sont alignés. Il en est de même pour les points $A$, $F'$ et $F$. Donc d'après le théorème de Thalès
\begin{tabular}{|c|c|c|c|}
\hline
Triangle AEF & AE = 38.2 & AF = 15.3 & EF \\
\hline
Triangle AE'F' & AE' = 19.1 & AF' & E'F' = 25.95 \\
\hline
\end{tabular}
est un tableau de proportionnalité. Donc on peut faire un produit en croix pour calcul $EF$.
\begin{align*}
EF = \frac{E'F' \times AE}{AE'} = \frac{25.95 \times 38.2}{19.1} = 51.9 m
\end{align*}
Donc le parcours AEFA mesure
\begin{align*}
AF + AE + EF = 15.3 + 38.2 + 51.9 = 105.4m
\end{align*}
\item Choix du parcours:
Il faudra choisir le tour $AFEA$ car sa longueur est plus proche de 107m.
\end{itemize}
\end{solution}
\end{exercise}
\begin{exercise}[subtitle={Tirages au sort}]
\renewcommand{\arraystretch}{1.5}
Dans une urne, on a placé des boules colorées indiscernables au touché. Il y a 9 boules bleu, 2 boules jaunes, 2 boules vertes et 6 boules rouges.
\begin{enumerate}
\item % Proba
\begin{enumerate}
\item Quelle est la probabilité de tirer une boule bleu?
\begin{solution}
$\dfrac{9}{19} \approx 0.47$
\end{solution}
\item Quelle est la probabilté de tirer une boule jaune ou bleu?
\begin{solution}
$\dfrac{11}{19} \approx 0.58$
\end{solution}
\item A-t-on plus de chance de tirer une boule verte ou une boule rouge?
\begin{solution}
Boules vertes: $\dfrac{2}{19} \approx 0.11$
Boules rouges: $\dfrac{6}{19} \approx 0.32$
Une boule rouge
\end{solution}
\end{enumerate}
\item % Stat
On effectue 14 tirages (avec remise) dans cette urne et on obtient les couleurs suivantes:
\begin{center}
B \hspace{0.4cm}J \hspace{0.4cm}B \hspace{0.4cm}J \hspace{0.4cm}B \hspace{0.4cm}B \hspace{0.4cm}B \hspace{0.4cm}V \hspace{0.4cm}R \hspace{0.4cm}B \hspace{0.4cm}J \hspace{0.4cm}B \hspace{0.4cm}B \hspace{0.4cm}J \\
\end{center}
\begin{enumerate}
\item Compléter le tableau des effectifs ci-dessous
\begin{tabular}{|c|*{4}{c|}}
\hline
Couleur & Bleu & Jaune & Vert & Rouge \\
\hline
Effectif & & & & \\
\hline
\end{tabular}
\begin{solution}
\begin{tabular}{|c|*{4}{c|}}
\hline
Couleur & Bleu & Jaune & Vert & Rouge \\
\hline
Effectif & 8 & 4 & 1 & 1 \\
\hline
\end{tabular}
\end{solution}
\item Calculer la fréquence des boules vertes.
\begin{solution}
Fréquence de boules vertes: $\frac{1}{14}$
\end{solution}
\end{enumerate}
\item À chaque couleur, on associe des points. Une boule bleu rapporte 10 points, une boule jaune 5 points, une boule verte 2 points et une boule rouge 0 points.
\begin{enumerate}
\item Combien de points a-t-on gagné au total?
\begin{solution}
17
\end{solution}
\item Calculer la moyenne des gains.
\begin{solution}
4.25
\end{solution}
\item Calculer la médiane des gains.
\begin{solution}
3.5
\end{solution}
\end{enumerate}
\end{enumerate}
\end{exercise}
\end{document}
%%% Local Variables:
%%% mode: latex
%%% TeX-master: "master"
%%% End:

View File

@ -0,0 +1,278 @@
\documentclass[a4paper,12pt]{article}
\usepackage{myXsim}
\usepackage{tkz-fct}
\usepackage{wrapfig}
\title{DM de Paques}
\tribe{302}
\date{Jeudi 3 mai 2018}
\sujet{02}
%\geometry{left=10mm,right=10mm, bottom= 10mm, top=10mm}
\xsimsetup{
solution/print = false
}
\begin{document}
\maketitle
\bigskip
{\Large \textbf{Nom - Prénom:}}
\begin{exercise}[subtitle={Battle of the year}]
% theme: Fonction linéaire, Fonction affine
% require: tkz-fct
Taraina veut inscrire ses 21 élèves à un entrainement pour l'évènement \textbf{Battle of the year}.
Deux tarifs lui sont proposés:
\begin{itemize}
\item Tarif Individuel: 112 \euro par danseur inscrit.
\item Tarif Groupe : Paiement d'un forfait de 435 \euro pour le groupe puis 83 \euro par danseur inscrit.
\end{itemize}
\medskip
\begin{enumerate}
\item Complète le tableau suivant :
\medskip
\begin{tabularx}{0.7\linewidth}{|c|*{3}{>{\centering \arraybackslash}X|}}\hline
Nombre d'inscriptions & 0 & 10 & 25\\
\hline
Prix au tarif Individuel en \euro & & & \\
\hline
Prix au tarif Groupe en \euro & & & \\
\hline
\end{tabularx}
\begin{solution}
\begin{tabularx}{0.7\linewidth}{|c|*{3}{>{\centering \arraybackslash}X|}}\hline
Nombre d'inscriptions & 0 & 10 & 25\\
\hline
Prix au tarif Individuel en \euro & 0 & 1120 & 2800 \\
\hline
Prix au tarif Groupe en \euro & 435 & 1265 & 2510\\
\hline
\end{tabularx}
\end{solution}
\medskip
\item Pour chacun des tarifs, exprimer le prix en fonction du nombre de danseurs inscrits.
\begin{solution}
$x$ représente ici le nombre d'élèves inscrits.
\begin{itemize}
\item Tarif Individuel: $f: x \mapsto 112x$
\item Tarif Groupe: $g: x \mapsto 83x + 435$
\end{itemize}
\end{solution}
\item Tracer sur le graphique suivant, les courbes représentants les 2 tarifs proposés.
% On force que le graphique soit légèrement plus grand que 12 cm
\begin{tikzpicture}[yscale=0.8]
\tkzInit[xmin=0,xmax=26,
ymin=0,ymax=2912,
xstep=2,ystep=200]
\tkzAxeX[thick, poslabel=right,label=]
\tkzAxeY[thick, poslabel=above,label=]
\tkzDrawX[label={\textit{Danseurs inscrits}},below= -12pt]
\tkzDrawY[label={\textit{Prix}}, below=-10pt]
\tkzGrid
\tkzFct[domain=0:26, color=blue, very thick]{112*\x}
\tkzFct[domain=0:26, color=red, very thick]{83*\x+435}
\end{tikzpicture}
\item Pour quel nombre d'inscriptions paye-t-on le même prix quel que soit le tarif choisi?
\begin{solution}
$15.0$ % est entre 10 et 20 par contruction des paramètres
\end{solution}
\end{enumerate}
\end{exercise}
\begin{exercise}[subtitle={Parcours}]
% exo de geometrie comme au brevet blanc.
Une commune souhaite aménager des parcours de santé sur son territoire. On fait deux propositions au conseil municipale, schématisés ci-dessous:
\begin{itemize}
\item Le parcours ACDA
\item Le parcours AEFA
\end{itemize}
Ils souhaitent faire un parcours dont la longueur s'approche le plus possible de 296m.
Peux-tu les aider à choisir le parcours? Justifie
\textbf{Attention: La figure proposée au conseil municipale n'est pas à l'échelle, mais les codages et les dimension données sont correctes.}
\begin{minipage}{0.6\textwidth}
\includegraphics[scale = 0.4]{./fig/parcours}
\end{minipage}
\begin{minipage}{0.4\textwidth}
\begin{itemize}
\item $AC = 100m$
\item $CD = 75m$
\item $AE' = 8.7m$
\item $AE = 26.1m$
\item $AF = 33.6m$
\item $E'F' = 78.23m$
\item $(E'F') // (EF)$
\item L'angle $\widehat{EAF}$ vaut $30^o$
\end{itemize}
\end{minipage}
\begin{solution}
\begin{itemize}
\item Parcours ACDA:
D'après la figure, on voit que le triangle $ACD$ est rectangle en $C$ donc d'après le théorème de Pythagore, on a
\begin{align*}
AD^2 &= AC^2 + DC^2 \\
AD^2 &= 100^2 + 75^2 \\
AD^2 &= 10000 + 5625 \\
AD^2 &= 15625 \\
AD &= \sqrt{15625} = 125m
\end{align*}
Donc le parcours ACDA mesure
\begin{align*}
AD + AC + CD = 125 + 100 + 75 = 300m
\end{align*}
\item Parcours AEFA:
D'après les données, on sait que $(EF) // (E'F')$. On voit aussi que $A$, $E'$ et $E$ sont alignés. Il en est de même pour les points $A$, $F'$ et $F$. Donc d'après le théorème de Thalès
\begin{tabular}{|c|c|c|c|}
\hline
Triangle AEF & AE = 26.1 & AF = 33.6 & EF \\
\hline
Triangle AE'F' & AE' = 8.7 & AF' & E'F' = 78.23 \\
\hline
\end{tabular}
est un tableau de proportionnalité. Donc on peut faire un produit en croix pour calcul $EF$.
\begin{align*}
EF = \frac{E'F' \times AE}{AE'} = \frac{78.23 \times 26.1}{8.7} = 234.7 m
\end{align*}
Donc le parcours AEFA mesure
\begin{align*}
AF + AE + EF = 33.6 + 26.1 + 234.7 = 294.4m
\end{align*}
\item Choix du parcours:
Il faudra choisir le tour $AFEA$ car sa longueur est plus proche de 296m.
\end{itemize}
\end{solution}
\end{exercise}
\begin{exercise}[subtitle={Tirages au sort}]
\renewcommand{\arraystretch}{1.5}
Dans une urne, on a placé des boules colorées indiscernables au touché. Il y a 2 boules bleu, 6 boules jaunes, 8 boules vertes et 6 boules rouges.
\begin{enumerate}
\item % Proba
\begin{enumerate}
\item Quelle est la probabilité de tirer une boule bleu?
\begin{solution}
$\dfrac{2}{22} \approx 0.09$
\end{solution}
\item Quelle est la probabilté de tirer une boule jaune ou bleu?
\begin{solution}
$\dfrac{8}{22} \approx 0.36$
\end{solution}
\item A-t-on plus de chance de tirer une boule verte ou une boule rouge?
\begin{solution}
Boules vertes: $\dfrac{8}{22} \approx 0.36$
Boules rouges: $\dfrac{6}{22} \approx 0.27$
Une boule verte
\end{solution}
\end{enumerate}
\item % Stat
On effectue 14 tirages (avec remise) dans cette urne et on obtient les couleurs suivantes:
\begin{center}
J \hspace{0.4cm}J \hspace{0.4cm}J \hspace{0.4cm}R \hspace{0.4cm}R \hspace{0.4cm}J \hspace{0.4cm}R \hspace{0.4cm}R \hspace{0.4cm}V \hspace{0.4cm}V \hspace{0.4cm}V \hspace{0.4cm}V \hspace{0.4cm}V \hspace{0.4cm}V \\
\end{center}
\begin{enumerate}
\item Compléter le tableau des effectifs ci-dessous
\begin{tabular}{|c|*{4}{c|}}
\hline
Couleur & Bleu & Jaune & Vert & Rouge \\
\hline
Effectif & & & & \\
\hline
\end{tabular}
\begin{solution}
\begin{tabular}{|c|*{4}{c|}}
\hline
Couleur & Bleu & Jaune & Vert & Rouge \\
\hline
Effectif & & 4 & 6 & 4 \\
\hline
\end{tabular}
\end{solution}
\item Calculer la fréquence des boules vertes.
\begin{solution}
Fréquence de boules vertes: $\frac{6}{14}$
\end{solution}
\end{enumerate}
\item À chaque couleur, on associe des points. Une boule bleu rapporte 10 points, une boule jaune 5 points, une boule verte 2 points et une boule rouge 0 points.
\begin{enumerate}
\item Combien de points a-t-on gagné au total?
\begin{solution}
17
\end{solution}
\item Calculer la moyenne des gains.
\begin{solution}
4.25
\end{solution}
\item Calculer la médiane des gains.
\begin{solution}
3.5
\end{solution}
\end{enumerate}
\end{enumerate}
\end{exercise}
\end{document}
%%% Local Variables:
%%% mode: latex
%%% TeX-master: "master"
%%% End:

View File

@ -0,0 +1,278 @@
\documentclass[a4paper,12pt]{article}
\usepackage{myXsim}
\usepackage{tkz-fct}
\usepackage{wrapfig}
\title{DM de Paques}
\tribe{302}
\date{Jeudi 3 mai 2018}
\sujet{03}
%\geometry{left=10mm,right=10mm, bottom= 10mm, top=10mm}
\xsimsetup{
solution/print = false
}
\begin{document}
\maketitle
\bigskip
{\Large \textbf{Nom - Prénom:}}
\begin{exercise}[subtitle={Battle of the year}]
% theme: Fonction linéaire, Fonction affine
% require: tkz-fct
Taraina veut inscrire ses 21 élèves à un entrainement pour l'évènement \textbf{Battle of the year}.
Deux tarifs lui sont proposés:
\begin{itemize}
\item Tarif Individuel: 151 \euro par danseur inscrit.
\item Tarif Groupe : Paiement d'un forfait de 453 \euro pour le groupe puis 109 \euro par danseur inscrit.
\end{itemize}
\medskip
\begin{enumerate}
\item Complète le tableau suivant :
\medskip
\begin{tabularx}{0.7\linewidth}{|c|*{3}{>{\centering \arraybackslash}X|}}\hline
Nombre d'inscriptions & 0 & 10 & 25\\
\hline
Prix au tarif Individuel en \euro & & & \\
\hline
Prix au tarif Groupe en \euro & & & \\
\hline
\end{tabularx}
\begin{solution}
\begin{tabularx}{0.7\linewidth}{|c|*{3}{>{\centering \arraybackslash}X|}}\hline
Nombre d'inscriptions & 0 & 10 & 25\\
\hline
Prix au tarif Individuel en \euro & 0 & 1510 & 3775 \\
\hline
Prix au tarif Groupe en \euro & 453 & 1543 & 3178\\
\hline
\end{tabularx}
\end{solution}
\medskip
\item Pour chacun des tarifs, exprimer le prix en fonction du nombre de danseurs inscrits.
\begin{solution}
$x$ représente ici le nombre d'élèves inscrits.
\begin{itemize}
\item Tarif Individuel: $f: x \mapsto 151x$
\item Tarif Groupe: $g: x \mapsto 109x + 453$
\end{itemize}
\end{solution}
\item Tracer sur le graphique suivant, les courbes représentants les 2 tarifs proposés.
% On force que le graphique soit légèrement plus grand que 12 cm
\begin{tikzpicture}[yscale=0.6]
\tkzInit[xmin=0,xmax=26,
ymin=0,ymax=3926,
xstep=2,ystep=200]
\tkzAxeX[thick, poslabel=right,label=]
\tkzAxeY[thick, poslabel=above,label=]
\tkzDrawX[label={\textit{Danseurs inscrits}},below= -12pt]
\tkzDrawY[label={\textit{Prix}}, below=-10pt]
\tkzGrid
\tkzFct[domain=0:26, color=blue, very thick]{151*\x}
\tkzFct[domain=0:26, color=red, very thick]{109*\x+453}
\end{tikzpicture}
\item Pour quel nombre d'inscriptions paye-t-on le même prix quel que soit le tarif choisi?
\begin{solution}
$10.785714285714286$ % est entre 10 et 20 par contruction des paramètres
\end{solution}
\end{enumerate}
\end{exercise}
\begin{exercise}[subtitle={Parcours}]
% exo de geometrie comme au brevet blanc.
Une commune souhaite aménager des parcours de santé sur son territoire. On fait deux propositions au conseil municipale, schématisés ci-dessous:
\begin{itemize}
\item Le parcours ACDA
\item Le parcours AEFA
\end{itemize}
Ils souhaitent faire un parcours dont la longueur s'approche le plus possible de 60km.
Peux-tu les aider à choisir le parcours? Justifie
\textbf{Attention: La figure proposée au conseil municipale n'est pas à l'échelle, mais les codages et les dimension données sont correctes.}
\begin{minipage}{0.6\textwidth}
\includegraphics[scale = 0.4]{./fig/parcours}
\end{minipage}
\begin{minipage}{0.4\textwidth}
\begin{itemize}
\item $AC = 10km$
\item $CD = 24km$
\item $AE' = 12.35km$
\item $AE = 24.7km$
\item $AF = 12.2km$
\item $E'F' = 8.55km$
\item $(E'F') // (EF)$
\item L'angle $\widehat{EAF}$ vaut $30^o$
\end{itemize}
\end{minipage}
\begin{solution}
\begin{itemize}
\item Parcours ACDA:
D'après la figure, on voit que le triangle $ACD$ est rectangle en $C$ donc d'après le théorème de Pythagore, on a
\begin{align*}
AD^2 &= AC^2 + DC^2 \\
AD^2 &= 10^2 + 24^2 \\
AD^2 &= 100 + 576 \\
AD^2 &= 676 \\
AD &= \sqrt{676} = 26km
\end{align*}
Donc le parcours ACDA mesure
\begin{align*}
AD + AC + CD = 26 + 10 + 24 = 60km
\end{align*}
\item Parcours AEFA:
D'après les données, on sait que $(EF) // (E'F')$. On voit aussi que $A$, $E'$ et $E$ sont alignés. Il en est de même pour les points $A$, $F'$ et $F$. Donc d'après le théorème de Thalès
\begin{tabular}{|c|c|c|c|}
\hline
Triangle AEF & AE = 24.7 & AF = 12.2 & EF \\
\hline
Triangle AE'F' & AE' = 12.35 & AF' & E'F' = 8.55 \\
\hline
\end{tabular}
est un tableau de proportionnalité. Donc on peut faire un produit en croix pour calcul $EF$.
\begin{align*}
EF = \frac{E'F' \times AE}{AE'} = \frac{8.55 \times 24.7}{12.35} = 17.1 km
\end{align*}
Donc le parcours AEFA mesure
\begin{align*}
AF + AE + EF = 12.2 + 24.7 + 17.1 = 54.0km
\end{align*}
\item Choix du parcours:
Il faudra choisir le tour $ACDA$ car sa longueur est plus proche de 60km.
\end{itemize}
\end{solution}
\end{exercise}
\begin{exercise}[subtitle={Tirages au sort}]
\renewcommand{\arraystretch}{1.5}
Dans une urne, on a placé des boules colorées indiscernables au touché. Il y a 5 boules bleu, 5 boules jaunes, 5 boules vertes et 10 boules rouges.
\begin{enumerate}
\item % Proba
\begin{enumerate}
\item Quelle est la probabilité de tirer une boule bleu?
\begin{solution}
$\dfrac{5}{25} \approx 0.2$
\end{solution}
\item Quelle est la probabilté de tirer une boule jaune ou bleu?
\begin{solution}
$\dfrac{10}{25} \approx 0.4$
\end{solution}
\item A-t-on plus de chance de tirer une boule verte ou une boule rouge?
\begin{solution}
Boules vertes: $\dfrac{5}{25} \approx 0.2$
Boules rouges: $\dfrac{10}{25} \approx 0.4$
Une boule rouge
\end{solution}
\end{enumerate}
\item % Stat
On effectue 14 tirages (avec remise) dans cette urne et on obtient les couleurs suivantes:
\begin{center}
R \hspace{0.4cm}R \hspace{0.4cm}J \hspace{0.4cm}B \hspace{0.4cm}R \hspace{0.4cm}V \hspace{0.4cm}R \hspace{0.4cm}V \hspace{0.4cm}J \hspace{0.4cm}B \hspace{0.4cm}B \hspace{0.4cm}J \hspace{0.4cm}B \hspace{0.4cm}B \\
\end{center}
\begin{enumerate}
\item Compléter le tableau des effectifs ci-dessous
\begin{tabular}{|c|*{4}{c|}}
\hline
Couleur & Bleu & Jaune & Vert & Rouge \\
\hline
Effectif & & & & \\
\hline
\end{tabular}
\begin{solution}
\begin{tabular}{|c|*{4}{c|}}
\hline
Couleur & Bleu & Jaune & Vert & Rouge \\
\hline
Effectif & 5 & 3 & 2 & 4 \\
\hline
\end{tabular}
\end{solution}
\item Calculer la fréquence des boules vertes.
\begin{solution}
Fréquence de boules vertes: $\frac{2}{14}$
\end{solution}
\end{enumerate}
\item À chaque couleur, on associe des points. Une boule bleu rapporte 10 points, une boule jaune 5 points, une boule verte 2 points et une boule rouge 0 points.
\begin{enumerate}
\item Combien de points a-t-on gagné au total?
\begin{solution}
17
\end{solution}
\item Calculer la moyenne des gains.
\begin{solution}
4.25
\end{solution}
\item Calculer la médiane des gains.
\begin{solution}
3.5
\end{solution}
\end{enumerate}
\end{enumerate}
\end{exercise}
\end{document}
%%% Local Variables:
%%% mode: latex
%%% TeX-master: "master"
%%% End:

View File

@ -0,0 +1,278 @@
\documentclass[a4paper,12pt]{article}
\usepackage{myXsim}
\usepackage{tkz-fct}
\usepackage{wrapfig}
\title{DM de Paques}
\tribe{302}
\date{Jeudi 3 mai 2018}
\sujet{04}
%\geometry{left=10mm,right=10mm, bottom= 10mm, top=10mm}
\xsimsetup{
solution/print = false
}
\begin{document}
\maketitle
\bigskip
{\Large \textbf{Nom - Prénom:}}
\begin{exercise}[subtitle={Battle of the year}]
% theme: Fonction linéaire, Fonction affine
% require: tkz-fct
Taraina veut inscrire ses 21 élèves à un entrainement pour l'évènement \textbf{Battle of the year}.
Deux tarifs lui sont proposés:
\begin{itemize}
\item Tarif Individuel: 171 \euro par danseur inscrit.
\item Tarif Groupe : Paiement d'un forfait de 594 \euro pour le groupe puis 127 \euro par danseur inscrit.
\end{itemize}
\medskip
\begin{enumerate}
\item Complète le tableau suivant :
\medskip
\begin{tabularx}{0.7\linewidth}{|c|*{3}{>{\centering \arraybackslash}X|}}\hline
Nombre d'inscriptions & 0 & 10 & 25\\
\hline
Prix au tarif Individuel en \euro & & & \\
\hline
Prix au tarif Groupe en \euro & & & \\
\hline
\end{tabularx}
\begin{solution}
\begin{tabularx}{0.7\linewidth}{|c|*{3}{>{\centering \arraybackslash}X|}}\hline
Nombre d'inscriptions & 0 & 10 & 25\\
\hline
Prix au tarif Individuel en \euro & 0 & 1710 & 4275 \\
\hline
Prix au tarif Groupe en \euro & 594 & 1864 & 3769\\
\hline
\end{tabularx}
\end{solution}
\medskip
\item Pour chacun des tarifs, exprimer le prix en fonction du nombre de danseurs inscrits.
\begin{solution}
$x$ représente ici le nombre d'élèves inscrits.
\begin{itemize}
\item Tarif Individuel: $f: x \mapsto 171x$
\item Tarif Groupe: $g: x \mapsto 127x + 594$
\end{itemize}
\end{solution}
\item Tracer sur le graphique suivant, les courbes représentants les 2 tarifs proposés.
% On force que le graphique soit légèrement plus grand que 12 cm
\begin{tikzpicture}[yscale=0.5]
\tkzInit[xmin=0,xmax=26,
ymin=0,ymax=4446,
xstep=2,ystep=200]
\tkzAxeX[thick, poslabel=right,label=]
\tkzAxeY[thick, poslabel=above,label=]
\tkzDrawX[label={\textit{Danseurs inscrits}},below= -12pt]
\tkzDrawY[label={\textit{Prix}}, below=-10pt]
\tkzGrid
\tkzFct[domain=0:26, color=blue, very thick]{171*\x}
\tkzFct[domain=0:26, color=red, very thick]{127*\x+594}
\end{tikzpicture}
\item Pour quel nombre d'inscriptions paye-t-on le même prix quel que soit le tarif choisi?
\begin{solution}
$13.5$ % est entre 10 et 20 par contruction des paramètres
\end{solution}
\end{enumerate}
\end{exercise}
\begin{exercise}[subtitle={Parcours}]
% exo de geometrie comme au brevet blanc.
Une commune souhaite aménager des parcours de santé sur son territoire. On fait deux propositions au conseil municipale, schématisés ci-dessous:
\begin{itemize}
\item Le parcours ACDA
\item Le parcours AEFA
\end{itemize}
Ils souhaitent faire un parcours dont la longueur s'approche le plus possible de 297m.
Peux-tu les aider à choisir le parcours? Justifie
\textbf{Attention: La figure proposée au conseil municipale n'est pas à l'échelle, mais les codages et les dimension données sont correctes.}
\begin{minipage}{0.6\textwidth}
\includegraphics[scale = 0.4]{./fig/parcours}
\end{minipage}
\begin{minipage}{0.4\textwidth}
\begin{itemize}
\item $AC = 100m$
\item $CD = 75m$
\item $AE' = 1.45m$
\item $AE = 5.8m$
\item $AF = 91.2m$
\item $E'F' = 49.5m$
\item $(E'F') // (EF)$
\item L'angle $\widehat{EAF}$ vaut $30^o$
\end{itemize}
\end{minipage}
\begin{solution}
\begin{itemize}
\item Parcours ACDA:
D'après la figure, on voit que le triangle $ACD$ est rectangle en $C$ donc d'après le théorème de Pythagore, on a
\begin{align*}
AD^2 &= AC^2 + DC^2 \\
AD^2 &= 100^2 + 75^2 \\
AD^2 &= 10000 + 5625 \\
AD^2 &= 15625 \\
AD &= \sqrt{15625} = 125m
\end{align*}
Donc le parcours ACDA mesure
\begin{align*}
AD + AC + CD = 125 + 100 + 75 = 300m
\end{align*}
\item Parcours AEFA:
D'après les données, on sait que $(EF) // (E'F')$. On voit aussi que $A$, $E'$ et $E$ sont alignés. Il en est de même pour les points $A$, $F'$ et $F$. Donc d'après le théorème de Thalès
\begin{tabular}{|c|c|c|c|}
\hline
Triangle AEF & AE = 5.8 & AF = 91.2 & EF \\
\hline
Triangle AE'F' & AE' = 1.45 & AF' & E'F' = 49.5 \\
\hline
\end{tabular}
est un tableau de proportionnalité. Donc on peut faire un produit en croix pour calcul $EF$.
\begin{align*}
EF = \frac{E'F' \times AE}{AE'} = \frac{49.5 \times 5.8}{1.45} = 198.0 m
\end{align*}
Donc le parcours AEFA mesure
\begin{align*}
AF + AE + EF = 91.2 + 5.8 + 198.0 = 295.0m
\end{align*}
\item Choix du parcours:
Il faudra choisir le tour $AFEA$ car sa longueur est plus proche de 297m.
\end{itemize}
\end{solution}
\end{exercise}
\begin{exercise}[subtitle={Tirages au sort}]
\renewcommand{\arraystretch}{1.5}
Dans une urne, on a placé des boules colorées indiscernables au touché. Il y a 5 boules bleu, 2 boules jaunes, 7 boules vertes et 4 boules rouges.
\begin{enumerate}
\item % Proba
\begin{enumerate}
\item Quelle est la probabilité de tirer une boule bleu?
\begin{solution}
$\dfrac{5}{18} \approx 0.28$
\end{solution}
\item Quelle est la probabilté de tirer une boule jaune ou bleu?
\begin{solution}
$\dfrac{7}{18} \approx 0.39$
\end{solution}
\item A-t-on plus de chance de tirer une boule verte ou une boule rouge?
\begin{solution}
Boules vertes: $\dfrac{7}{18} \approx 0.39$
Boules rouges: $\dfrac{4}{18} \approx 0.22$
Une boule verte
\end{solution}
\end{enumerate}
\item % Stat
On effectue 14 tirages (avec remise) dans cette urne et on obtient les couleurs suivantes:
\begin{center}
R \hspace{0.4cm}V \hspace{0.4cm}R \hspace{0.4cm}R \hspace{0.4cm}R \hspace{0.4cm}B \hspace{0.4cm}R \hspace{0.4cm}V \hspace{0.4cm}B \hspace{0.4cm}B \hspace{0.4cm}V \hspace{0.4cm}V \hspace{0.4cm}V \hspace{0.4cm}R \\
\end{center}
\begin{enumerate}
\item Compléter le tableau des effectifs ci-dessous
\begin{tabular}{|c|*{4}{c|}}
\hline
Couleur & Bleu & Jaune & Vert & Rouge \\
\hline
Effectif & & & & \\
\hline
\end{tabular}
\begin{solution}
\begin{tabular}{|c|*{4}{c|}}
\hline
Couleur & Bleu & Jaune & Vert & Rouge \\
\hline
Effectif & 3 & & 5 & 6 \\
\hline
\end{tabular}
\end{solution}
\item Calculer la fréquence des boules vertes.
\begin{solution}
Fréquence de boules vertes: $\frac{5}{14}$
\end{solution}
\end{enumerate}
\item À chaque couleur, on associe des points. Une boule bleu rapporte 10 points, une boule jaune 5 points, une boule verte 2 points et une boule rouge 0 points.
\begin{enumerate}
\item Combien de points a-t-on gagné au total?
\begin{solution}
17
\end{solution}
\item Calculer la moyenne des gains.
\begin{solution}
4.25
\end{solution}
\item Calculer la médiane des gains.
\begin{solution}
3.5
\end{solution}
\end{enumerate}
\end{enumerate}
\end{exercise}
\end{document}
%%% Local Variables:
%%% mode: latex
%%% TeX-master: "master"
%%% End:

View File

@ -0,0 +1,278 @@
\documentclass[a4paper,12pt]{article}
\usepackage{myXsim}
\usepackage{tkz-fct}
\usepackage{wrapfig}
\title{DM de Paques}
\tribe{302}
\date{Jeudi 3 mai 2018}
\sujet{05}
%\geometry{left=10mm,right=10mm, bottom= 10mm, top=10mm}
\xsimsetup{
solution/print = false
}
\begin{document}
\maketitle
\bigskip
{\Large \textbf{Nom - Prénom:}}
\begin{exercise}[subtitle={Battle of the year}]
% theme: Fonction linéaire, Fonction affine
% require: tkz-fct
Taraina veut inscrire ses 21 élèves à un entrainement pour l'évènement \textbf{Battle of the year}.
Deux tarifs lui sont proposés:
\begin{itemize}
\item Tarif Individuel: 151 \euro par danseur inscrit.
\item Tarif Groupe : Paiement d'un forfait de 498 \euro pour le groupe puis 115 \euro par danseur inscrit.
\end{itemize}
\medskip
\begin{enumerate}
\item Complète le tableau suivant :
\medskip
\begin{tabularx}{0.7\linewidth}{|c|*{3}{>{\centering \arraybackslash}X|}}\hline
Nombre d'inscriptions & 0 & 10 & 25\\
\hline
Prix au tarif Individuel en \euro & & & \\
\hline
Prix au tarif Groupe en \euro & & & \\
\hline
\end{tabularx}
\begin{solution}
\begin{tabularx}{0.7\linewidth}{|c|*{3}{>{\centering \arraybackslash}X|}}\hline
Nombre d'inscriptions & 0 & 10 & 25\\
\hline
Prix au tarif Individuel en \euro & 0 & 1510 & 3775 \\
\hline
Prix au tarif Groupe en \euro & 498 & 1648 & 3373\\
\hline
\end{tabularx}
\end{solution}
\medskip
\item Pour chacun des tarifs, exprimer le prix en fonction du nombre de danseurs inscrits.
\begin{solution}
$x$ représente ici le nombre d'élèves inscrits.
\begin{itemize}
\item Tarif Individuel: $f: x \mapsto 151x$
\item Tarif Groupe: $g: x \mapsto 115x + 498$
\end{itemize}
\end{solution}
\item Tracer sur le graphique suivant, les courbes représentants les 2 tarifs proposés.
% On force que le graphique soit légèrement plus grand que 12 cm
\begin{tikzpicture}[yscale=0.6]
\tkzInit[xmin=0,xmax=26,
ymin=0,ymax=3926,
xstep=2,ystep=200]
\tkzAxeX[thick, poslabel=right,label=]
\tkzAxeY[thick, poslabel=above,label=]
\tkzDrawX[label={\textit{Danseurs inscrits}},below= -12pt]
\tkzDrawY[label={\textit{Prix}}, below=-10pt]
\tkzGrid
\tkzFct[domain=0:26, color=blue, very thick]{151*\x}
\tkzFct[domain=0:26, color=red, very thick]{115*\x+498}
\end{tikzpicture}
\item Pour quel nombre d'inscriptions paye-t-on le même prix quel que soit le tarif choisi?
\begin{solution}
$13.833333333333334$ % est entre 10 et 20 par contruction des paramètres
\end{solution}
\end{enumerate}
\end{exercise}
\begin{exercise}[subtitle={Parcours}]
% exo de geometrie comme au brevet blanc.
Une commune souhaite aménager des parcours de santé sur son territoire. On fait deux propositions au conseil municipale, schématisés ci-dessous:
\begin{itemize}
\item Le parcours ACDA
\item Le parcours AEFA
\end{itemize}
Ils souhaitent faire un parcours dont la longueur s'approche le plus possible de 300m.
Peux-tu les aider à choisir le parcours? Justifie
\textbf{Attention: La figure proposée au conseil municipale n'est pas à l'échelle, mais les codages et les dimension données sont correctes.}
\begin{minipage}{0.6\textwidth}
\includegraphics[scale = 0.4]{./fig/parcours}
\end{minipage}
\begin{minipage}{0.4\textwidth}
\begin{itemize}
\item $AC = 144m$
\item $CD = 17m$
\item $AE' = 20.64m$
\item $AE = 103.2m$
\item $AF = 139.2m$
\item $E'F' = 11.68m$
\item $(E'F') // (EF)$
\item L'angle $\widehat{EAF}$ vaut $30^o$
\end{itemize}
\end{minipage}
\begin{solution}
\begin{itemize}
\item Parcours ACDA:
D'après la figure, on voit que le triangle $ACD$ est rectangle en $C$ donc d'après le théorème de Pythagore, on a
\begin{align*}
AD^2 &= AC^2 + DC^2 \\
AD^2 &= 144^2 + 17^2 \\
AD^2 &= 20736 + 289 \\
AD^2 &= 21025 \\
AD &= \sqrt{21025} = 145m
\end{align*}
Donc le parcours ACDA mesure
\begin{align*}
AD + AC + CD = 145 + 144 + 17 = 306m
\end{align*}
\item Parcours AEFA:
D'après les données, on sait que $(EF) // (E'F')$. On voit aussi que $A$, $E'$ et $E$ sont alignés. Il en est de même pour les points $A$, $F'$ et $F$. Donc d'après le théorème de Thalès
\begin{tabular}{|c|c|c|c|}
\hline
Triangle AEF & AE = 103.2 & AF = 139.2 & EF \\
\hline
Triangle AE'F' & AE' = 20.64 & AF' & E'F' = 11.68 \\
\hline
\end{tabular}
est un tableau de proportionnalité. Donc on peut faire un produit en croix pour calcul $EF$.
\begin{align*}
EF = \frac{E'F' \times AE}{AE'} = \frac{11.68 \times 103.2}{20.64} = 58.4 m
\end{align*}
Donc le parcours AEFA mesure
\begin{align*}
AF + AE + EF = 139.2 + 103.2 + 58.4 = 300.8m
\end{align*}
\item Choix du parcours:
Il faudra choisir le tour $AFEA$ car sa longueur est plus proche de 300m.
\end{itemize}
\end{solution}
\end{exercise}
\begin{exercise}[subtitle={Tirages au sort}]
\renewcommand{\arraystretch}{1.5}
Dans une urne, on a placé des boules colorées indiscernables au touché. Il y a 9 boules bleu, 4 boules jaunes, 8 boules vertes et 3 boules rouges.
\begin{enumerate}
\item % Proba
\begin{enumerate}
\item Quelle est la probabilité de tirer une boule bleu?
\begin{solution}
$\dfrac{9}{24} \approx 0.38$
\end{solution}
\item Quelle est la probabilté de tirer une boule jaune ou bleu?
\begin{solution}
$\dfrac{13}{24} \approx 0.54$
\end{solution}
\item A-t-on plus de chance de tirer une boule verte ou une boule rouge?
\begin{solution}
Boules vertes: $\dfrac{8}{24} \approx 0.33$
Boules rouges: $\dfrac{3}{24} \approx 0.12$
Une boule verte
\end{solution}
\end{enumerate}
\item % Stat
On effectue 14 tirages (avec remise) dans cette urne et on obtient les couleurs suivantes:
\begin{center}
B \hspace{0.4cm}B \hspace{0.4cm}B \hspace{0.4cm}B \hspace{0.4cm}J \hspace{0.4cm}V \hspace{0.4cm}V \hspace{0.4cm}J \hspace{0.4cm}J \hspace{0.4cm}V \hspace{0.4cm}V \hspace{0.4cm}R \hspace{0.4cm}B \hspace{0.4cm}V \\
\end{center}
\begin{enumerate}
\item Compléter le tableau des effectifs ci-dessous
\begin{tabular}{|c|*{4}{c|}}
\hline
Couleur & Bleu & Jaune & Vert & Rouge \\
\hline
Effectif & & & & \\
\hline
\end{tabular}
\begin{solution}
\begin{tabular}{|c|*{4}{c|}}
\hline
Couleur & Bleu & Jaune & Vert & Rouge \\
\hline
Effectif & 5 & 3 & 5 & 1 \\
\hline
\end{tabular}
\end{solution}
\item Calculer la fréquence des boules vertes.
\begin{solution}
Fréquence de boules vertes: $\frac{5}{14}$
\end{solution}
\end{enumerate}
\item À chaque couleur, on associe des points. Une boule bleu rapporte 10 points, une boule jaune 5 points, une boule verte 2 points et une boule rouge 0 points.
\begin{enumerate}
\item Combien de points a-t-on gagné au total?
\begin{solution}
17
\end{solution}
\item Calculer la moyenne des gains.
\begin{solution}
4.25
\end{solution}
\item Calculer la médiane des gains.
\begin{solution}
3.5
\end{solution}
\end{enumerate}
\end{enumerate}
\end{exercise}
\end{document}
%%% Local Variables:
%%% mode: latex
%%% TeX-master: "master"
%%% End:

View File

@ -0,0 +1,278 @@
\documentclass[a4paper,12pt]{article}
\usepackage{myXsim}
\usepackage{tkz-fct}
\usepackage{wrapfig}
\title{DM de Paques}
\tribe{302}
\date{Jeudi 3 mai 2018}
\sujet{06}
%\geometry{left=10mm,right=10mm, bottom= 10mm, top=10mm}
\xsimsetup{
solution/print = false
}
\begin{document}
\maketitle
\bigskip
{\Large \textbf{Nom - Prénom:}}
\begin{exercise}[subtitle={Battle of the year}]
% theme: Fonction linéaire, Fonction affine
% require: tkz-fct
Taraina veut inscrire ses 21 élèves à un entrainement pour l'évènement \textbf{Battle of the year}.
Deux tarifs lui sont proposés:
\begin{itemize}
\item Tarif Individuel: 145 \euro par danseur inscrit.
\item Tarif Groupe : Paiement d'un forfait de 569 \euro pour le groupe puis 106 \euro par danseur inscrit.
\end{itemize}
\medskip
\begin{enumerate}
\item Complète le tableau suivant :
\medskip
\begin{tabularx}{0.7\linewidth}{|c|*{3}{>{\centering \arraybackslash}X|}}\hline
Nombre d'inscriptions & 0 & 10 & 25\\
\hline
Prix au tarif Individuel en \euro & & & \\
\hline
Prix au tarif Groupe en \euro & & & \\
\hline
\end{tabularx}
\begin{solution}
\begin{tabularx}{0.7\linewidth}{|c|*{3}{>{\centering \arraybackslash}X|}}\hline
Nombre d'inscriptions & 0 & 10 & 25\\
\hline
Prix au tarif Individuel en \euro & 0 & 1450 & 3625 \\
\hline
Prix au tarif Groupe en \euro & 569 & 1629 & 3219\\
\hline
\end{tabularx}
\end{solution}
\medskip
\item Pour chacun des tarifs, exprimer le prix en fonction du nombre de danseurs inscrits.
\begin{solution}
$x$ représente ici le nombre d'élèves inscrits.
\begin{itemize}
\item Tarif Individuel: $f: x \mapsto 145x$
\item Tarif Groupe: $g: x \mapsto 106x + 569$
\end{itemize}
\end{solution}
\item Tracer sur le graphique suivant, les courbes représentants les 2 tarifs proposés.
% On force que le graphique soit légèrement plus grand que 12 cm
\begin{tikzpicture}[yscale=0.6]
\tkzInit[xmin=0,xmax=26,
ymin=0,ymax=3770,
xstep=2,ystep=200]
\tkzAxeX[thick, poslabel=right,label=]
\tkzAxeY[thick, poslabel=above,label=]
\tkzDrawX[label={\textit{Danseurs inscrits}},below= -12pt]
\tkzDrawY[label={\textit{Prix}}, below=-10pt]
\tkzGrid
\tkzFct[domain=0:26, color=blue, very thick]{145*\x}
\tkzFct[domain=0:26, color=red, very thick]{106*\x+569}
\end{tikzpicture}
\item Pour quel nombre d'inscriptions paye-t-on le même prix quel que soit le tarif choisi?
\begin{solution}
$14.58974358974359$ % est entre 10 et 20 par contruction des paramètres
\end{solution}
\end{enumerate}
\end{exercise}
\begin{exercise}[subtitle={Parcours}]
% exo de geometrie comme au brevet blanc.
Une commune souhaite aménager des parcours de santé sur son territoire. On fait deux propositions au conseil municipale, schématisés ci-dessous:
\begin{itemize}
\item Le parcours ACDA
\item Le parcours AEFA
\end{itemize}
Ils souhaitent faire un parcours dont la longueur s'approche le plus possible de 167m.
Peux-tu les aider à choisir le parcours? Justifie
\textbf{Attention: La figure proposée au conseil municipale n'est pas à l'échelle, mais les codages et les dimension données sont correctes.}
\begin{minipage}{0.6\textwidth}
\includegraphics[scale = 0.4]{./fig/parcours}
\end{minipage}
\begin{minipage}{0.4\textwidth}
\begin{itemize}
\item $AC = 70m$
\item $CD = 24m$
\item $AE' = 0.57m$
\item $AE = 1.7m$
\item $AF = 36.7m$
\item $E'F' = 41.4m$
\item $(E'F') // (EF)$
\item L'angle $\widehat{EAF}$ vaut $30^o$
\end{itemize}
\end{minipage}
\begin{solution}
\begin{itemize}
\item Parcours ACDA:
D'après la figure, on voit que le triangle $ACD$ est rectangle en $C$ donc d'après le théorème de Pythagore, on a
\begin{align*}
AD^2 &= AC^2 + DC^2 \\
AD^2 &= 70^2 + 24^2 \\
AD^2 &= 4900 + 576 \\
AD^2 &= 5476 \\
AD &= \sqrt{5476} = 74m
\end{align*}
Donc le parcours ACDA mesure
\begin{align*}
AD + AC + CD = 74 + 70 + 24 = 168m
\end{align*}
\item Parcours AEFA:
D'après les données, on sait que $(EF) // (E'F')$. On voit aussi que $A$, $E'$ et $E$ sont alignés. Il en est de même pour les points $A$, $F'$ et $F$. Donc d'après le théorème de Thalès
\begin{tabular}{|c|c|c|c|}
\hline
Triangle AEF & AE = 1.7 & AF = 36.7 & EF \\
\hline
Triangle AE'F' & AE' = 0.57 & AF' & E'F' = 41.4 \\
\hline
\end{tabular}
est un tableau de proportionnalité. Donc on peut faire un produit en croix pour calcul $EF$.
\begin{align*}
EF = \frac{E'F' \times AE}{AE'} = \frac{41.4 \times 1.7}{0.57} = 124.2 m
\end{align*}
Donc le parcours AEFA mesure
\begin{align*}
AF + AE + EF = 36.7 + 1.7 + 124.2 = 162.6m
\end{align*}
\item Choix du parcours:
Il faudra choisir le tour $ACDA$ car sa longueur est plus proche de 167m.
\end{itemize}
\end{solution}
\end{exercise}
\begin{exercise}[subtitle={Tirages au sort}]
\renewcommand{\arraystretch}{1.5}
Dans une urne, on a placé des boules colorées indiscernables au touché. Il y a 10 boules bleu, 6 boules jaunes, 9 boules vertes et 7 boules rouges.
\begin{enumerate}
\item % Proba
\begin{enumerate}
\item Quelle est la probabilité de tirer une boule bleu?
\begin{solution}
$\dfrac{10}{32} \approx 0.31$
\end{solution}
\item Quelle est la probabilté de tirer une boule jaune ou bleu?
\begin{solution}
$\dfrac{16}{32} \approx 0.5$
\end{solution}
\item A-t-on plus de chance de tirer une boule verte ou une boule rouge?
\begin{solution}
Boules vertes: $\dfrac{9}{32} \approx 0.28$
Boules rouges: $\dfrac{7}{32} \approx 0.22$
Une boule verte
\end{solution}
\end{enumerate}
\item % Stat
On effectue 14 tirages (avec remise) dans cette urne et on obtient les couleurs suivantes:
\begin{center}
R \hspace{0.4cm}V \hspace{0.4cm}R \hspace{0.4cm}V \hspace{0.4cm}V \hspace{0.4cm}B \hspace{0.4cm}J \hspace{0.4cm}R \hspace{0.4cm}V \hspace{0.4cm}B \hspace{0.4cm}B \hspace{0.4cm}V \hspace{0.4cm}V \hspace{0.4cm}B \\
\end{center}
\begin{enumerate}
\item Compléter le tableau des effectifs ci-dessous
\begin{tabular}{|c|*{4}{c|}}
\hline
Couleur & Bleu & Jaune & Vert & Rouge \\
\hline
Effectif & & & & \\
\hline
\end{tabular}
\begin{solution}
\begin{tabular}{|c|*{4}{c|}}
\hline
Couleur & Bleu & Jaune & Vert & Rouge \\
\hline
Effectif & 4 & 1 & 6 & 3 \\
\hline
\end{tabular}
\end{solution}
\item Calculer la fréquence des boules vertes.
\begin{solution}
Fréquence de boules vertes: $\frac{6}{14}$
\end{solution}
\end{enumerate}
\item À chaque couleur, on associe des points. Une boule bleu rapporte 10 points, une boule jaune 5 points, une boule verte 2 points et une boule rouge 0 points.
\begin{enumerate}
\item Combien de points a-t-on gagné au total?
\begin{solution}
17
\end{solution}
\item Calculer la moyenne des gains.
\begin{solution}
4.25
\end{solution}
\item Calculer la médiane des gains.
\begin{solution}
3.5
\end{solution}
\end{enumerate}
\end{enumerate}
\end{exercise}
\end{document}
%%% Local Variables:
%%% mode: latex
%%% TeX-master: "master"
%%% End:

View File

@ -0,0 +1,278 @@
\documentclass[a4paper,12pt]{article}
\usepackage{myXsim}
\usepackage{tkz-fct}
\usepackage{wrapfig}
\title{DM de Paques}
\tribe{302}
\date{Jeudi 3 mai 2018}
\sujet{07}
%\geometry{left=10mm,right=10mm, bottom= 10mm, top=10mm}
\xsimsetup{
solution/print = false
}
\begin{document}
\maketitle
\bigskip
{\Large \textbf{Nom - Prénom:}}
\begin{exercise}[subtitle={Battle of the year}]
% theme: Fonction linéaire, Fonction affine
% require: tkz-fct
Taraina veut inscrire ses 21 élèves à un entrainement pour l'évènement \textbf{Battle of the year}.
Deux tarifs lui sont proposés:
\begin{itemize}
\item Tarif Individuel: 154 \euro par danseur inscrit.
\item Tarif Groupe : Paiement d'un forfait de 556 \euro pour le groupe puis 120 \euro par danseur inscrit.
\end{itemize}
\medskip
\begin{enumerate}
\item Complète le tableau suivant :
\medskip
\begin{tabularx}{0.7\linewidth}{|c|*{3}{>{\centering \arraybackslash}X|}}\hline
Nombre d'inscriptions & 0 & 10 & 25\\
\hline
Prix au tarif Individuel en \euro & & & \\
\hline
Prix au tarif Groupe en \euro & & & \\
\hline
\end{tabularx}
\begin{solution}
\begin{tabularx}{0.7\linewidth}{|c|*{3}{>{\centering \arraybackslash}X|}}\hline
Nombre d'inscriptions & 0 & 10 & 25\\
\hline
Prix au tarif Individuel en \euro & 0 & 1540 & 3850 \\
\hline
Prix au tarif Groupe en \euro & 556 & 1756 & 3556\\
\hline
\end{tabularx}
\end{solution}
\medskip
\item Pour chacun des tarifs, exprimer le prix en fonction du nombre de danseurs inscrits.
\begin{solution}
$x$ représente ici le nombre d'élèves inscrits.
\begin{itemize}
\item Tarif Individuel: $f: x \mapsto 154x$
\item Tarif Groupe: $g: x \mapsto 120x + 556$
\end{itemize}
\end{solution}
\item Tracer sur le graphique suivant, les courbes représentants les 2 tarifs proposés.
% On force que le graphique soit légèrement plus grand que 12 cm
\begin{tikzpicture}[yscale=0.6]
\tkzInit[xmin=0,xmax=26,
ymin=0,ymax=4004,
xstep=2,ystep=200]
\tkzAxeX[thick, poslabel=right,label=]
\tkzAxeY[thick, poslabel=above,label=]
\tkzDrawX[label={\textit{Danseurs inscrits}},below= -12pt]
\tkzDrawY[label={\textit{Prix}}, below=-10pt]
\tkzGrid
\tkzFct[domain=0:26, color=blue, very thick]{154*\x}
\tkzFct[domain=0:26, color=red, very thick]{120*\x+556}
\end{tikzpicture}
\item Pour quel nombre d'inscriptions paye-t-on le même prix quel que soit le tarif choisi?
\begin{solution}
$16.352941176470587$ % est entre 10 et 20 par contruction des paramètres
\end{solution}
\end{enumerate}
\end{exercise}
\begin{exercise}[subtitle={Parcours}]
% exo de geometrie comme au brevet blanc.
Une commune souhaite aménager des parcours de santé sur son territoire. On fait deux propositions au conseil municipale, schématisés ci-dessous:
\begin{itemize}
\item Le parcours ACDA
\item Le parcours AEFA
\end{itemize}
Ils souhaitent faire un parcours dont la longueur s'approche le plus possible de 118m.
Peux-tu les aider à choisir le parcours? Justifie
\textbf{Attention: La figure proposée au conseil municipale n'est pas à l'échelle, mais les codages et les dimension données sont correctes.}
\begin{minipage}{0.6\textwidth}
\includegraphics[scale = 0.4]{./fig/parcours}
\end{minipage}
\begin{minipage}{0.4\textwidth}
\begin{itemize}
\item $AC = 28m$
\item $CD = 45m$
\item $AE' = 25.6m$
\item $AE = 51.2m$
\item $AF = 3.4m$
\item $E'F' = 31.7m$
\item $(E'F') // (EF)$
\item L'angle $\widehat{EAF}$ vaut $30^o$
\end{itemize}
\end{minipage}
\begin{solution}
\begin{itemize}
\item Parcours ACDA:
D'après la figure, on voit que le triangle $ACD$ est rectangle en $C$ donc d'après le théorème de Pythagore, on a
\begin{align*}
AD^2 &= AC^2 + DC^2 \\
AD^2 &= 28^2 + 45^2 \\
AD^2 &= 784 + 2025 \\
AD^2 &= 2809 \\
AD &= \sqrt{2809} = 53m
\end{align*}
Donc le parcours ACDA mesure
\begin{align*}
AD + AC + CD = 53 + 28 + 45 = 126m
\end{align*}
\item Parcours AEFA:
D'après les données, on sait que $(EF) // (E'F')$. On voit aussi que $A$, $E'$ et $E$ sont alignés. Il en est de même pour les points $A$, $F'$ et $F$. Donc d'après le théorème de Thalès
\begin{tabular}{|c|c|c|c|}
\hline
Triangle AEF & AE = 51.2 & AF = 3.4 & EF \\
\hline
Triangle AE'F' & AE' = 25.6 & AF' & E'F' = 31.7 \\
\hline
\end{tabular}
est un tableau de proportionnalité. Donc on peut faire un produit en croix pour calcul $EF$.
\begin{align*}
EF = \frac{E'F' \times AE}{AE'} = \frac{31.7 \times 51.2}{25.6} = 63.4 m
\end{align*}
Donc le parcours AEFA mesure
\begin{align*}
AF + AE + EF = 3.4 + 51.2 + 63.4 = 118.0m
\end{align*}
\item Choix du parcours:
Il faudra choisir le tour $AFEA$ car sa longueur est plus proche de 118m.
\end{itemize}
\end{solution}
\end{exercise}
\begin{exercise}[subtitle={Tirages au sort}]
\renewcommand{\arraystretch}{1.5}
Dans une urne, on a placé des boules colorées indiscernables au touché. Il y a 8 boules bleu, 8 boules jaunes, 9 boules vertes et 8 boules rouges.
\begin{enumerate}
\item % Proba
\begin{enumerate}
\item Quelle est la probabilité de tirer une boule bleu?
\begin{solution}
$\dfrac{8}{33} \approx 0.24$
\end{solution}
\item Quelle est la probabilté de tirer une boule jaune ou bleu?
\begin{solution}
$\dfrac{16}{33} \approx 0.48$
\end{solution}
\item A-t-on plus de chance de tirer une boule verte ou une boule rouge?
\begin{solution}
Boules vertes: $\dfrac{9}{33} \approx 0.27$
Boules rouges: $\dfrac{8}{33} \approx 0.24$
Une boule verte
\end{solution}
\end{enumerate}
\item % Stat
On effectue 14 tirages (avec remise) dans cette urne et on obtient les couleurs suivantes:
\begin{center}
B \hspace{0.4cm}B \hspace{0.4cm}J \hspace{0.4cm}J \hspace{0.4cm}J \hspace{0.4cm}V \hspace{0.4cm}V \hspace{0.4cm}V \hspace{0.4cm}R \hspace{0.4cm}B \hspace{0.4cm}V \hspace{0.4cm}R \hspace{0.4cm}V \hspace{0.4cm}J \\
\end{center}
\begin{enumerate}
\item Compléter le tableau des effectifs ci-dessous
\begin{tabular}{|c|*{4}{c|}}
\hline
Couleur & Bleu & Jaune & Vert & Rouge \\
\hline
Effectif & & & & \\
\hline
\end{tabular}
\begin{solution}
\begin{tabular}{|c|*{4}{c|}}
\hline
Couleur & Bleu & Jaune & Vert & Rouge \\
\hline
Effectif & 3 & 4 & 5 & 2 \\
\hline
\end{tabular}
\end{solution}
\item Calculer la fréquence des boules vertes.
\begin{solution}
Fréquence de boules vertes: $\frac{5}{14}$
\end{solution}
\end{enumerate}
\item À chaque couleur, on associe des points. Une boule bleu rapporte 10 points, une boule jaune 5 points, une boule verte 2 points et une boule rouge 0 points.
\begin{enumerate}
\item Combien de points a-t-on gagné au total?
\begin{solution}
17
\end{solution}
\item Calculer la moyenne des gains.
\begin{solution}
4.25
\end{solution}
\item Calculer la médiane des gains.
\begin{solution}
3.5
\end{solution}
\end{enumerate}
\end{enumerate}
\end{exercise}
\end{document}
%%% Local Variables:
%%% mode: latex
%%% TeX-master: "master"
%%% End:

View File

@ -0,0 +1,278 @@
\documentclass[a4paper,12pt]{article}
\usepackage{myXsim}
\usepackage{tkz-fct}
\usepackage{wrapfig}
\title{DM de Paques}
\tribe{302}
\date{Jeudi 3 mai 2018}
\sujet{08}
%\geometry{left=10mm,right=10mm, bottom= 10mm, top=10mm}
\xsimsetup{
solution/print = false
}
\begin{document}
\maketitle
\bigskip
{\Large \textbf{Nom - Prénom:}}
\begin{exercise}[subtitle={Battle of the year}]
% theme: Fonction linéaire, Fonction affine
% require: tkz-fct
Taraina veut inscrire ses 21 élèves à un entrainement pour l'évènement \textbf{Battle of the year}.
Deux tarifs lui sont proposés:
\begin{itemize}
\item Tarif Individuel: 125 \euro par danseur inscrit.
\item Tarif Groupe : Paiement d'un forfait de 384 \euro pour le groupe puis 87 \euro par danseur inscrit.
\end{itemize}
\medskip
\begin{enumerate}
\item Complète le tableau suivant :
\medskip
\begin{tabularx}{0.7\linewidth}{|c|*{3}{>{\centering \arraybackslash}X|}}\hline
Nombre d'inscriptions & 0 & 10 & 25\\
\hline
Prix au tarif Individuel en \euro & & & \\
\hline
Prix au tarif Groupe en \euro & & & \\
\hline
\end{tabularx}
\begin{solution}
\begin{tabularx}{0.7\linewidth}{|c|*{3}{>{\centering \arraybackslash}X|}}\hline
Nombre d'inscriptions & 0 & 10 & 25\\
\hline
Prix au tarif Individuel en \euro & 0 & 1250 & 3125 \\
\hline
Prix au tarif Groupe en \euro & 384 & 1254 & 2559\\
\hline
\end{tabularx}
\end{solution}
\medskip
\item Pour chacun des tarifs, exprimer le prix en fonction du nombre de danseurs inscrits.
\begin{solution}
$x$ représente ici le nombre d'élèves inscrits.
\begin{itemize}
\item Tarif Individuel: $f: x \mapsto 125x$
\item Tarif Groupe: $g: x \mapsto 87x + 384$
\end{itemize}
\end{solution}
\item Tracer sur le graphique suivant, les courbes représentants les 2 tarifs proposés.
% On force que le graphique soit légèrement plus grand que 12 cm
\begin{tikzpicture}[yscale=0.7]
\tkzInit[xmin=0,xmax=26,
ymin=0,ymax=3250,
xstep=2,ystep=200]
\tkzAxeX[thick, poslabel=right,label=]
\tkzAxeY[thick, poslabel=above,label=]
\tkzDrawX[label={\textit{Danseurs inscrits}},below= -12pt]
\tkzDrawY[label={\textit{Prix}}, below=-10pt]
\tkzGrid
\tkzFct[domain=0:26, color=blue, very thick]{125*\x}
\tkzFct[domain=0:26, color=red, very thick]{87*\x+384}
\end{tikzpicture}
\item Pour quel nombre d'inscriptions paye-t-on le même prix quel que soit le tarif choisi?
\begin{solution}
$10.105263157894736$ % est entre 10 et 20 par contruction des paramètres
\end{solution}
\end{enumerate}
\end{exercise}
\begin{exercise}[subtitle={Parcours}]
% exo de geometrie comme au brevet blanc.
Une commune souhaite aménager des parcours de santé sur son territoire. On fait deux propositions au conseil municipale, schématisés ci-dessous:
\begin{itemize}
\item Le parcours ACDA
\item Le parcours AEFA
\end{itemize}
Ils souhaitent faire un parcours dont la longueur s'approche le plus possible de 219m.
Peux-tu les aider à choisir le parcours? Justifie
\textbf{Attention: La figure proposée au conseil municipale n'est pas à l'échelle, mais les codages et les dimension données sont correctes.}
\begin{minipage}{0.6\textwidth}
\includegraphics[scale = 0.4]{./fig/parcours}
\end{minipage}
\begin{minipage}{0.4\textwidth}
\begin{itemize}
\item $AC = 96m$
\item $CD = 28m$
\item $AE' = 36.15m$
\item $AE = 72.3m$
\item $AF = 7.7m$
\item $E'F' = 69.8m$
\item $(E'F') // (EF)$
\item L'angle $\widehat{EAF}$ vaut $30^o$
\end{itemize}
\end{minipage}
\begin{solution}
\begin{itemize}
\item Parcours ACDA:
D'après la figure, on voit que le triangle $ACD$ est rectangle en $C$ donc d'après le théorème de Pythagore, on a
\begin{align*}
AD^2 &= AC^2 + DC^2 \\
AD^2 &= 96^2 + 28^2 \\
AD^2 &= 9216 + 784 \\
AD^2 &= 10000 \\
AD &= \sqrt{10000} = 100m
\end{align*}
Donc le parcours ACDA mesure
\begin{align*}
AD + AC + CD = 100 + 96 + 28 = 224m
\end{align*}
\item Parcours AEFA:
D'après les données, on sait que $(EF) // (E'F')$. On voit aussi que $A$, $E'$ et $E$ sont alignés. Il en est de même pour les points $A$, $F'$ et $F$. Donc d'après le théorème de Thalès
\begin{tabular}{|c|c|c|c|}
\hline
Triangle AEF & AE = 72.3 & AF = 7.7 & EF \\
\hline
Triangle AE'F' & AE' = 36.15 & AF' & E'F' = 69.8 \\
\hline
\end{tabular}
est un tableau de proportionnalité. Donc on peut faire un produit en croix pour calcul $EF$.
\begin{align*}
EF = \frac{E'F' \times AE}{AE'} = \frac{69.8 \times 72.3}{36.15} = 139.6 m
\end{align*}
Donc le parcours AEFA mesure
\begin{align*}
AF + AE + EF = 7.7 + 72.3 + 139.6 = 219.6m
\end{align*}
\item Choix du parcours:
Il faudra choisir le tour $AFEA$ car sa longueur est plus proche de 219m.
\end{itemize}
\end{solution}
\end{exercise}
\begin{exercise}[subtitle={Tirages au sort}]
\renewcommand{\arraystretch}{1.5}
Dans une urne, on a placé des boules colorées indiscernables au touché. Il y a 10 boules bleu, 7 boules jaunes, 10 boules vertes et 6 boules rouges.
\begin{enumerate}
\item % Proba
\begin{enumerate}
\item Quelle est la probabilité de tirer une boule bleu?
\begin{solution}
$\dfrac{10}{33} \approx 0.3$
\end{solution}
\item Quelle est la probabilté de tirer une boule jaune ou bleu?
\begin{solution}
$\dfrac{17}{33} \approx 0.52$
\end{solution}
\item A-t-on plus de chance de tirer une boule verte ou une boule rouge?
\begin{solution}
Boules vertes: $\dfrac{10}{33} \approx 0.3$
Boules rouges: $\dfrac{6}{33} \approx 0.18$
Une boule verte
\end{solution}
\end{enumerate}
\item % Stat
On effectue 14 tirages (avec remise) dans cette urne et on obtient les couleurs suivantes:
\begin{center}
V \hspace{0.4cm}B \hspace{0.4cm}B \hspace{0.4cm}V \hspace{0.4cm}V \hspace{0.4cm}B \hspace{0.4cm}B \hspace{0.4cm}B \hspace{0.4cm}J \hspace{0.4cm}B \hspace{0.4cm}J \hspace{0.4cm}V \hspace{0.4cm}V \hspace{0.4cm}J \\
\end{center}
\begin{enumerate}
\item Compléter le tableau des effectifs ci-dessous
\begin{tabular}{|c|*{4}{c|}}
\hline
Couleur & Bleu & Jaune & Vert & Rouge \\
\hline
Effectif & & & & \\
\hline
\end{tabular}
\begin{solution}
\begin{tabular}{|c|*{4}{c|}}
\hline
Couleur & Bleu & Jaune & Vert & Rouge \\
\hline
Effectif & 6 & 3 & 5 & \\
\hline
\end{tabular}
\end{solution}
\item Calculer la fréquence des boules vertes.
\begin{solution}
Fréquence de boules vertes: $\frac{5}{14}$
\end{solution}
\end{enumerate}
\item À chaque couleur, on associe des points. Une boule bleu rapporte 10 points, une boule jaune 5 points, une boule verte 2 points et une boule rouge 0 points.
\begin{enumerate}
\item Combien de points a-t-on gagné au total?
\begin{solution}
17
\end{solution}
\item Calculer la moyenne des gains.
\begin{solution}
4.25
\end{solution}
\item Calculer la médiane des gains.
\begin{solution}
3.5
\end{solution}
\end{enumerate}
\end{enumerate}
\end{exercise}
\end{document}
%%% Local Variables:
%%% mode: latex
%%% TeX-master: "master"
%%% End:

View File

@ -0,0 +1,278 @@
\documentclass[a4paper,12pt]{article}
\usepackage{myXsim}
\usepackage{tkz-fct}
\usepackage{wrapfig}
\title{DM de Paques}
\tribe{302}
\date{Jeudi 3 mai 2018}
\sujet{09}
%\geometry{left=10mm,right=10mm, bottom= 10mm, top=10mm}
\xsimsetup{
solution/print = false
}
\begin{document}
\maketitle
\bigskip
{\Large \textbf{Nom - Prénom:}}
\begin{exercise}[subtitle={Battle of the year}]
% theme: Fonction linéaire, Fonction affine
% require: tkz-fct
Taraina veut inscrire ses 21 élèves à un entrainement pour l'évènement \textbf{Battle of the year}.
Deux tarifs lui sont proposés:
\begin{itemize}
\item Tarif Individuel: 184 \euro par danseur inscrit.
\item Tarif Groupe : Paiement d'un forfait de 688 \euro pour le groupe puis 146 \euro par danseur inscrit.
\end{itemize}
\medskip
\begin{enumerate}
\item Complète le tableau suivant :
\medskip
\begin{tabularx}{0.7\linewidth}{|c|*{3}{>{\centering \arraybackslash}X|}}\hline
Nombre d'inscriptions & 0 & 10 & 25\\
\hline
Prix au tarif Individuel en \euro & & & \\
\hline
Prix au tarif Groupe en \euro & & & \\
\hline
\end{tabularx}
\begin{solution}
\begin{tabularx}{0.7\linewidth}{|c|*{3}{>{\centering \arraybackslash}X|}}\hline
Nombre d'inscriptions & 0 & 10 & 25\\
\hline
Prix au tarif Individuel en \euro & 0 & 1840 & 4600 \\
\hline
Prix au tarif Groupe en \euro & 688 & 2148 & 4338\\
\hline
\end{tabularx}
\end{solution}
\medskip
\item Pour chacun des tarifs, exprimer le prix en fonction du nombre de danseurs inscrits.
\begin{solution}
$x$ représente ici le nombre d'élèves inscrits.
\begin{itemize}
\item Tarif Individuel: $f: x \mapsto 184x$
\item Tarif Groupe: $g: x \mapsto 146x + 688$
\end{itemize}
\end{solution}
\item Tracer sur le graphique suivant, les courbes représentants les 2 tarifs proposés.
% On force que le graphique soit légèrement plus grand que 12 cm
\begin{tikzpicture}[yscale=0.5]
\tkzInit[xmin=0,xmax=26,
ymin=0,ymax=4784,
xstep=2,ystep=200]
\tkzAxeX[thick, poslabel=right,label=]
\tkzAxeY[thick, poslabel=above,label=]
\tkzDrawX[label={\textit{Danseurs inscrits}},below= -12pt]
\tkzDrawY[label={\textit{Prix}}, below=-10pt]
\tkzGrid
\tkzFct[domain=0:26, color=blue, very thick]{184*\x}
\tkzFct[domain=0:26, color=red, very thick]{146*\x+688}
\end{tikzpicture}
\item Pour quel nombre d'inscriptions paye-t-on le même prix quel que soit le tarif choisi?
\begin{solution}
$18.105263157894736$ % est entre 10 et 20 par contruction des paramètres
\end{solution}
\end{enumerate}
\end{exercise}
\begin{exercise}[subtitle={Parcours}]
% exo de geometrie comme au brevet blanc.
Une commune souhaite aménager des parcours de santé sur son territoire. On fait deux propositions au conseil municipale, schématisés ci-dessous:
\begin{itemize}
\item Le parcours ACDA
\item Le parcours AEFA
\end{itemize}
Ils souhaitent faire un parcours dont la longueur s'approche le plus possible de 90km.
Peux-tu les aider à choisir le parcours? Justifie
\textbf{Attention: La figure proposée au conseil municipale n'est pas à l'échelle, mais les codages et les dimension données sont correctes.}
\begin{minipage}{0.6\textwidth}
\includegraphics[scale = 0.4]{./fig/parcours}
\end{minipage}
\begin{minipage}{0.4\textwidth}
\begin{itemize}
\item $AC = 24km$
\item $CD = 32km$
\item $AE' = 2.82km$
\item $AE = 14.1km$
\item $AF = 18.4km$
\item $E'F' = 11.58km$
\item $(E'F') // (EF)$
\item L'angle $\widehat{EAF}$ vaut $30^o$
\end{itemize}
\end{minipage}
\begin{solution}
\begin{itemize}
\item Parcours ACDA:
D'après la figure, on voit que le triangle $ACD$ est rectangle en $C$ donc d'après le théorème de Pythagore, on a
\begin{align*}
AD^2 &= AC^2 + DC^2 \\
AD^2 &= 24^2 + 32^2 \\
AD^2 &= 576 + 1024 \\
AD^2 &= 1600 \\
AD &= \sqrt{1600} = 40km
\end{align*}
Donc le parcours ACDA mesure
\begin{align*}
AD + AC + CD = 40 + 24 + 32 = 96km
\end{align*}
\item Parcours AEFA:
D'après les données, on sait que $(EF) // (E'F')$. On voit aussi que $A$, $E'$ et $E$ sont alignés. Il en est de même pour les points $A$, $F'$ et $F$. Donc d'après le théorème de Thalès
\begin{tabular}{|c|c|c|c|}
\hline
Triangle AEF & AE = 14.1 & AF = 18.4 & EF \\
\hline
Triangle AE'F' & AE' = 2.82 & AF' & E'F' = 11.58 \\
\hline
\end{tabular}
est un tableau de proportionnalité. Donc on peut faire un produit en croix pour calcul $EF$.
\begin{align*}
EF = \frac{E'F' \times AE}{AE'} = \frac{11.58 \times 14.1}{2.82} = 57.9 km
\end{align*}
Donc le parcours AEFA mesure
\begin{align*}
AF + AE + EF = 18.4 + 14.1 + 57.9 = 90.4km
\end{align*}
\item Choix du parcours:
Il faudra choisir le tour $AFEA$ car sa longueur est plus proche de 90km.
\end{itemize}
\end{solution}
\end{exercise}
\begin{exercise}[subtitle={Tirages au sort}]
\renewcommand{\arraystretch}{1.5}
Dans une urne, on a placé des boules colorées indiscernables au touché. Il y a 7 boules bleu, 6 boules jaunes, 4 boules vertes et 2 boules rouges.
\begin{enumerate}
\item % Proba
\begin{enumerate}
\item Quelle est la probabilité de tirer une boule bleu?
\begin{solution}
$\dfrac{7}{19} \approx 0.37$
\end{solution}
\item Quelle est la probabilté de tirer une boule jaune ou bleu?
\begin{solution}
$\dfrac{13}{19} \approx 0.68$
\end{solution}
\item A-t-on plus de chance de tirer une boule verte ou une boule rouge?
\begin{solution}
Boules vertes: $\dfrac{4}{19} \approx 0.21$
Boules rouges: $\dfrac{2}{19} \approx 0.11$
Une boule verte
\end{solution}
\end{enumerate}
\item % Stat
On effectue 14 tirages (avec remise) dans cette urne et on obtient les couleurs suivantes:
\begin{center}
J \hspace{0.4cm}B \hspace{0.4cm}V \hspace{0.4cm}J \hspace{0.4cm}V \hspace{0.4cm}J \hspace{0.4cm}B \hspace{0.4cm}B \hspace{0.4cm}J \hspace{0.4cm}V \hspace{0.4cm}B \hspace{0.4cm}B \hspace{0.4cm}V \hspace{0.4cm}B \\
\end{center}
\begin{enumerate}
\item Compléter le tableau des effectifs ci-dessous
\begin{tabular}{|c|*{4}{c|}}
\hline
Couleur & Bleu & Jaune & Vert & Rouge \\
\hline
Effectif & & & & \\
\hline
\end{tabular}
\begin{solution}
\begin{tabular}{|c|*{4}{c|}}
\hline
Couleur & Bleu & Jaune & Vert & Rouge \\
\hline
Effectif & 6 & 4 & 4 & \\
\hline
\end{tabular}
\end{solution}
\item Calculer la fréquence des boules vertes.
\begin{solution}
Fréquence de boules vertes: $\frac{4}{14}$
\end{solution}
\end{enumerate}
\item À chaque couleur, on associe des points. Une boule bleu rapporte 10 points, une boule jaune 5 points, une boule verte 2 points et une boule rouge 0 points.
\begin{enumerate}
\item Combien de points a-t-on gagné au total?
\begin{solution}
17
\end{solution}
\item Calculer la moyenne des gains.
\begin{solution}
4.25
\end{solution}
\item Calculer la médiane des gains.
\begin{solution}
3.5
\end{solution}
\end{enumerate}
\end{enumerate}
\end{exercise}
\end{document}
%%% Local Variables:
%%% mode: latex
%%% TeX-master: "master"
%%% End:

View File

@ -0,0 +1,278 @@
\documentclass[a4paper,12pt]{article}
\usepackage{myXsim}
\usepackage{tkz-fct}
\usepackage{wrapfig}
\title{DM de Paques}
\tribe{302}
\date{Jeudi 3 mai 2018}
\sujet{10}
%\geometry{left=10mm,right=10mm, bottom= 10mm, top=10mm}
\xsimsetup{
solution/print = false
}
\begin{document}
\maketitle
\bigskip
{\Large \textbf{Nom - Prénom:}}
\begin{exercise}[subtitle={Battle of the year}]
% theme: Fonction linéaire, Fonction affine
% require: tkz-fct
Taraina veut inscrire ses 21 élèves à un entrainement pour l'évènement \textbf{Battle of the year}.
Deux tarifs lui sont proposés:
\begin{itemize}
\item Tarif Individuel: 125 \euro par danseur inscrit.
\item Tarif Groupe : Paiement d'un forfait de 432 \euro pour le groupe puis 87 \euro par danseur inscrit.
\end{itemize}
\medskip
\begin{enumerate}
\item Complète le tableau suivant :
\medskip
\begin{tabularx}{0.7\linewidth}{|c|*{3}{>{\centering \arraybackslash}X|}}\hline
Nombre d'inscriptions & 0 & 10 & 25\\
\hline
Prix au tarif Individuel en \euro & & & \\
\hline
Prix au tarif Groupe en \euro & & & \\
\hline
\end{tabularx}
\begin{solution}
\begin{tabularx}{0.7\linewidth}{|c|*{3}{>{\centering \arraybackslash}X|}}\hline
Nombre d'inscriptions & 0 & 10 & 25\\
\hline
Prix au tarif Individuel en \euro & 0 & 1250 & 3125 \\
\hline
Prix au tarif Groupe en \euro & 432 & 1302 & 2607\\
\hline
\end{tabularx}
\end{solution}
\medskip
\item Pour chacun des tarifs, exprimer le prix en fonction du nombre de danseurs inscrits.
\begin{solution}
$x$ représente ici le nombre d'élèves inscrits.
\begin{itemize}
\item Tarif Individuel: $f: x \mapsto 125x$
\item Tarif Groupe: $g: x \mapsto 87x + 432$
\end{itemize}
\end{solution}
\item Tracer sur le graphique suivant, les courbes représentants les 2 tarifs proposés.
% On force que le graphique soit légèrement plus grand que 12 cm
\begin{tikzpicture}[yscale=0.7]
\tkzInit[xmin=0,xmax=26,
ymin=0,ymax=3250,
xstep=2,ystep=200]
\tkzAxeX[thick, poslabel=right,label=]
\tkzAxeY[thick, poslabel=above,label=]
\tkzDrawX[label={\textit{Danseurs inscrits}},below= -12pt]
\tkzDrawY[label={\textit{Prix}}, below=-10pt]
\tkzGrid
\tkzFct[domain=0:26, color=blue, very thick]{125*\x}
\tkzFct[domain=0:26, color=red, very thick]{87*\x+432}
\end{tikzpicture}
\item Pour quel nombre d'inscriptions paye-t-on le même prix quel que soit le tarif choisi?
\begin{solution}
$11.368421052631579$ % est entre 10 et 20 par contruction des paramètres
\end{solution}
\end{enumerate}
\end{exercise}
\begin{exercise}[subtitle={Parcours}]
% exo de geometrie comme au brevet blanc.
Une commune souhaite aménager des parcours de santé sur son territoire. On fait deux propositions au conseil municipale, schématisés ci-dessous:
\begin{itemize}
\item Le parcours ACDA
\item Le parcours AEFA
\end{itemize}
Ils souhaitent faire un parcours dont la longueur s'approche le plus possible de 233m.
Peux-tu les aider à choisir le parcours? Justifie
\textbf{Attention: La figure proposée au conseil municipale n'est pas à l'échelle, mais les codages et les dimension données sont correctes.}
\begin{minipage}{0.6\textwidth}
\includegraphics[scale = 0.4]{./fig/parcours}
\end{minipage}
\begin{minipage}{0.4\textwidth}
\begin{itemize}
\item $AC = 40m$
\item $CD = 96m$
\item $AE' = 13.47m$
\item $AE = 40.4m$
\item $AF = 113.6m$
\item $E'F' = 26.47m$
\item $(E'F') // (EF)$
\item L'angle $\widehat{EAF}$ vaut $30^o$
\end{itemize}
\end{minipage}
\begin{solution}
\begin{itemize}
\item Parcours ACDA:
D'après la figure, on voit que le triangle $ACD$ est rectangle en $C$ donc d'après le théorème de Pythagore, on a
\begin{align*}
AD^2 &= AC^2 + DC^2 \\
AD^2 &= 40^2 + 96^2 \\
AD^2 &= 1600 + 9216 \\
AD^2 &= 10816 \\
AD &= \sqrt{10816} = 104m
\end{align*}
Donc le parcours ACDA mesure
\begin{align*}
AD + AC + CD = 104 + 40 + 96 = 240m
\end{align*}
\item Parcours AEFA:
D'après les données, on sait que $(EF) // (E'F')$. On voit aussi que $A$, $E'$ et $E$ sont alignés. Il en est de même pour les points $A$, $F'$ et $F$. Donc d'après le théorème de Thalès
\begin{tabular}{|c|c|c|c|}
\hline
Triangle AEF & AE = 40.4 & AF = 113.6 & EF \\
\hline
Triangle AE'F' & AE' = 13.47 & AF' & E'F' = 26.47 \\
\hline
\end{tabular}
est un tableau de proportionnalité. Donc on peut faire un produit en croix pour calcul $EF$.
\begin{align*}
EF = \frac{E'F' \times AE}{AE'} = \frac{26.47 \times 40.4}{13.47} = 79.4 m
\end{align*}
Donc le parcours AEFA mesure
\begin{align*}
AF + AE + EF = 113.6 + 40.4 + 79.4 = 233.4m
\end{align*}
\item Choix du parcours:
Il faudra choisir le tour $AFEA$ car sa longueur est plus proche de 233m.
\end{itemize}
\end{solution}
\end{exercise}
\begin{exercise}[subtitle={Tirages au sort}]
\renewcommand{\arraystretch}{1.5}
Dans une urne, on a placé des boules colorées indiscernables au touché. Il y a 8 boules bleu, 7 boules jaunes, 7 boules vertes et 4 boules rouges.
\begin{enumerate}
\item % Proba
\begin{enumerate}
\item Quelle est la probabilité de tirer une boule bleu?
\begin{solution}
$\dfrac{8}{26} \approx 0.31$
\end{solution}
\item Quelle est la probabilté de tirer une boule jaune ou bleu?
\begin{solution}
$\dfrac{15}{26} \approx 0.58$
\end{solution}
\item A-t-on plus de chance de tirer une boule verte ou une boule rouge?
\begin{solution}
Boules vertes: $\dfrac{7}{26} \approx 0.27$
Boules rouges: $\dfrac{4}{26} \approx 0.15$
Une boule verte
\end{solution}
\end{enumerate}
\item % Stat
On effectue 14 tirages (avec remise) dans cette urne et on obtient les couleurs suivantes:
\begin{center}
R \hspace{0.4cm}R \hspace{0.4cm}V \hspace{0.4cm}V \hspace{0.4cm}R \hspace{0.4cm}B \hspace{0.4cm}J \hspace{0.4cm}B \hspace{0.4cm}J \hspace{0.4cm}R \hspace{0.4cm}B \hspace{0.4cm}R \hspace{0.4cm}R \hspace{0.4cm}B \\
\end{center}
\begin{enumerate}
\item Compléter le tableau des effectifs ci-dessous
\begin{tabular}{|c|*{4}{c|}}
\hline
Couleur & Bleu & Jaune & Vert & Rouge \\
\hline
Effectif & & & & \\
\hline
\end{tabular}
\begin{solution}
\begin{tabular}{|c|*{4}{c|}}
\hline
Couleur & Bleu & Jaune & Vert & Rouge \\
\hline
Effectif & 4 & 2 & 2 & 6 \\
\hline
\end{tabular}
\end{solution}
\item Calculer la fréquence des boules vertes.
\begin{solution}
Fréquence de boules vertes: $\frac{2}{14}$
\end{solution}
\end{enumerate}
\item À chaque couleur, on associe des points. Une boule bleu rapporte 10 points, une boule jaune 5 points, une boule verte 2 points et une boule rouge 0 points.
\begin{enumerate}
\item Combien de points a-t-on gagné au total?
\begin{solution}
17
\end{solution}
\item Calculer la moyenne des gains.
\begin{solution}
4.25
\end{solution}
\item Calculer la médiane des gains.
\begin{solution}
3.5
\end{solution}
\end{enumerate}
\end{enumerate}
\end{exercise}
\end{document}
%%% Local Variables:
%%% mode: latex
%%% TeX-master: "master"
%%% End:

View File

@ -0,0 +1,278 @@
\documentclass[a4paper,12pt]{article}
\usepackage{myXsim}
\usepackage{tkz-fct}
\usepackage{wrapfig}
\title{DM de Paques}
\tribe{302}
\date{Jeudi 3 mai 2018}
\sujet{11}
%\geometry{left=10mm,right=10mm, bottom= 10mm, top=10mm}
\xsimsetup{
solution/print = false
}
\begin{document}
\maketitle
\bigskip
{\Large \textbf{Nom - Prénom:}}
\begin{exercise}[subtitle={Battle of the year}]
% theme: Fonction linéaire, Fonction affine
% require: tkz-fct
Taraina veut inscrire ses 21 élèves à un entrainement pour l'évènement \textbf{Battle of the year}.
Deux tarifs lui sont proposés:
\begin{itemize}
\item Tarif Individuel: 154 \euro par danseur inscrit.
\item Tarif Groupe : Paiement d'un forfait de 527 \euro pour le groupe puis 120 \euro par danseur inscrit.
\end{itemize}
\medskip
\begin{enumerate}
\item Complète le tableau suivant :
\medskip
\begin{tabularx}{0.7\linewidth}{|c|*{3}{>{\centering \arraybackslash}X|}}\hline
Nombre d'inscriptions & 0 & 10 & 25\\
\hline
Prix au tarif Individuel en \euro & & & \\
\hline
Prix au tarif Groupe en \euro & & & \\
\hline
\end{tabularx}
\begin{solution}
\begin{tabularx}{0.7\linewidth}{|c|*{3}{>{\centering \arraybackslash}X|}}\hline
Nombre d'inscriptions & 0 & 10 & 25\\
\hline
Prix au tarif Individuel en \euro & 0 & 1540 & 3850 \\
\hline
Prix au tarif Groupe en \euro & 527 & 1727 & 3527\\
\hline
\end{tabularx}
\end{solution}
\medskip
\item Pour chacun des tarifs, exprimer le prix en fonction du nombre de danseurs inscrits.
\begin{solution}
$x$ représente ici le nombre d'élèves inscrits.
\begin{itemize}
\item Tarif Individuel: $f: x \mapsto 154x$
\item Tarif Groupe: $g: x \mapsto 120x + 527$
\end{itemize}
\end{solution}
\item Tracer sur le graphique suivant, les courbes représentants les 2 tarifs proposés.
% On force que le graphique soit légèrement plus grand que 12 cm
\begin{tikzpicture}[yscale=0.6]
\tkzInit[xmin=0,xmax=26,
ymin=0,ymax=4004,
xstep=2,ystep=200]
\tkzAxeX[thick, poslabel=right,label=]
\tkzAxeY[thick, poslabel=above,label=]
\tkzDrawX[label={\textit{Danseurs inscrits}},below= -12pt]
\tkzDrawY[label={\textit{Prix}}, below=-10pt]
\tkzGrid
\tkzFct[domain=0:26, color=blue, very thick]{154*\x}
\tkzFct[domain=0:26, color=red, very thick]{120*\x+527}
\end{tikzpicture}
\item Pour quel nombre d'inscriptions paye-t-on le même prix quel que soit le tarif choisi?
\begin{solution}
$15.5$ % est entre 10 et 20 par contruction des paramètres
\end{solution}
\end{enumerate}
\end{exercise}
\begin{exercise}[subtitle={Parcours}]
% exo de geometrie comme au brevet blanc.
Une commune souhaite aménager des parcours de santé sur son territoire. On fait deux propositions au conseil municipale, schématisés ci-dessous:
\begin{itemize}
\item Le parcours ACDA
\item Le parcours AEFA
\end{itemize}
Ils souhaitent faire un parcours dont la longueur s'approche le plus possible de 339m.
Peux-tu les aider à choisir le parcours? Justifie
\textbf{Attention: La figure proposée au conseil municipale n'est pas à l'échelle, mais les codages et les dimension données sont correctes.}
\begin{minipage}{0.6\textwidth}
\includegraphics[scale = 0.4]{./fig/parcours}
\end{minipage}
\begin{minipage}{0.4\textwidth}
\begin{itemize}
\item $AC = 140m$
\item $CD = 51m$
\item $AE' = 1.44m$
\item $AE = 7.2m$
\item $AF = 140.3m$
\item $E'F' = 36.98m$
\item $(E'F') // (EF)$
\item L'angle $\widehat{EAF}$ vaut $30^o$
\end{itemize}
\end{minipage}
\begin{solution}
\begin{itemize}
\item Parcours ACDA:
D'après la figure, on voit que le triangle $ACD$ est rectangle en $C$ donc d'après le théorème de Pythagore, on a
\begin{align*}
AD^2 &= AC^2 + DC^2 \\
AD^2 &= 140^2 + 51^2 \\
AD^2 &= 19600 + 2601 \\
AD^2 &= 22201 \\
AD &= \sqrt{22201} = 149m
\end{align*}
Donc le parcours ACDA mesure
\begin{align*}
AD + AC + CD = 149 + 140 + 51 = 340m
\end{align*}
\item Parcours AEFA:
D'après les données, on sait que $(EF) // (E'F')$. On voit aussi que $A$, $E'$ et $E$ sont alignés. Il en est de même pour les points $A$, $F'$ et $F$. Donc d'après le théorème de Thalès
\begin{tabular}{|c|c|c|c|}
\hline
Triangle AEF & AE = 7.2 & AF = 140.3 & EF \\
\hline
Triangle AE'F' & AE' = 1.44 & AF' & E'F' = 36.98 \\
\hline
\end{tabular}
est un tableau de proportionnalité. Donc on peut faire un produit en croix pour calcul $EF$.
\begin{align*}
EF = \frac{E'F' \times AE}{AE'} = \frac{36.98 \times 7.2}{1.44} = 184.9 m
\end{align*}
Donc le parcours AEFA mesure
\begin{align*}
AF + AE + EF = 140.3 + 7.2 + 184.9 = 332.4m
\end{align*}
\item Choix du parcours:
Il faudra choisir le tour $ACDA$ car sa longueur est plus proche de 339m.
\end{itemize}
\end{solution}
\end{exercise}
\begin{exercise}[subtitle={Tirages au sort}]
\renewcommand{\arraystretch}{1.5}
Dans une urne, on a placé des boules colorées indiscernables au touché. Il y a 10 boules bleu, 8 boules jaunes, 9 boules vertes et 5 boules rouges.
\begin{enumerate}
\item % Proba
\begin{enumerate}
\item Quelle est la probabilité de tirer une boule bleu?
\begin{solution}
$\dfrac{10}{32} \approx 0.31$
\end{solution}
\item Quelle est la probabilté de tirer une boule jaune ou bleu?
\begin{solution}
$\dfrac{18}{32} \approx 0.56$
\end{solution}
\item A-t-on plus de chance de tirer une boule verte ou une boule rouge?
\begin{solution}
Boules vertes: $\dfrac{9}{32} \approx 0.28$
Boules rouges: $\dfrac{5}{32} \approx 0.16$
Une boule verte
\end{solution}
\end{enumerate}
\item % Stat
On effectue 14 tirages (avec remise) dans cette urne et on obtient les couleurs suivantes:
\begin{center}
V \hspace{0.4cm}V \hspace{0.4cm}B \hspace{0.4cm}J \hspace{0.4cm}B \hspace{0.4cm}J \hspace{0.4cm}J \hspace{0.4cm}B \hspace{0.4cm}B \hspace{0.4cm}V \hspace{0.4cm}B \hspace{0.4cm}R \hspace{0.4cm}V \hspace{0.4cm}B \\
\end{center}
\begin{enumerate}
\item Compléter le tableau des effectifs ci-dessous
\begin{tabular}{|c|*{4}{c|}}
\hline
Couleur & Bleu & Jaune & Vert & Rouge \\
\hline
Effectif & & & & \\
\hline
\end{tabular}
\begin{solution}
\begin{tabular}{|c|*{4}{c|}}
\hline
Couleur & Bleu & Jaune & Vert & Rouge \\
\hline
Effectif & 6 & 3 & 4 & 1 \\
\hline
\end{tabular}
\end{solution}
\item Calculer la fréquence des boules vertes.
\begin{solution}
Fréquence de boules vertes: $\frac{4}{14}$
\end{solution}
\end{enumerate}
\item À chaque couleur, on associe des points. Une boule bleu rapporte 10 points, une boule jaune 5 points, une boule verte 2 points et une boule rouge 0 points.
\begin{enumerate}
\item Combien de points a-t-on gagné au total?
\begin{solution}
17
\end{solution}
\item Calculer la moyenne des gains.
\begin{solution}
4.25
\end{solution}
\item Calculer la médiane des gains.
\begin{solution}
3.5
\end{solution}
\end{enumerate}
\end{enumerate}
\end{exercise}
\end{document}
%%% Local Variables:
%%% mode: latex
%%% TeX-master: "master"
%%% End:

View File

@ -0,0 +1,278 @@
\documentclass[a4paper,12pt]{article}
\usepackage{myXsim}
\usepackage{tkz-fct}
\usepackage{wrapfig}
\title{DM de Paques}
\tribe{302}
\date{Jeudi 3 mai 2018}
\sujet{12}
%\geometry{left=10mm,right=10mm, bottom= 10mm, top=10mm}
\xsimsetup{
solution/print = false
}
\begin{document}
\maketitle
\bigskip
{\Large \textbf{Nom - Prénom:}}
\begin{exercise}[subtitle={Battle of the year}]
% theme: Fonction linéaire, Fonction affine
% require: tkz-fct
Taraina veut inscrire ses 21 élèves à un entrainement pour l'évènement \textbf{Battle of the year}.
Deux tarifs lui sont proposés:
\begin{itemize}
\item Tarif Individuel: 165 \euro par danseur inscrit.
\item Tarif Groupe : Paiement d'un forfait de 590 \euro pour le groupe puis 119 \euro par danseur inscrit.
\end{itemize}
\medskip
\begin{enumerate}
\item Complète le tableau suivant :
\medskip
\begin{tabularx}{0.7\linewidth}{|c|*{3}{>{\centering \arraybackslash}X|}}\hline
Nombre d'inscriptions & 0 & 10 & 25\\
\hline
Prix au tarif Individuel en \euro & & & \\
\hline
Prix au tarif Groupe en \euro & & & \\
\hline
\end{tabularx}
\begin{solution}
\begin{tabularx}{0.7\linewidth}{|c|*{3}{>{\centering \arraybackslash}X|}}\hline
Nombre d'inscriptions & 0 & 10 & 25\\
\hline
Prix au tarif Individuel en \euro & 0 & 1650 & 4125 \\
\hline
Prix au tarif Groupe en \euro & 590 & 1780 & 3565\\
\hline
\end{tabularx}
\end{solution}
\medskip
\item Pour chacun des tarifs, exprimer le prix en fonction du nombre de danseurs inscrits.
\begin{solution}
$x$ représente ici le nombre d'élèves inscrits.
\begin{itemize}
\item Tarif Individuel: $f: x \mapsto 165x$
\item Tarif Groupe: $g: x \mapsto 119x + 590$
\end{itemize}
\end{solution}
\item Tracer sur le graphique suivant, les courbes représentants les 2 tarifs proposés.
% On force que le graphique soit légèrement plus grand que 12 cm
\begin{tikzpicture}[yscale=0.6]
\tkzInit[xmin=0,xmax=26,
ymin=0,ymax=4290,
xstep=2,ystep=200]
\tkzAxeX[thick, poslabel=right,label=]
\tkzAxeY[thick, poslabel=above,label=]
\tkzDrawX[label={\textit{Danseurs inscrits}},below= -12pt]
\tkzDrawY[label={\textit{Prix}}, below=-10pt]
\tkzGrid
\tkzFct[domain=0:26, color=blue, very thick]{165*\x}
\tkzFct[domain=0:26, color=red, very thick]{119*\x+590}
\end{tikzpicture}
\item Pour quel nombre d'inscriptions paye-t-on le même prix quel que soit le tarif choisi?
\begin{solution}
$12.826086956521738$ % est entre 10 et 20 par contruction des paramètres
\end{solution}
\end{enumerate}
\end{exercise}
\begin{exercise}[subtitle={Parcours}]
% exo de geometrie comme au brevet blanc.
Une commune souhaite aménager des parcours de santé sur son territoire. On fait deux propositions au conseil municipale, schématisés ci-dessous:
\begin{itemize}
\item Le parcours ACDA
\item Le parcours AEFA
\end{itemize}
Ils souhaitent faire un parcours dont la longueur s'approche le plus possible de 175m.
Peux-tu les aider à choisir le parcours? Justifie
\textbf{Attention: La figure proposée au conseil municipale n'est pas à l'échelle, mais les codages et les dimension données sont correctes.}
\begin{minipage}{0.6\textwidth}
\includegraphics[scale = 0.4]{./fig/parcours}
\end{minipage}
\begin{minipage}{0.4\textwidth}
\begin{itemize}
\item $AC = 48m$
\item $CD = 55m$
\item $AE' = 14.3m$
\item $AE = 71.5m$
\item $AF = 1.3m$
\item $E'F' = 19.76m$
\item $(E'F') // (EF)$
\item L'angle $\widehat{EAF}$ vaut $30^o$
\end{itemize}
\end{minipage}
\begin{solution}
\begin{itemize}
\item Parcours ACDA:
D'après la figure, on voit que le triangle $ACD$ est rectangle en $C$ donc d'après le théorème de Pythagore, on a
\begin{align*}
AD^2 &= AC^2 + DC^2 \\
AD^2 &= 48^2 + 55^2 \\
AD^2 &= 2304 + 3025 \\
AD^2 &= 5329 \\
AD &= \sqrt{5329} = 73m
\end{align*}
Donc le parcours ACDA mesure
\begin{align*}
AD + AC + CD = 73 + 48 + 55 = 176m
\end{align*}
\item Parcours AEFA:
D'après les données, on sait que $(EF) // (E'F')$. On voit aussi que $A$, $E'$ et $E$ sont alignés. Il en est de même pour les points $A$, $F'$ et $F$. Donc d'après le théorème de Thalès
\begin{tabular}{|c|c|c|c|}
\hline
Triangle AEF & AE = 71.5 & AF = 1.3 & EF \\
\hline
Triangle AE'F' & AE' = 14.3 & AF' & E'F' = 19.76 \\
\hline
\end{tabular}
est un tableau de proportionnalité. Donc on peut faire un produit en croix pour calcul $EF$.
\begin{align*}
EF = \frac{E'F' \times AE}{AE'} = \frac{19.76 \times 71.5}{14.3} = 98.8 m
\end{align*}
Donc le parcours AEFA mesure
\begin{align*}
AF + AE + EF = 1.3 + 71.5 + 98.8 = 171.6m
\end{align*}
\item Choix du parcours:
Il faudra choisir le tour $ACDA$ car sa longueur est plus proche de 175m.
\end{itemize}
\end{solution}
\end{exercise}
\begin{exercise}[subtitle={Tirages au sort}]
\renewcommand{\arraystretch}{1.5}
Dans une urne, on a placé des boules colorées indiscernables au touché. Il y a 8 boules bleu, 7 boules jaunes, 4 boules vertes et 9 boules rouges.
\begin{enumerate}
\item % Proba
\begin{enumerate}
\item Quelle est la probabilité de tirer une boule bleu?
\begin{solution}
$\dfrac{8}{28} \approx 0.29$
\end{solution}
\item Quelle est la probabilté de tirer une boule jaune ou bleu?
\begin{solution}
$\dfrac{15}{28} \approx 0.54$
\end{solution}
\item A-t-on plus de chance de tirer une boule verte ou une boule rouge?
\begin{solution}
Boules vertes: $\dfrac{4}{28} \approx 0.14$
Boules rouges: $\dfrac{9}{28} \approx 0.32$
Une boule rouge
\end{solution}
\end{enumerate}
\item % Stat
On effectue 14 tirages (avec remise) dans cette urne et on obtient les couleurs suivantes:
\begin{center}
B \hspace{0.4cm}J \hspace{0.4cm}B \hspace{0.4cm}R \hspace{0.4cm}B \hspace{0.4cm}R \hspace{0.4cm}R \hspace{0.4cm}B \hspace{0.4cm}J \hspace{0.4cm}R \hspace{0.4cm}J \hspace{0.4cm}V \hspace{0.4cm}V \hspace{0.4cm}R \\
\end{center}
\begin{enumerate}
\item Compléter le tableau des effectifs ci-dessous
\begin{tabular}{|c|*{4}{c|}}
\hline
Couleur & Bleu & Jaune & Vert & Rouge \\
\hline
Effectif & & & & \\
\hline
\end{tabular}
\begin{solution}
\begin{tabular}{|c|*{4}{c|}}
\hline
Couleur & Bleu & Jaune & Vert & Rouge \\
\hline
Effectif & 4 & 3 & 2 & 5 \\
\hline
\end{tabular}
\end{solution}
\item Calculer la fréquence des boules vertes.
\begin{solution}
Fréquence de boules vertes: $\frac{2}{14}$
\end{solution}
\end{enumerate}
\item À chaque couleur, on associe des points. Une boule bleu rapporte 10 points, une boule jaune 5 points, une boule verte 2 points et une boule rouge 0 points.
\begin{enumerate}
\item Combien de points a-t-on gagné au total?
\begin{solution}
17
\end{solution}
\item Calculer la moyenne des gains.
\begin{solution}
4.25
\end{solution}
\item Calculer la médiane des gains.
\begin{solution}
3.5
\end{solution}
\end{enumerate}
\end{enumerate}
\end{exercise}
\end{document}
%%% Local Variables:
%%% mode: latex
%%% TeX-master: "master"
%%% End:

View File

@ -0,0 +1,278 @@
\documentclass[a4paper,12pt]{article}
\usepackage{myXsim}
\usepackage{tkz-fct}
\usepackage{wrapfig}
\title{DM de Paques}
\tribe{302}
\date{Jeudi 3 mai 2018}
\sujet{13}
%\geometry{left=10mm,right=10mm, bottom= 10mm, top=10mm}
\xsimsetup{
solution/print = false
}
\begin{document}
\maketitle
\bigskip
{\Large \textbf{Nom - Prénom:}}
\begin{exercise}[subtitle={Battle of the year}]
% theme: Fonction linéaire, Fonction affine
% require: tkz-fct
Taraina veut inscrire ses 21 élèves à un entrainement pour l'évènement \textbf{Battle of the year}.
Deux tarifs lui sont proposés:
\begin{itemize}
\item Tarif Individuel: 163 \euro par danseur inscrit.
\item Tarif Groupe : Paiement d'un forfait de 604 \euro pour le groupe puis 124 \euro par danseur inscrit.
\end{itemize}
\medskip
\begin{enumerate}
\item Complète le tableau suivant :
\medskip
\begin{tabularx}{0.7\linewidth}{|c|*{3}{>{\centering \arraybackslash}X|}}\hline
Nombre d'inscriptions & 0 & 10 & 25\\
\hline
Prix au tarif Individuel en \euro & & & \\
\hline
Prix au tarif Groupe en \euro & & & \\
\hline
\end{tabularx}
\begin{solution}
\begin{tabularx}{0.7\linewidth}{|c|*{3}{>{\centering \arraybackslash}X|}}\hline
Nombre d'inscriptions & 0 & 10 & 25\\
\hline
Prix au tarif Individuel en \euro & 0 & 1630 & 4075 \\
\hline
Prix au tarif Groupe en \euro & 604 & 1844 & 3704\\
\hline
\end{tabularx}
\end{solution}
\medskip
\item Pour chacun des tarifs, exprimer le prix en fonction du nombre de danseurs inscrits.
\begin{solution}
$x$ représente ici le nombre d'élèves inscrits.
\begin{itemize}
\item Tarif Individuel: $f: x \mapsto 163x$
\item Tarif Groupe: $g: x \mapsto 124x + 604$
\end{itemize}
\end{solution}
\item Tracer sur le graphique suivant, les courbes représentants les 2 tarifs proposés.
% On force que le graphique soit légèrement plus grand que 12 cm
\begin{tikzpicture}[yscale=0.6]
\tkzInit[xmin=0,xmax=26,
ymin=0,ymax=4238,
xstep=2,ystep=200]
\tkzAxeX[thick, poslabel=right,label=]
\tkzAxeY[thick, poslabel=above,label=]
\tkzDrawX[label={\textit{Danseurs inscrits}},below= -12pt]
\tkzDrawY[label={\textit{Prix}}, below=-10pt]
\tkzGrid
\tkzFct[domain=0:26, color=blue, very thick]{163*\x}
\tkzFct[domain=0:26, color=red, very thick]{124*\x+604}
\end{tikzpicture}
\item Pour quel nombre d'inscriptions paye-t-on le même prix quel que soit le tarif choisi?
\begin{solution}
$15.487179487179487$ % est entre 10 et 20 par contruction des paramètres
\end{solution}
\end{enumerate}
\end{exercise}
\begin{exercise}[subtitle={Parcours}]
% exo de geometrie comme au brevet blanc.
Une commune souhaite aménager des parcours de santé sur son territoire. On fait deux propositions au conseil municipale, schématisés ci-dessous:
\begin{itemize}
\item Le parcours ACDA
\item Le parcours AEFA
\end{itemize}
Ils souhaitent faire un parcours dont la longueur s'approche le plus possible de 6km.
Peux-tu les aider à choisir le parcours? Justifie
\textbf{Attention: La figure proposée au conseil municipale n'est pas à l'échelle, mais les codages et les dimension données sont correctes.}
\begin{minipage}{0.6\textwidth}
\includegraphics[scale = 0.4]{./fig/parcours}
\end{minipage}
\begin{minipage}{0.4\textwidth}
\begin{itemize}
\item $AC = 4km$
\item $CD = 3km$
\item $AE' = 1.02km$
\item $AE = 4.1km$
\item $AF = 0.2km$
\item $E'F' = 0.38km$
\item $(E'F') // (EF)$
\item L'angle $\widehat{EAF}$ vaut $30^o$
\end{itemize}
\end{minipage}
\begin{solution}
\begin{itemize}
\item Parcours ACDA:
D'après la figure, on voit que le triangle $ACD$ est rectangle en $C$ donc d'après le théorème de Pythagore, on a
\begin{align*}
AD^2 &= AC^2 + DC^2 \\
AD^2 &= 4^2 + 3^2 \\
AD^2 &= 16 + 9 \\
AD^2 &= 25 \\
AD &= \sqrt{25} = 5km
\end{align*}
Donc le parcours ACDA mesure
\begin{align*}
AD + AC + CD = 5 + 4 + 3 = 12km
\end{align*}
\item Parcours AEFA:
D'après les données, on sait que $(EF) // (E'F')$. On voit aussi que $A$, $E'$ et $E$ sont alignés. Il en est de même pour les points $A$, $F'$ et $F$. Donc d'après le théorème de Thalès
\begin{tabular}{|c|c|c|c|}
\hline
Triangle AEF & AE = 4.1 & AF = 0.2 & EF \\
\hline
Triangle AE'F' & AE' = 1.02 & AF' & E'F' = 0.38 \\
\hline
\end{tabular}
est un tableau de proportionnalité. Donc on peut faire un produit en croix pour calcul $EF$.
\begin{align*}
EF = \frac{E'F' \times AE}{AE'} = \frac{0.38 \times 4.1}{1.02} = 1.5 km
\end{align*}
Donc le parcours AEFA mesure
\begin{align*}
AF + AE + EF = 0.2 + 4.1 + 1.5 = 5.8km
\end{align*}
\item Choix du parcours:
Il faudra choisir le tour $AFEA$ car sa longueur est plus proche de 6km.
\end{itemize}
\end{solution}
\end{exercise}
\begin{exercise}[subtitle={Tirages au sort}]
\renewcommand{\arraystretch}{1.5}
Dans une urne, on a placé des boules colorées indiscernables au touché. Il y a 3 boules bleu, 7 boules jaunes, 8 boules vertes et 8 boules rouges.
\begin{enumerate}
\item % Proba
\begin{enumerate}
\item Quelle est la probabilité de tirer une boule bleu?
\begin{solution}
$\dfrac{3}{26} \approx 0.12$
\end{solution}
\item Quelle est la probabilté de tirer une boule jaune ou bleu?
\begin{solution}
$\dfrac{10}{26} \approx 0.38$
\end{solution}
\item A-t-on plus de chance de tirer une boule verte ou une boule rouge?
\begin{solution}
Boules vertes: $\dfrac{8}{26} \approx 0.31$
Boules rouges: $\dfrac{8}{26} \approx 0.31$
Une boule rouge
\end{solution}
\end{enumerate}
\item % Stat
On effectue 14 tirages (avec remise) dans cette urne et on obtient les couleurs suivantes:
\begin{center}
J \hspace{0.4cm}V \hspace{0.4cm}J \hspace{0.4cm}R \hspace{0.4cm}J \hspace{0.4cm}R \hspace{0.4cm}B \hspace{0.4cm}V \hspace{0.4cm}R \hspace{0.4cm}V \hspace{0.4cm}R \hspace{0.4cm}R \hspace{0.4cm}V \hspace{0.4cm}J \\
\end{center}
\begin{enumerate}
\item Compléter le tableau des effectifs ci-dessous
\begin{tabular}{|c|*{4}{c|}}
\hline
Couleur & Bleu & Jaune & Vert & Rouge \\
\hline
Effectif & & & & \\
\hline
\end{tabular}
\begin{solution}
\begin{tabular}{|c|*{4}{c|}}
\hline
Couleur & Bleu & Jaune & Vert & Rouge \\
\hline
Effectif & 1 & 4 & 4 & 5 \\
\hline
\end{tabular}
\end{solution}
\item Calculer la fréquence des boules vertes.
\begin{solution}
Fréquence de boules vertes: $\frac{4}{14}$
\end{solution}
\end{enumerate}
\item À chaque couleur, on associe des points. Une boule bleu rapporte 10 points, une boule jaune 5 points, une boule verte 2 points et une boule rouge 0 points.
\begin{enumerate}
\item Combien de points a-t-on gagné au total?
\begin{solution}
17
\end{solution}
\item Calculer la moyenne des gains.
\begin{solution}
4.25
\end{solution}
\item Calculer la médiane des gains.
\begin{solution}
3.5
\end{solution}
\end{enumerate}
\end{enumerate}
\end{exercise}
\end{document}
%%% Local Variables:
%%% mode: latex
%%% TeX-master: "master"
%%% End:

View File

@ -0,0 +1,278 @@
\documentclass[a4paper,12pt]{article}
\usepackage{myXsim}
\usepackage{tkz-fct}
\usepackage{wrapfig}
\title{DM de Paques}
\tribe{302}
\date{Jeudi 3 mai 2018}
\sujet{14}
%\geometry{left=10mm,right=10mm, bottom= 10mm, top=10mm}
\xsimsetup{
solution/print = false
}
\begin{document}
\maketitle
\bigskip
{\Large \textbf{Nom - Prénom:}}
\begin{exercise}[subtitle={Battle of the year}]
% theme: Fonction linéaire, Fonction affine
% require: tkz-fct
Taraina veut inscrire ses 21 élèves à un entrainement pour l'évènement \textbf{Battle of the year}.
Deux tarifs lui sont proposés:
\begin{itemize}
\item Tarif Individuel: 119 \euro par danseur inscrit.
\item Tarif Groupe : Paiement d'un forfait de 365 \euro pour le groupe puis 93 \euro par danseur inscrit.
\end{itemize}
\medskip
\begin{enumerate}
\item Complète le tableau suivant :
\medskip
\begin{tabularx}{0.7\linewidth}{|c|*{3}{>{\centering \arraybackslash}X|}}\hline
Nombre d'inscriptions & 0 & 10 & 25\\
\hline
Prix au tarif Individuel en \euro & & & \\
\hline
Prix au tarif Groupe en \euro & & & \\
\hline
\end{tabularx}
\begin{solution}
\begin{tabularx}{0.7\linewidth}{|c|*{3}{>{\centering \arraybackslash}X|}}\hline
Nombre d'inscriptions & 0 & 10 & 25\\
\hline
Prix au tarif Individuel en \euro & 0 & 1190 & 2975 \\
\hline
Prix au tarif Groupe en \euro & 365 & 1295 & 2690\\
\hline
\end{tabularx}
\end{solution}
\medskip
\item Pour chacun des tarifs, exprimer le prix en fonction du nombre de danseurs inscrits.
\begin{solution}
$x$ représente ici le nombre d'élèves inscrits.
\begin{itemize}
\item Tarif Individuel: $f: x \mapsto 119x$
\item Tarif Groupe: $g: x \mapsto 93x + 365$
\end{itemize}
\end{solution}
\item Tracer sur le graphique suivant, les courbes représentants les 2 tarifs proposés.
% On force que le graphique soit légèrement plus grand que 12 cm
\begin{tikzpicture}[yscale=0.8]
\tkzInit[xmin=0,xmax=26,
ymin=0,ymax=3094,
xstep=2,ystep=200]
\tkzAxeX[thick, poslabel=right,label=]
\tkzAxeY[thick, poslabel=above,label=]
\tkzDrawX[label={\textit{Danseurs inscrits}},below= -12pt]
\tkzDrawY[label={\textit{Prix}}, below=-10pt]
\tkzGrid
\tkzFct[domain=0:26, color=blue, very thick]{119*\x}
\tkzFct[domain=0:26, color=red, very thick]{93*\x+365}
\end{tikzpicture}
\item Pour quel nombre d'inscriptions paye-t-on le même prix quel que soit le tarif choisi?
\begin{solution}
$14.038461538461538$ % est entre 10 et 20 par contruction des paramètres
\end{solution}
\end{enumerate}
\end{exercise}
\begin{exercise}[subtitle={Parcours}]
% exo de geometrie comme au brevet blanc.
Une commune souhaite aménager des parcours de santé sur son territoire. On fait deux propositions au conseil municipale, schématisés ci-dessous:
\begin{itemize}
\item Le parcours ACDA
\item Le parcours AEFA
\end{itemize}
Ils souhaitent faire un parcours dont la longueur s'approche le plus possible de 377m.
Peux-tu les aider à choisir le parcours? Justifie
\textbf{Attention: La figure proposée au conseil municipale n'est pas à l'échelle, mais les codages et les dimension données sont correctes.}
\begin{minipage}{0.6\textwidth}
\includegraphics[scale = 0.4]{./fig/parcours}
\end{minipage}
\begin{minipage}{0.4\textwidth}
\begin{itemize}
\item $AC = 180m$
\item $CD = 19m$
\item $AE' = 31.3m$
\item $AE = 93.9m$
\item $AF = 88.2m$
\item $E'F' = 64.03m$
\item $(E'F') // (EF)$
\item L'angle $\widehat{EAF}$ vaut $30^o$
\end{itemize}
\end{minipage}
\begin{solution}
\begin{itemize}
\item Parcours ACDA:
D'après la figure, on voit que le triangle $ACD$ est rectangle en $C$ donc d'après le théorème de Pythagore, on a
\begin{align*}
AD^2 &= AC^2 + DC^2 \\
AD^2 &= 180^2 + 19^2 \\
AD^2 &= 32400 + 361 \\
AD^2 &= 32761 \\
AD &= \sqrt{32761} = 181m
\end{align*}
Donc le parcours ACDA mesure
\begin{align*}
AD + AC + CD = 181 + 180 + 19 = 380m
\end{align*}
\item Parcours AEFA:
D'après les données, on sait que $(EF) // (E'F')$. On voit aussi que $A$, $E'$ et $E$ sont alignés. Il en est de même pour les points $A$, $F'$ et $F$. Donc d'après le théorème de Thalès
\begin{tabular}{|c|c|c|c|}
\hline
Triangle AEF & AE = 93.9 & AF = 88.2 & EF \\
\hline
Triangle AE'F' & AE' = 31.3 & AF' & E'F' = 64.03 \\
\hline
\end{tabular}
est un tableau de proportionnalité. Donc on peut faire un produit en croix pour calcul $EF$.
\begin{align*}
EF = \frac{E'F' \times AE}{AE'} = \frac{64.03 \times 93.9}{31.3} = 192.1 m
\end{align*}
Donc le parcours AEFA mesure
\begin{align*}
AF + AE + EF = 88.2 + 93.9 + 192.1 = 374.2m
\end{align*}
\item Choix du parcours:
Il faudra choisir le tour $AFEA$ car sa longueur est plus proche de 377m.
\end{itemize}
\end{solution}
\end{exercise}
\begin{exercise}[subtitle={Tirages au sort}]
\renewcommand{\arraystretch}{1.5}
Dans une urne, on a placé des boules colorées indiscernables au touché. Il y a 7 boules bleu, 9 boules jaunes, 5 boules vertes et 10 boules rouges.
\begin{enumerate}
\item % Proba
\begin{enumerate}
\item Quelle est la probabilité de tirer une boule bleu?
\begin{solution}
$\dfrac{7}{31} \approx 0.23$
\end{solution}
\item Quelle est la probabilté de tirer une boule jaune ou bleu?
\begin{solution}
$\dfrac{16}{31} \approx 0.52$
\end{solution}
\item A-t-on plus de chance de tirer une boule verte ou une boule rouge?
\begin{solution}
Boules vertes: $\dfrac{5}{31} \approx 0.16$
Boules rouges: $\dfrac{10}{31} \approx 0.32$
Une boule rouge
\end{solution}
\end{enumerate}
\item % Stat
On effectue 14 tirages (avec remise) dans cette urne et on obtient les couleurs suivantes:
\begin{center}
J \hspace{0.4cm}B \hspace{0.4cm}B \hspace{0.4cm}R \hspace{0.4cm}R \hspace{0.4cm}V \hspace{0.4cm}R \hspace{0.4cm}R \hspace{0.4cm}J \hspace{0.4cm}R \hspace{0.4cm}R \hspace{0.4cm}R \hspace{0.4cm}R \hspace{0.4cm}B \\
\end{center}
\begin{enumerate}
\item Compléter le tableau des effectifs ci-dessous
\begin{tabular}{|c|*{4}{c|}}
\hline
Couleur & Bleu & Jaune & Vert & Rouge \\
\hline
Effectif & & & & \\
\hline
\end{tabular}
\begin{solution}
\begin{tabular}{|c|*{4}{c|}}
\hline
Couleur & Bleu & Jaune & Vert & Rouge \\
\hline
Effectif & 3 & 2 & 1 & 8 \\
\hline
\end{tabular}
\end{solution}
\item Calculer la fréquence des boules vertes.
\begin{solution}
Fréquence de boules vertes: $\frac{1}{14}$
\end{solution}
\end{enumerate}
\item À chaque couleur, on associe des points. Une boule bleu rapporte 10 points, une boule jaune 5 points, une boule verte 2 points et une boule rouge 0 points.
\begin{enumerate}
\item Combien de points a-t-on gagné au total?
\begin{solution}
17
\end{solution}
\item Calculer la moyenne des gains.
\begin{solution}
4.25
\end{solution}
\item Calculer la médiane des gains.
\begin{solution}
3.5
\end{solution}
\end{enumerate}
\end{enumerate}
\end{exercise}
\end{document}
%%% Local Variables:
%%% mode: latex
%%% TeX-master: "master"
%%% End:

View File

@ -0,0 +1,278 @@
\documentclass[a4paper,12pt]{article}
\usepackage{myXsim}
\usepackage{tkz-fct}
\usepackage{wrapfig}
\title{DM de Paques}
\tribe{302}
\date{Jeudi 3 mai 2018}
\sujet{15}
%\geometry{left=10mm,right=10mm, bottom= 10mm, top=10mm}
\xsimsetup{
solution/print = false
}
\begin{document}
\maketitle
\bigskip
{\Large \textbf{Nom - Prénom:}}
\begin{exercise}[subtitle={Battle of the year}]
% theme: Fonction linéaire, Fonction affine
% require: tkz-fct
Taraina veut inscrire ses 21 élèves à un entrainement pour l'évènement \textbf{Battle of the year}.
Deux tarifs lui sont proposés:
\begin{itemize}
\item Tarif Individuel: 151 \euro par danseur inscrit.
\item Tarif Groupe : Paiement d'un forfait de 493 \euro pour le groupe puis 118 \euro par danseur inscrit.
\end{itemize}
\medskip
\begin{enumerate}
\item Complète le tableau suivant :
\medskip
\begin{tabularx}{0.7\linewidth}{|c|*{3}{>{\centering \arraybackslash}X|}}\hline
Nombre d'inscriptions & 0 & 10 & 25\\
\hline
Prix au tarif Individuel en \euro & & & \\
\hline
Prix au tarif Groupe en \euro & & & \\
\hline
\end{tabularx}
\begin{solution}
\begin{tabularx}{0.7\linewidth}{|c|*{3}{>{\centering \arraybackslash}X|}}\hline
Nombre d'inscriptions & 0 & 10 & 25\\
\hline
Prix au tarif Individuel en \euro & 0 & 1510 & 3775 \\
\hline
Prix au tarif Groupe en \euro & 493 & 1673 & 3443\\
\hline
\end{tabularx}
\end{solution}
\medskip
\item Pour chacun des tarifs, exprimer le prix en fonction du nombre de danseurs inscrits.
\begin{solution}
$x$ représente ici le nombre d'élèves inscrits.
\begin{itemize}
\item Tarif Individuel: $f: x \mapsto 151x$
\item Tarif Groupe: $g: x \mapsto 118x + 493$
\end{itemize}
\end{solution}
\item Tracer sur le graphique suivant, les courbes représentants les 2 tarifs proposés.
% On force que le graphique soit légèrement plus grand que 12 cm
\begin{tikzpicture}[yscale=0.6]
\tkzInit[xmin=0,xmax=26,
ymin=0,ymax=3926,
xstep=2,ystep=200]
\tkzAxeX[thick, poslabel=right,label=]
\tkzAxeY[thick, poslabel=above,label=]
\tkzDrawX[label={\textit{Danseurs inscrits}},below= -12pt]
\tkzDrawY[label={\textit{Prix}}, below=-10pt]
\tkzGrid
\tkzFct[domain=0:26, color=blue, very thick]{151*\x}
\tkzFct[domain=0:26, color=red, very thick]{118*\x+493}
\end{tikzpicture}
\item Pour quel nombre d'inscriptions paye-t-on le même prix quel que soit le tarif choisi?
\begin{solution}
$14.93939393939394$ % est entre 10 et 20 par contruction des paramètres
\end{solution}
\end{enumerate}
\end{exercise}
\begin{exercise}[subtitle={Parcours}]
% exo de geometrie comme au brevet blanc.
Une commune souhaite aménager des parcours de santé sur son territoire. On fait deux propositions au conseil municipale, schématisés ci-dessous:
\begin{itemize}
\item Le parcours ACDA
\item Le parcours AEFA
\end{itemize}
Ils souhaitent faire un parcours dont la longueur s'approche le plus possible de 10km.
Peux-tu les aider à choisir le parcours? Justifie
\textbf{Attention: La figure proposée au conseil municipale n'est pas à l'échelle, mais les codages et les dimension données sont correctes.}
\begin{minipage}{0.6\textwidth}
\includegraphics[scale = 0.4]{./fig/parcours}
\end{minipage}
\begin{minipage}{0.4\textwidth}
\begin{itemize}
\item $AC = 4km$
\item $CD = 3km$
\item $AE' = 1.9km$
\item $AE = 3.8km$
\item $AF = 2.2km$
\item $E'F' = -0.3km$
\item $(E'F') // (EF)$
\item L'angle $\widehat{EAF}$ vaut $30^o$
\end{itemize}
\end{minipage}
\begin{solution}
\begin{itemize}
\item Parcours ACDA:
D'après la figure, on voit que le triangle $ACD$ est rectangle en $C$ donc d'après le théorème de Pythagore, on a
\begin{align*}
AD^2 &= AC^2 + DC^2 \\
AD^2 &= 4^2 + 3^2 \\
AD^2 &= 16 + 9 \\
AD^2 &= 25 \\
AD &= \sqrt{25} = 5km
\end{align*}
Donc le parcours ACDA mesure
\begin{align*}
AD + AC + CD = 5 + 4 + 3 = 12km
\end{align*}
\item Parcours AEFA:
D'après les données, on sait que $(EF) // (E'F')$. On voit aussi que $A$, $E'$ et $E$ sont alignés. Il en est de même pour les points $A$, $F'$ et $F$. Donc d'après le théorème de Thalès
\begin{tabular}{|c|c|c|c|}
\hline
Triangle AEF & AE = 3.8 & AF = 2.2 & EF \\
\hline
Triangle AE'F' & AE' = 1.9 & AF' & E'F' = -0.3 \\
\hline
\end{tabular}
est un tableau de proportionnalité. Donc on peut faire un produit en croix pour calcul $EF$.
\begin{align*}
EF = \frac{E'F' \times AE}{AE'} = \frac{-0.3 \times 3.8}{1.9} = -0.6 km
\end{align*}
Donc le parcours AEFA mesure
\begin{align*}
AF + AE + EF = 2.2 + 3.8 + -0.6 = 5.4km
\end{align*}
\item Choix du parcours:
Il faudra choisir le tour $ACDA$ car sa longueur est plus proche de 10km.
\end{itemize}
\end{solution}
\end{exercise}
\begin{exercise}[subtitle={Tirages au sort}]
\renewcommand{\arraystretch}{1.5}
Dans une urne, on a placé des boules colorées indiscernables au touché. Il y a 7 boules bleu, 7 boules jaunes, 8 boules vertes et 6 boules rouges.
\begin{enumerate}
\item % Proba
\begin{enumerate}
\item Quelle est la probabilité de tirer une boule bleu?
\begin{solution}
$\dfrac{7}{28} \approx 0.25$
\end{solution}
\item Quelle est la probabilté de tirer une boule jaune ou bleu?
\begin{solution}
$\dfrac{14}{28} \approx 0.5$
\end{solution}
\item A-t-on plus de chance de tirer une boule verte ou une boule rouge?
\begin{solution}
Boules vertes: $\dfrac{8}{28} \approx 0.29$
Boules rouges: $\dfrac{6}{28} \approx 0.21$
Une boule verte
\end{solution}
\end{enumerate}
\item % Stat
On effectue 14 tirages (avec remise) dans cette urne et on obtient les couleurs suivantes:
\begin{center}
R \hspace{0.4cm}V \hspace{0.4cm}J \hspace{0.4cm}J \hspace{0.4cm}B \hspace{0.4cm}B \hspace{0.4cm}B \hspace{0.4cm}R \hspace{0.4cm}R \hspace{0.4cm}V \hspace{0.4cm}V \hspace{0.4cm}B \hspace{0.4cm}J \hspace{0.4cm}J \\
\end{center}
\begin{enumerate}
\item Compléter le tableau des effectifs ci-dessous
\begin{tabular}{|c|*{4}{c|}}
\hline
Couleur & Bleu & Jaune & Vert & Rouge \\
\hline
Effectif & & & & \\
\hline
\end{tabular}
\begin{solution}
\begin{tabular}{|c|*{4}{c|}}
\hline
Couleur & Bleu & Jaune & Vert & Rouge \\
\hline
Effectif & 4 & 4 & 3 & 3 \\
\hline
\end{tabular}
\end{solution}
\item Calculer la fréquence des boules vertes.
\begin{solution}
Fréquence de boules vertes: $\frac{3}{14}$
\end{solution}
\end{enumerate}
\item À chaque couleur, on associe des points. Une boule bleu rapporte 10 points, une boule jaune 5 points, une boule verte 2 points et une boule rouge 0 points.
\begin{enumerate}
\item Combien de points a-t-on gagné au total?
\begin{solution}
17
\end{solution}
\item Calculer la moyenne des gains.
\begin{solution}
4.25
\end{solution}
\item Calculer la médiane des gains.
\begin{solution}
3.5
\end{solution}
\end{enumerate}
\end{enumerate}
\end{exercise}
\end{document}
%%% Local Variables:
%%% mode: latex
%%% TeX-master: "master"
%%% End:

View File

@ -0,0 +1,278 @@
\documentclass[a4paper,12pt]{article}
\usepackage{myXsim}
\usepackage{tkz-fct}
\usepackage{wrapfig}
\title{DM de Paques}
\tribe{302}
\date{Jeudi 3 mai 2018}
\sujet{16}
%\geometry{left=10mm,right=10mm, bottom= 10mm, top=10mm}
\xsimsetup{
solution/print = false
}
\begin{document}
\maketitle
\bigskip
{\Large \textbf{Nom - Prénom:}}
\begin{exercise}[subtitle={Battle of the year}]
% theme: Fonction linéaire, Fonction affine
% require: tkz-fct
Taraina veut inscrire ses 21 élèves à un entrainement pour l'évènement \textbf{Battle of the year}.
Deux tarifs lui sont proposés:
\begin{itemize}
\item Tarif Individuel: 136 \euro par danseur inscrit.
\item Tarif Groupe : Paiement d'un forfait de 496 \euro pour le groupe puis 97 \euro par danseur inscrit.
\end{itemize}
\medskip
\begin{enumerate}
\item Complète le tableau suivant :
\medskip
\begin{tabularx}{0.7\linewidth}{|c|*{3}{>{\centering \arraybackslash}X|}}\hline
Nombre d'inscriptions & 0 & 10 & 25\\
\hline
Prix au tarif Individuel en \euro & & & \\
\hline
Prix au tarif Groupe en \euro & & & \\
\hline
\end{tabularx}
\begin{solution}
\begin{tabularx}{0.7\linewidth}{|c|*{3}{>{\centering \arraybackslash}X|}}\hline
Nombre d'inscriptions & 0 & 10 & 25\\
\hline
Prix au tarif Individuel en \euro & 0 & 1360 & 3400 \\
\hline
Prix au tarif Groupe en \euro & 496 & 1466 & 2921\\
\hline
\end{tabularx}
\end{solution}
\medskip
\item Pour chacun des tarifs, exprimer le prix en fonction du nombre de danseurs inscrits.
\begin{solution}
$x$ représente ici le nombre d'élèves inscrits.
\begin{itemize}
\item Tarif Individuel: $f: x \mapsto 136x$
\item Tarif Groupe: $g: x \mapsto 97x + 496$
\end{itemize}
\end{solution}
\item Tracer sur le graphique suivant, les courbes représentants les 2 tarifs proposés.
% On force que le graphique soit légèrement plus grand que 12 cm
\begin{tikzpicture}[yscale=0.7]
\tkzInit[xmin=0,xmax=26,
ymin=0,ymax=3536,
xstep=2,ystep=200]
\tkzAxeX[thick, poslabel=right,label=]
\tkzAxeY[thick, poslabel=above,label=]
\tkzDrawX[label={\textit{Danseurs inscrits}},below= -12pt]
\tkzDrawY[label={\textit{Prix}}, below=-10pt]
\tkzGrid
\tkzFct[domain=0:26, color=blue, very thick]{136*\x}
\tkzFct[domain=0:26, color=red, very thick]{97*\x+496}
\end{tikzpicture}
\item Pour quel nombre d'inscriptions paye-t-on le même prix quel que soit le tarif choisi?
\begin{solution}
$12.717948717948717$ % est entre 10 et 20 par contruction des paramètres
\end{solution}
\end{enumerate}
\end{exercise}
\begin{exercise}[subtitle={Parcours}]
% exo de geometrie comme au brevet blanc.
Une commune souhaite aménager des parcours de santé sur son territoire. On fait deux propositions au conseil municipale, schématisés ci-dessous:
\begin{itemize}
\item Le parcours ACDA
\item Le parcours AEFA
\end{itemize}
Ils souhaitent faire un parcours dont la longueur s'approche le plus possible de 287m.
Peux-tu les aider à choisir le parcours? Justifie
\textbf{Attention: La figure proposée au conseil municipale n'est pas à l'échelle, mais les codages et les dimension données sont correctes.}
\begin{minipage}{0.6\textwidth}
\includegraphics[scale = 0.4]{./fig/parcours}
\end{minipage}
\begin{minipage}{0.4\textwidth}
\begin{itemize}
\item $AC = 126m$
\item $CD = 32m$
\item $AE' = 13.43m$
\item $AE = 53.7m$
\item $AF = 1.3m$
\item $E'F' = 56.95m$
\item $(E'F') // (EF)$
\item L'angle $\widehat{EAF}$ vaut $30^o$
\end{itemize}
\end{minipage}
\begin{solution}
\begin{itemize}
\item Parcours ACDA:
D'après la figure, on voit que le triangle $ACD$ est rectangle en $C$ donc d'après le théorème de Pythagore, on a
\begin{align*}
AD^2 &= AC^2 + DC^2 \\
AD^2 &= 126^2 + 32^2 \\
AD^2 &= 15876 + 1024 \\
AD^2 &= 16900 \\
AD &= \sqrt{16900} = 130m
\end{align*}
Donc le parcours ACDA mesure
\begin{align*}
AD + AC + CD = 130 + 126 + 32 = 288m
\end{align*}
\item Parcours AEFA:
D'après les données, on sait que $(EF) // (E'F')$. On voit aussi que $A$, $E'$ et $E$ sont alignés. Il en est de même pour les points $A$, $F'$ et $F$. Donc d'après le théorème de Thalès
\begin{tabular}{|c|c|c|c|}
\hline
Triangle AEF & AE = 53.7 & AF = 1.3 & EF \\
\hline
Triangle AE'F' & AE' = 13.43 & AF' & E'F' = 56.95 \\
\hline
\end{tabular}
est un tableau de proportionnalité. Donc on peut faire un produit en croix pour calcul $EF$.
\begin{align*}
EF = \frac{E'F' \times AE}{AE'} = \frac{56.95 \times 53.7}{13.43} = 227.8 m
\end{align*}
Donc le parcours AEFA mesure
\begin{align*}
AF + AE + EF = 1.3 + 53.7 + 227.8 = 282.8m
\end{align*}
\item Choix du parcours:
Il faudra choisir le tour $ACDA$ car sa longueur est plus proche de 287m.
\end{itemize}
\end{solution}
\end{exercise}
\begin{exercise}[subtitle={Tirages au sort}]
\renewcommand{\arraystretch}{1.5}
Dans une urne, on a placé des boules colorées indiscernables au touché. Il y a 3 boules bleu, 9 boules jaunes, 6 boules vertes et 10 boules rouges.
\begin{enumerate}
\item % Proba
\begin{enumerate}
\item Quelle est la probabilité de tirer une boule bleu?
\begin{solution}
$\dfrac{3}{28} \approx 0.11$
\end{solution}
\item Quelle est la probabilté de tirer une boule jaune ou bleu?
\begin{solution}
$\dfrac{12}{28} \approx 0.43$
\end{solution}
\item A-t-on plus de chance de tirer une boule verte ou une boule rouge?
\begin{solution}
Boules vertes: $\dfrac{6}{28} \approx 0.21$
Boules rouges: $\dfrac{10}{28} \approx 0.36$
Une boule rouge
\end{solution}
\end{enumerate}
\item % Stat
On effectue 14 tirages (avec remise) dans cette urne et on obtient les couleurs suivantes:
\begin{center}
R \hspace{0.4cm}J \hspace{0.4cm}J \hspace{0.4cm}J \hspace{0.4cm}R \hspace{0.4cm}R \hspace{0.4cm}R \hspace{0.4cm}J \hspace{0.4cm}R \hspace{0.4cm}J \hspace{0.4cm}J \hspace{0.4cm}R \hspace{0.4cm}J \hspace{0.4cm}V \\
\end{center}
\begin{enumerate}
\item Compléter le tableau des effectifs ci-dessous
\begin{tabular}{|c|*{4}{c|}}
\hline
Couleur & Bleu & Jaune & Vert & Rouge \\
\hline
Effectif & & & & \\
\hline
\end{tabular}
\begin{solution}
\begin{tabular}{|c|*{4}{c|}}
\hline
Couleur & Bleu & Jaune & Vert & Rouge \\
\hline
Effectif & & 7 & 1 & 6 \\
\hline
\end{tabular}
\end{solution}
\item Calculer la fréquence des boules vertes.
\begin{solution}
Fréquence de boules vertes: $\frac{1}{14}$
\end{solution}
\end{enumerate}
\item À chaque couleur, on associe des points. Une boule bleu rapporte 10 points, une boule jaune 5 points, une boule verte 2 points et une boule rouge 0 points.
\begin{enumerate}
\item Combien de points a-t-on gagné au total?
\begin{solution}
17
\end{solution}
\item Calculer la moyenne des gains.
\begin{solution}
4.25
\end{solution}
\item Calculer la médiane des gains.
\begin{solution}
3.5
\end{solution}
\end{enumerate}
\end{enumerate}
\end{exercise}
\end{document}
%%% Local Variables:
%%% mode: latex
%%% TeX-master: "master"
%%% End:

View File

@ -0,0 +1,278 @@
\documentclass[a4paper,12pt]{article}
\usepackage{myXsim}
\usepackage{tkz-fct}
\usepackage{wrapfig}
\title{DM de Paques}
\tribe{302}
\date{Jeudi 3 mai 2018}
\sujet{17}
%\geometry{left=10mm,right=10mm, bottom= 10mm, top=10mm}
\xsimsetup{
solution/print = false
}
\begin{document}
\maketitle
\bigskip
{\Large \textbf{Nom - Prénom:}}
\begin{exercise}[subtitle={Battle of the year}]
% theme: Fonction linéaire, Fonction affine
% require: tkz-fct
Taraina veut inscrire ses 21 élèves à un entrainement pour l'évènement \textbf{Battle of the year}.
Deux tarifs lui sont proposés:
\begin{itemize}
\item Tarif Individuel: 163 \euro par danseur inscrit.
\item Tarif Groupe : Paiement d'un forfait de 521 \euro pour le groupe puis 121 \euro par danseur inscrit.
\end{itemize}
\medskip
\begin{enumerate}
\item Complète le tableau suivant :
\medskip
\begin{tabularx}{0.7\linewidth}{|c|*{3}{>{\centering \arraybackslash}X|}}\hline
Nombre d'inscriptions & 0 & 10 & 25\\
\hline
Prix au tarif Individuel en \euro & & & \\
\hline
Prix au tarif Groupe en \euro & & & \\
\hline
\end{tabularx}
\begin{solution}
\begin{tabularx}{0.7\linewidth}{|c|*{3}{>{\centering \arraybackslash}X|}}\hline
Nombre d'inscriptions & 0 & 10 & 25\\
\hline
Prix au tarif Individuel en \euro & 0 & 1630 & 4075 \\
\hline
Prix au tarif Groupe en \euro & 521 & 1731 & 3546\\
\hline
\end{tabularx}
\end{solution}
\medskip
\item Pour chacun des tarifs, exprimer le prix en fonction du nombre de danseurs inscrits.
\begin{solution}
$x$ représente ici le nombre d'élèves inscrits.
\begin{itemize}
\item Tarif Individuel: $f: x \mapsto 163x$
\item Tarif Groupe: $g: x \mapsto 121x + 521$
\end{itemize}
\end{solution}
\item Tracer sur le graphique suivant, les courbes représentants les 2 tarifs proposés.
% On force que le graphique soit légèrement plus grand que 12 cm
\begin{tikzpicture}[yscale=0.6]
\tkzInit[xmin=0,xmax=26,
ymin=0,ymax=4238,
xstep=2,ystep=200]
\tkzAxeX[thick, poslabel=right,label=]
\tkzAxeY[thick, poslabel=above,label=]
\tkzDrawX[label={\textit{Danseurs inscrits}},below= -12pt]
\tkzDrawY[label={\textit{Prix}}, below=-10pt]
\tkzGrid
\tkzFct[domain=0:26, color=blue, very thick]{163*\x}
\tkzFct[domain=0:26, color=red, very thick]{121*\x+521}
\end{tikzpicture}
\item Pour quel nombre d'inscriptions paye-t-on le même prix quel que soit le tarif choisi?
\begin{solution}
$12.404761904761905$ % est entre 10 et 20 par contruction des paramètres
\end{solution}
\end{enumerate}
\end{exercise}
\begin{exercise}[subtitle={Parcours}]
% exo de geometrie comme au brevet blanc.
Une commune souhaite aménager des parcours de santé sur son territoire. On fait deux propositions au conseil municipale, schématisés ci-dessous:
\begin{itemize}
\item Le parcours ACDA
\item Le parcours AEFA
\end{itemize}
Ils souhaitent faire un parcours dont la longueur s'approche le plus possible de 117m.
Peux-tu les aider à choisir le parcours? Justifie
\textbf{Attention: La figure proposée au conseil municipale n'est pas à l'échelle, mais les codages et les dimension données sont correctes.}
\begin{minipage}{0.6\textwidth}
\includegraphics[scale = 0.4]{./fig/parcours}
\end{minipage}
\begin{minipage}{0.4\textwidth}
\begin{itemize}
\item $AC = 48m$
\item $CD = 20m$
\item $AE' = 3.88m$
\item $AE = 15.5m$
\item $AF = 37.3m$
\item $E'F' = 14.8m$
\item $(E'F') // (EF)$
\item L'angle $\widehat{EAF}$ vaut $30^o$
\end{itemize}
\end{minipage}
\begin{solution}
\begin{itemize}
\item Parcours ACDA:
D'après la figure, on voit que le triangle $ACD$ est rectangle en $C$ donc d'après le théorème de Pythagore, on a
\begin{align*}
AD^2 &= AC^2 + DC^2 \\
AD^2 &= 48^2 + 20^2 \\
AD^2 &= 2304 + 400 \\
AD^2 &= 2704 \\
AD &= \sqrt{2704} = 52m
\end{align*}
Donc le parcours ACDA mesure
\begin{align*}
AD + AC + CD = 52 + 48 + 20 = 120m
\end{align*}
\item Parcours AEFA:
D'après les données, on sait que $(EF) // (E'F')$. On voit aussi que $A$, $E'$ et $E$ sont alignés. Il en est de même pour les points $A$, $F'$ et $F$. Donc d'après le théorème de Thalès
\begin{tabular}{|c|c|c|c|}
\hline
Triangle AEF & AE = 15.5 & AF = 37.3 & EF \\
\hline
Triangle AE'F' & AE' = 3.88 & AF' & E'F' = 14.8 \\
\hline
\end{tabular}
est un tableau de proportionnalité. Donc on peut faire un produit en croix pour calcul $EF$.
\begin{align*}
EF = \frac{E'F' \times AE}{AE'} = \frac{14.8 \times 15.5}{3.88} = 59.2 m
\end{align*}
Donc le parcours AEFA mesure
\begin{align*}
AF + AE + EF = 37.3 + 15.5 + 59.2 = 112.0m
\end{align*}
\item Choix du parcours:
Il faudra choisir le tour $ACDA$ car sa longueur est plus proche de 117m.
\end{itemize}
\end{solution}
\end{exercise}
\begin{exercise}[subtitle={Tirages au sort}]
\renewcommand{\arraystretch}{1.5}
Dans une urne, on a placé des boules colorées indiscernables au touché. Il y a 10 boules bleu, 7 boules jaunes, 4 boules vertes et 4 boules rouges.
\begin{enumerate}
\item % Proba
\begin{enumerate}
\item Quelle est la probabilité de tirer une boule bleu?
\begin{solution}
$\dfrac{10}{25} \approx 0.4$
\end{solution}
\item Quelle est la probabilté de tirer une boule jaune ou bleu?
\begin{solution}
$\dfrac{17}{25} \approx 0.68$
\end{solution}
\item A-t-on plus de chance de tirer une boule verte ou une boule rouge?
\begin{solution}
Boules vertes: $\dfrac{4}{25} \approx 0.16$
Boules rouges: $\dfrac{4}{25} \approx 0.16$
Une boule rouge
\end{solution}
\end{enumerate}
\item % Stat
On effectue 14 tirages (avec remise) dans cette urne et on obtient les couleurs suivantes:
\begin{center}
V \hspace{0.4cm}B \hspace{0.4cm}J \hspace{0.4cm}V \hspace{0.4cm}B \hspace{0.4cm}V \hspace{0.4cm}B \hspace{0.4cm}B \hspace{0.4cm}B \hspace{0.4cm}J \hspace{0.4cm}V \hspace{0.4cm}J \hspace{0.4cm}V \hspace{0.4cm}J \\
\end{center}
\begin{enumerate}
\item Compléter le tableau des effectifs ci-dessous
\begin{tabular}{|c|*{4}{c|}}
\hline
Couleur & Bleu & Jaune & Vert & Rouge \\
\hline
Effectif & & & & \\
\hline
\end{tabular}
\begin{solution}
\begin{tabular}{|c|*{4}{c|}}
\hline
Couleur & Bleu & Jaune & Vert & Rouge \\
\hline
Effectif & 5 & 4 & 5 & \\
\hline
\end{tabular}
\end{solution}
\item Calculer la fréquence des boules vertes.
\begin{solution}
Fréquence de boules vertes: $\frac{5}{14}$
\end{solution}
\end{enumerate}
\item À chaque couleur, on associe des points. Une boule bleu rapporte 10 points, une boule jaune 5 points, une boule verte 2 points et une boule rouge 0 points.
\begin{enumerate}
\item Combien de points a-t-on gagné au total?
\begin{solution}
17
\end{solution}
\item Calculer la moyenne des gains.
\begin{solution}
4.25
\end{solution}
\item Calculer la médiane des gains.
\begin{solution}
3.5
\end{solution}
\end{enumerate}
\end{enumerate}
\end{exercise}
\end{document}
%%% Local Variables:
%%% mode: latex
%%% TeX-master: "master"
%%% End:

View File

@ -0,0 +1,278 @@
\documentclass[a4paper,12pt]{article}
\usepackage{myXsim}
\usepackage{tkz-fct}
\usepackage{wrapfig}
\title{DM de Paques}
\tribe{302}
\date{Jeudi 3 mai 2018}
\sujet{18}
%\geometry{left=10mm,right=10mm, bottom= 10mm, top=10mm}
\xsimsetup{
solution/print = false
}
\begin{document}
\maketitle
\bigskip
{\Large \textbf{Nom - Prénom:}}
\begin{exercise}[subtitle={Battle of the year}]
% theme: Fonction linéaire, Fonction affine
% require: tkz-fct
Taraina veut inscrire ses 21 élèves à un entrainement pour l'évènement \textbf{Battle of the year}.
Deux tarifs lui sont proposés:
\begin{itemize}
\item Tarif Individuel: 125 \euro par danseur inscrit.
\item Tarif Groupe : Paiement d'un forfait de 377 \euro pour le groupe puis 92 \euro par danseur inscrit.
\end{itemize}
\medskip
\begin{enumerate}
\item Complète le tableau suivant :
\medskip
\begin{tabularx}{0.7\linewidth}{|c|*{3}{>{\centering \arraybackslash}X|}}\hline
Nombre d'inscriptions & 0 & 10 & 25\\
\hline
Prix au tarif Individuel en \euro & & & \\
\hline
Prix au tarif Groupe en \euro & & & \\
\hline
\end{tabularx}
\begin{solution}
\begin{tabularx}{0.7\linewidth}{|c|*{3}{>{\centering \arraybackslash}X|}}\hline
Nombre d'inscriptions & 0 & 10 & 25\\
\hline
Prix au tarif Individuel en \euro & 0 & 1250 & 3125 \\
\hline
Prix au tarif Groupe en \euro & 377 & 1297 & 2677\\
\hline
\end{tabularx}
\end{solution}
\medskip
\item Pour chacun des tarifs, exprimer le prix en fonction du nombre de danseurs inscrits.
\begin{solution}
$x$ représente ici le nombre d'élèves inscrits.
\begin{itemize}
\item Tarif Individuel: $f: x \mapsto 125x$
\item Tarif Groupe: $g: x \mapsto 92x + 377$
\end{itemize}
\end{solution}
\item Tracer sur le graphique suivant, les courbes représentants les 2 tarifs proposés.
% On force que le graphique soit légèrement plus grand que 12 cm
\begin{tikzpicture}[yscale=0.7]
\tkzInit[xmin=0,xmax=26,
ymin=0,ymax=3250,
xstep=2,ystep=200]
\tkzAxeX[thick, poslabel=right,label=]
\tkzAxeY[thick, poslabel=above,label=]
\tkzDrawX[label={\textit{Danseurs inscrits}},below= -12pt]
\tkzDrawY[label={\textit{Prix}}, below=-10pt]
\tkzGrid
\tkzFct[domain=0:26, color=blue, very thick]{125*\x}
\tkzFct[domain=0:26, color=red, very thick]{92*\x+377}
\end{tikzpicture}
\item Pour quel nombre d'inscriptions paye-t-on le même prix quel que soit le tarif choisi?
\begin{solution}
$11.424242424242424$ % est entre 10 et 20 par contruction des paramètres
\end{solution}
\end{enumerate}
\end{exercise}
\begin{exercise}[subtitle={Parcours}]
% exo de geometrie comme au brevet blanc.
Une commune souhaite aménager des parcours de santé sur son territoire. On fait deux propositions au conseil municipale, schématisés ci-dessous:
\begin{itemize}
\item Le parcours ACDA
\item Le parcours AEFA
\end{itemize}
Ils souhaitent faire un parcours dont la longueur s'approche le plus possible de 260m.
Peux-tu les aider à choisir le parcours? Justifie
\textbf{Attention: La figure proposée au conseil municipale n'est pas à l'échelle, mais les codages et les dimension données sont correctes.}
\begin{minipage}{0.6\textwidth}
\includegraphics[scale = 0.4]{./fig/parcours}
\end{minipage}
\begin{minipage}{0.4\textwidth}
\begin{itemize}
\item $AC = 60m$
\item $CD = 91m$
\item $AE' = 53.45m$
\item $AE = 106.9m$
\item $AF = 23.3m$
\item $E'F' = 62.6m$
\item $(E'F') // (EF)$
\item L'angle $\widehat{EAF}$ vaut $30^o$
\end{itemize}
\end{minipage}
\begin{solution}
\begin{itemize}
\item Parcours ACDA:
D'après la figure, on voit que le triangle $ACD$ est rectangle en $C$ donc d'après le théorème de Pythagore, on a
\begin{align*}
AD^2 &= AC^2 + DC^2 \\
AD^2 &= 60^2 + 91^2 \\
AD^2 &= 3600 + 8281 \\
AD^2 &= 11881 \\
AD &= \sqrt{11881} = 109m
\end{align*}
Donc le parcours ACDA mesure
\begin{align*}
AD + AC + CD = 109 + 60 + 91 = 260m
\end{align*}
\item Parcours AEFA:
D'après les données, on sait que $(EF) // (E'F')$. On voit aussi que $A$, $E'$ et $E$ sont alignés. Il en est de même pour les points $A$, $F'$ et $F$. Donc d'après le théorème de Thalès
\begin{tabular}{|c|c|c|c|}
\hline
Triangle AEF & AE = 106.9 & AF = 23.3 & EF \\
\hline
Triangle AE'F' & AE' = 53.45 & AF' & E'F' = 62.6 \\
\hline
\end{tabular}
est un tableau de proportionnalité. Donc on peut faire un produit en croix pour calcul $EF$.
\begin{align*}
EF = \frac{E'F' \times AE}{AE'} = \frac{62.6 \times 106.9}{53.45} = 125.2 m
\end{align*}
Donc le parcours AEFA mesure
\begin{align*}
AF + AE + EF = 23.3 + 106.9 + 125.2 = 255.4m
\end{align*}
\item Choix du parcours:
Il faudra choisir le tour $ACDA$ car sa longueur est plus proche de 260m.
\end{itemize}
\end{solution}
\end{exercise}
\begin{exercise}[subtitle={Tirages au sort}]
\renewcommand{\arraystretch}{1.5}
Dans une urne, on a placé des boules colorées indiscernables au touché. Il y a 2 boules bleu, 7 boules jaunes, 6 boules vertes et 10 boules rouges.
\begin{enumerate}
\item % Proba
\begin{enumerate}
\item Quelle est la probabilité de tirer une boule bleu?
\begin{solution}
$\dfrac{2}{25} \approx 0.08$
\end{solution}
\item Quelle est la probabilté de tirer une boule jaune ou bleu?
\begin{solution}
$\dfrac{9}{25} \approx 0.36$
\end{solution}
\item A-t-on plus de chance de tirer une boule verte ou une boule rouge?
\begin{solution}
Boules vertes: $\dfrac{6}{25} \approx 0.24$
Boules rouges: $\dfrac{10}{25} \approx 0.4$
Une boule rouge
\end{solution}
\end{enumerate}
\item % Stat
On effectue 14 tirages (avec remise) dans cette urne et on obtient les couleurs suivantes:
\begin{center}
R \hspace{0.4cm}J \hspace{0.4cm}R \hspace{0.4cm}R \hspace{0.4cm}R \hspace{0.4cm}R \hspace{0.4cm}J \hspace{0.4cm}B \hspace{0.4cm}R \hspace{0.4cm}J \hspace{0.4cm}R \hspace{0.4cm}B \hspace{0.4cm}J \hspace{0.4cm}R \\
\end{center}
\begin{enumerate}
\item Compléter le tableau des effectifs ci-dessous
\begin{tabular}{|c|*{4}{c|}}
\hline
Couleur & Bleu & Jaune & Vert & Rouge \\
\hline
Effectif & & & & \\
\hline
\end{tabular}
\begin{solution}
\begin{tabular}{|c|*{4}{c|}}
\hline
Couleur & Bleu & Jaune & Vert & Rouge \\
\hline
Effectif & 2 & 4 & & 8 \\
\hline
\end{tabular}
\end{solution}
\item Calculer la fréquence des boules vertes.
\begin{solution}
Fréquence de boules vertes: $\frac{0}{14}$
\end{solution}
\end{enumerate}
\item À chaque couleur, on associe des points. Une boule bleu rapporte 10 points, une boule jaune 5 points, une boule verte 2 points et une boule rouge 0 points.
\begin{enumerate}
\item Combien de points a-t-on gagné au total?
\begin{solution}
17
\end{solution}
\item Calculer la moyenne des gains.
\begin{solution}
4.25
\end{solution}
\item Calculer la médiane des gains.
\begin{solution}
3.5
\end{solution}
\end{enumerate}
\end{enumerate}
\end{exercise}
\end{document}
%%% Local Variables:
%%% mode: latex
%%% TeX-master: "master"
%%% End:

View File

@ -0,0 +1,278 @@
\documentclass[a4paper,12pt]{article}
\usepackage{myXsim}
\usepackage{tkz-fct}
\usepackage{wrapfig}
\title{DM de Paques}
\tribe{302}
\date{Jeudi 3 mai 2018}
\sujet{19}
%\geometry{left=10mm,right=10mm, bottom= 10mm, top=10mm}
\xsimsetup{
solution/print = false
}
\begin{document}
\maketitle
\bigskip
{\Large \textbf{Nom - Prénom:}}
\begin{exercise}[subtitle={Battle of the year}]
% theme: Fonction linéaire, Fonction affine
% require: tkz-fct
Taraina veut inscrire ses 21 élèves à un entrainement pour l'évènement \textbf{Battle of the year}.
Deux tarifs lui sont proposés:
\begin{itemize}
\item Tarif Individuel: 100 \euro par danseur inscrit.
\item Tarif Groupe : Paiement d'un forfait de 367 \euro pour le groupe puis 79 \euro par danseur inscrit.
\end{itemize}
\medskip
\begin{enumerate}
\item Complète le tableau suivant :
\medskip
\begin{tabularx}{0.7\linewidth}{|c|*{3}{>{\centering \arraybackslash}X|}}\hline
Nombre d'inscriptions & 0 & 10 & 25\\
\hline
Prix au tarif Individuel en \euro & & & \\
\hline
Prix au tarif Groupe en \euro & & & \\
\hline
\end{tabularx}
\begin{solution}
\begin{tabularx}{0.7\linewidth}{|c|*{3}{>{\centering \arraybackslash}X|}}\hline
Nombre d'inscriptions & 0 & 10 & 25\\
\hline
Prix au tarif Individuel en \euro & 0 & 1000 & 2500 \\
\hline
Prix au tarif Groupe en \euro & 367 & 1157 & 2342\\
\hline
\end{tabularx}
\end{solution}
\medskip
\item Pour chacun des tarifs, exprimer le prix en fonction du nombre de danseurs inscrits.
\begin{solution}
$x$ représente ici le nombre d'élèves inscrits.
\begin{itemize}
\item Tarif Individuel: $f: x \mapsto 100x$
\item Tarif Groupe: $g: x \mapsto 79x + 367$
\end{itemize}
\end{solution}
\item Tracer sur le graphique suivant, les courbes représentants les 2 tarifs proposés.
% On force que le graphique soit légèrement plus grand que 12 cm
\begin{tikzpicture}[yscale=0.9]
\tkzInit[xmin=0,xmax=26,
ymin=0,ymax=2600,
xstep=2,ystep=200]
\tkzAxeX[thick, poslabel=right,label=]
\tkzAxeY[thick, poslabel=above,label=]
\tkzDrawX[label={\textit{Danseurs inscrits}},below= -12pt]
\tkzDrawY[label={\textit{Prix}}, below=-10pt]
\tkzGrid
\tkzFct[domain=0:26, color=blue, very thick]{100*\x}
\tkzFct[domain=0:26, color=red, very thick]{79*\x+367}
\end{tikzpicture}
\item Pour quel nombre d'inscriptions paye-t-on le même prix quel que soit le tarif choisi?
\begin{solution}
$17.476190476190474$ % est entre 10 et 20 par contruction des paramètres
\end{solution}
\end{enumerate}
\end{exercise}
\begin{exercise}[subtitle={Parcours}]
% exo de geometrie comme au brevet blanc.
Une commune souhaite aménager des parcours de santé sur son territoire. On fait deux propositions au conseil municipale, schématisés ci-dessous:
\begin{itemize}
\item Le parcours ACDA
\item Le parcours AEFA
\end{itemize}
Ils souhaitent faire un parcours dont la longueur s'approche le plus possible de 84km.
Peux-tu les aider à choisir le parcours? Justifie
\textbf{Attention: La figure proposée au conseil municipale n'est pas à l'échelle, mais les codages et les dimension données sont correctes.}
\begin{minipage}{0.6\textwidth}
\includegraphics[scale = 0.4]{./fig/parcours}
\end{minipage}
\begin{minipage}{0.4\textwidth}
\begin{itemize}
\item $AC = 12km$
\item $CD = 35km$
\item $AE' = 17.15km$
\item $AE = 34.3km$
\item $AF = 37.1km$
\item $E'F' = 3.1km$
\item $(E'F') // (EF)$
\item L'angle $\widehat{EAF}$ vaut $30^o$
\end{itemize}
\end{minipage}
\begin{solution}
\begin{itemize}
\item Parcours ACDA:
D'après la figure, on voit que le triangle $ACD$ est rectangle en $C$ donc d'après le théorème de Pythagore, on a
\begin{align*}
AD^2 &= AC^2 + DC^2 \\
AD^2 &= 12^2 + 35^2 \\
AD^2 &= 144 + 1225 \\
AD^2 &= 1369 \\
AD &= \sqrt{1369} = 37km
\end{align*}
Donc le parcours ACDA mesure
\begin{align*}
AD + AC + CD = 37 + 12 + 35 = 84km
\end{align*}
\item Parcours AEFA:
D'après les données, on sait que $(EF) // (E'F')$. On voit aussi que $A$, $E'$ et $E$ sont alignés. Il en est de même pour les points $A$, $F'$ et $F$. Donc d'après le théorème de Thalès
\begin{tabular}{|c|c|c|c|}
\hline
Triangle AEF & AE = 34.3 & AF = 37.1 & EF \\
\hline
Triangle AE'F' & AE' = 17.15 & AF' & E'F' = 3.1 \\
\hline
\end{tabular}
est un tableau de proportionnalité. Donc on peut faire un produit en croix pour calcul $EF$.
\begin{align*}
EF = \frac{E'F' \times AE}{AE'} = \frac{3.1 \times 34.3}{17.15} = 6.2 km
\end{align*}
Donc le parcours AEFA mesure
\begin{align*}
AF + AE + EF = 37.1 + 34.3 + 6.2 = 77.6km
\end{align*}
\item Choix du parcours:
Il faudra choisir le tour $ACDA$ car sa longueur est plus proche de 84km.
\end{itemize}
\end{solution}
\end{exercise}
\begin{exercise}[subtitle={Tirages au sort}]
\renewcommand{\arraystretch}{1.5}
Dans une urne, on a placé des boules colorées indiscernables au touché. Il y a 2 boules bleu, 10 boules jaunes, 9 boules vertes et 8 boules rouges.
\begin{enumerate}
\item % Proba
\begin{enumerate}
\item Quelle est la probabilité de tirer une boule bleu?
\begin{solution}
$\dfrac{2}{29} \approx 0.07$
\end{solution}
\item Quelle est la probabilté de tirer une boule jaune ou bleu?
\begin{solution}
$\dfrac{12}{29} \approx 0.41$
\end{solution}
\item A-t-on plus de chance de tirer une boule verte ou une boule rouge?
\begin{solution}
Boules vertes: $\dfrac{9}{29} \approx 0.31$
Boules rouges: $\dfrac{8}{29} \approx 0.28$
Une boule verte
\end{solution}
\end{enumerate}
\item % Stat
On effectue 14 tirages (avec remise) dans cette urne et on obtient les couleurs suivantes:
\begin{center}
V \hspace{0.4cm}J \hspace{0.4cm}V \hspace{0.4cm}J \hspace{0.4cm}V \hspace{0.4cm}J \hspace{0.4cm}R \hspace{0.4cm}V \hspace{0.4cm}V \hspace{0.4cm}V \hspace{0.4cm}V \hspace{0.4cm}B \hspace{0.4cm}J \hspace{0.4cm}V \\
\end{center}
\begin{enumerate}
\item Compléter le tableau des effectifs ci-dessous
\begin{tabular}{|c|*{4}{c|}}
\hline
Couleur & Bleu & Jaune & Vert & Rouge \\
\hline
Effectif & & & & \\
\hline
\end{tabular}
\begin{solution}
\begin{tabular}{|c|*{4}{c|}}
\hline
Couleur & Bleu & Jaune & Vert & Rouge \\
\hline
Effectif & 1 & 4 & 8 & 1 \\
\hline
\end{tabular}
\end{solution}
\item Calculer la fréquence des boules vertes.
\begin{solution}
Fréquence de boules vertes: $\frac{8}{14}$
\end{solution}
\end{enumerate}
\item À chaque couleur, on associe des points. Une boule bleu rapporte 10 points, une boule jaune 5 points, une boule verte 2 points et une boule rouge 0 points.
\begin{enumerate}
\item Combien de points a-t-on gagné au total?
\begin{solution}
17
\end{solution}
\item Calculer la moyenne des gains.
\begin{solution}
4.25
\end{solution}
\item Calculer la médiane des gains.
\begin{solution}
3.5
\end{solution}
\end{enumerate}
\end{enumerate}
\end{exercise}
\end{document}
%%% Local Variables:
%%% mode: latex
%%% TeX-master: "master"
%%% End:

View File

@ -0,0 +1,278 @@
\documentclass[a4paper,12pt]{article}
\usepackage{myXsim}
\usepackage{tkz-fct}
\usepackage{wrapfig}
\title{DM de Paques}
\tribe{302}
\date{Jeudi 3 mai 2018}
\sujet{20}
%\geometry{left=10mm,right=10mm, bottom= 10mm, top=10mm}
\xsimsetup{
solution/print = false
}
\begin{document}
\maketitle
\bigskip
{\Large \textbf{Nom - Prénom:}}
\begin{exercise}[subtitle={Battle of the year}]
% theme: Fonction linéaire, Fonction affine
% require: tkz-fct
Taraina veut inscrire ses 21 élèves à un entrainement pour l'évènement \textbf{Battle of the year}.
Deux tarifs lui sont proposés:
\begin{itemize}
\item Tarif Individuel: 135 \euro par danseur inscrit.
\item Tarif Groupe : Paiement d'un forfait de 417 \euro pour le groupe puis 107 \euro par danseur inscrit.
\end{itemize}
\medskip
\begin{enumerate}
\item Complète le tableau suivant :
\medskip
\begin{tabularx}{0.7\linewidth}{|c|*{3}{>{\centering \arraybackslash}X|}}\hline
Nombre d'inscriptions & 0 & 10 & 25\\
\hline
Prix au tarif Individuel en \euro & & & \\
\hline
Prix au tarif Groupe en \euro & & & \\
\hline
\end{tabularx}
\begin{solution}
\begin{tabularx}{0.7\linewidth}{|c|*{3}{>{\centering \arraybackslash}X|}}\hline
Nombre d'inscriptions & 0 & 10 & 25\\
\hline
Prix au tarif Individuel en \euro & 0 & 1350 & 3375 \\
\hline
Prix au tarif Groupe en \euro & 417 & 1487 & 3092\\
\hline
\end{tabularx}
\end{solution}
\medskip
\item Pour chacun des tarifs, exprimer le prix en fonction du nombre de danseurs inscrits.
\begin{solution}
$x$ représente ici le nombre d'élèves inscrits.
\begin{itemize}
\item Tarif Individuel: $f: x \mapsto 135x$
\item Tarif Groupe: $g: x \mapsto 107x + 417$
\end{itemize}
\end{solution}
\item Tracer sur le graphique suivant, les courbes représentants les 2 tarifs proposés.
% On force que le graphique soit légèrement plus grand que 12 cm
\begin{tikzpicture}[yscale=0.7]
\tkzInit[xmin=0,xmax=26,
ymin=0,ymax=3510,
xstep=2,ystep=200]
\tkzAxeX[thick, poslabel=right,label=]
\tkzAxeY[thick, poslabel=above,label=]
\tkzDrawX[label={\textit{Danseurs inscrits}},below= -12pt]
\tkzDrawY[label={\textit{Prix}}, below=-10pt]
\tkzGrid
\tkzFct[domain=0:26, color=blue, very thick]{135*\x}
\tkzFct[domain=0:26, color=red, very thick]{107*\x+417}
\end{tikzpicture}
\item Pour quel nombre d'inscriptions paye-t-on le même prix quel que soit le tarif choisi?
\begin{solution}
$14.892857142857142$ % est entre 10 et 20 par contruction des paramètres
\end{solution}
\end{enumerate}
\end{exercise}
\begin{exercise}[subtitle={Parcours}]
% exo de geometrie comme au brevet blanc.
Une commune souhaite aménager des parcours de santé sur son territoire. On fait deux propositions au conseil municipale, schématisés ci-dessous:
\begin{itemize}
\item Le parcours ACDA
\item Le parcours AEFA
\end{itemize}
Ils souhaitent faire un parcours dont la longueur s'approche le plus possible de 52km.
Peux-tu les aider à choisir le parcours? Justifie
\textbf{Attention: La figure proposée au conseil municipale n'est pas à l'échelle, mais les codages et les dimension données sont correctes.}
\begin{minipage}{0.6\textwidth}
\includegraphics[scale = 0.4]{./fig/parcours}
\end{minipage}
\begin{minipage}{0.4\textwidth}
\begin{itemize}
\item $AC = 24km$
\item $CD = 7km$
\item $AE' = 3.15km$
\item $AE = 6.3km$
\item $AF = 12.3km$
\item $E'F' = 16.2km$
\item $(E'F') // (EF)$
\item L'angle $\widehat{EAF}$ vaut $30^o$
\end{itemize}
\end{minipage}
\begin{solution}
\begin{itemize}
\item Parcours ACDA:
D'après la figure, on voit que le triangle $ACD$ est rectangle en $C$ donc d'après le théorème de Pythagore, on a
\begin{align*}
AD^2 &= AC^2 + DC^2 \\
AD^2 &= 24^2 + 7^2 \\
AD^2 &= 576 + 49 \\
AD^2 &= 625 \\
AD &= \sqrt{625} = 25km
\end{align*}
Donc le parcours ACDA mesure
\begin{align*}
AD + AC + CD = 25 + 24 + 7 = 56km
\end{align*}
\item Parcours AEFA:
D'après les données, on sait que $(EF) // (E'F')$. On voit aussi que $A$, $E'$ et $E$ sont alignés. Il en est de même pour les points $A$, $F'$ et $F$. Donc d'après le théorème de Thalès
\begin{tabular}{|c|c|c|c|}
\hline
Triangle AEF & AE = 6.3 & AF = 12.3 & EF \\
\hline
Triangle AE'F' & AE' = 3.15 & AF' & E'F' = 16.2 \\
\hline
\end{tabular}
est un tableau de proportionnalité. Donc on peut faire un produit en croix pour calcul $EF$.
\begin{align*}
EF = \frac{E'F' \times AE}{AE'} = \frac{16.2 \times 6.3}{3.15} = 32.4 km
\end{align*}
Donc le parcours AEFA mesure
\begin{align*}
AF + AE + EF = 12.3 + 6.3 + 32.4 = 51.0km
\end{align*}
\item Choix du parcours:
Il faudra choisir le tour $AFEA$ car sa longueur est plus proche de 52km.
\end{itemize}
\end{solution}
\end{exercise}
\begin{exercise}[subtitle={Tirages au sort}]
\renewcommand{\arraystretch}{1.5}
Dans une urne, on a placé des boules colorées indiscernables au touché. Il y a 5 boules bleu, 8 boules jaunes, 3 boules vertes et 3 boules rouges.
\begin{enumerate}
\item % Proba
\begin{enumerate}
\item Quelle est la probabilité de tirer une boule bleu?
\begin{solution}
$\dfrac{5}{19} \approx 0.26$
\end{solution}
\item Quelle est la probabilté de tirer une boule jaune ou bleu?
\begin{solution}
$\dfrac{13}{19} \approx 0.68$
\end{solution}
\item A-t-on plus de chance de tirer une boule verte ou une boule rouge?
\begin{solution}
Boules vertes: $\dfrac{3}{19} \approx 0.16$
Boules rouges: $\dfrac{3}{19} \approx 0.16$
Une boule rouge
\end{solution}
\end{enumerate}
\item % Stat
On effectue 14 tirages (avec remise) dans cette urne et on obtient les couleurs suivantes:
\begin{center}
J \hspace{0.4cm}J \hspace{0.4cm}J \hspace{0.4cm}J \hspace{0.4cm}B \hspace{0.4cm}R \hspace{0.4cm}V \hspace{0.4cm}R \hspace{0.4cm}J \hspace{0.4cm}J \hspace{0.4cm}V \hspace{0.4cm}J \hspace{0.4cm}V \hspace{0.4cm}B \\
\end{center}
\begin{enumerate}
\item Compléter le tableau des effectifs ci-dessous
\begin{tabular}{|c|*{4}{c|}}
\hline
Couleur & Bleu & Jaune & Vert & Rouge \\
\hline
Effectif & & & & \\
\hline
\end{tabular}
\begin{solution}
\begin{tabular}{|c|*{4}{c|}}
\hline
Couleur & Bleu & Jaune & Vert & Rouge \\
\hline
Effectif & 2 & 7 & 3 & 2 \\
\hline
\end{tabular}
\end{solution}
\item Calculer la fréquence des boules vertes.
\begin{solution}
Fréquence de boules vertes: $\frac{3}{14}$
\end{solution}
\end{enumerate}
\item À chaque couleur, on associe des points. Une boule bleu rapporte 10 points, une boule jaune 5 points, une boule verte 2 points et une boule rouge 0 points.
\begin{enumerate}
\item Combien de points a-t-on gagné au total?
\begin{solution}
17
\end{solution}
\item Calculer la moyenne des gains.
\begin{solution}
4.25
\end{solution}
\item Calculer la médiane des gains.
\begin{solution}
3.5
\end{solution}
\end{enumerate}
\end{enumerate}
\end{exercise}
\end{document}
%%% Local Variables:
%%% mode: latex
%%% TeX-master: "master"
%%% End:

View File

@ -0,0 +1,278 @@
\documentclass[a4paper,12pt]{article}
\usepackage{myXsim}
\usepackage{tkz-fct}
\usepackage{wrapfig}
\title{DM de Paques}
\tribe{302}
\date{Jeudi 3 mai 2018}
\sujet{21}
%\geometry{left=10mm,right=10mm, bottom= 10mm, top=10mm}
\xsimsetup{
solution/print = false
}
\begin{document}
\maketitle
\bigskip
{\Large \textbf{Nom - Prénom:}}
\begin{exercise}[subtitle={Battle of the year}]
% theme: Fonction linéaire, Fonction affine
% require: tkz-fct
Taraina veut inscrire ses 21 élèves à un entrainement pour l'évènement \textbf{Battle of the year}.
Deux tarifs lui sont proposés:
\begin{itemize}
\item Tarif Individuel: 128 \euro par danseur inscrit.
\item Tarif Groupe : Paiement d'un forfait de 426 \euro pour le groupe puis 98 \euro par danseur inscrit.
\end{itemize}
\medskip
\begin{enumerate}
\item Complète le tableau suivant :
\medskip
\begin{tabularx}{0.7\linewidth}{|c|*{3}{>{\centering \arraybackslash}X|}}\hline
Nombre d'inscriptions & 0 & 10 & 25\\
\hline
Prix au tarif Individuel en \euro & & & \\
\hline
Prix au tarif Groupe en \euro & & & \\
\hline
\end{tabularx}
\begin{solution}
\begin{tabularx}{0.7\linewidth}{|c|*{3}{>{\centering \arraybackslash}X|}}\hline
Nombre d'inscriptions & 0 & 10 & 25\\
\hline
Prix au tarif Individuel en \euro & 0 & 1280 & 3200 \\
\hline
Prix au tarif Groupe en \euro & 426 & 1406 & 2876\\
\hline
\end{tabularx}
\end{solution}
\medskip
\item Pour chacun des tarifs, exprimer le prix en fonction du nombre de danseurs inscrits.
\begin{solution}
$x$ représente ici le nombre d'élèves inscrits.
\begin{itemize}
\item Tarif Individuel: $f: x \mapsto 128x$
\item Tarif Groupe: $g: x \mapsto 98x + 426$
\end{itemize}
\end{solution}
\item Tracer sur le graphique suivant, les courbes représentants les 2 tarifs proposés.
% On force que le graphique soit légèrement plus grand que 12 cm
\begin{tikzpicture}[yscale=0.7]
\tkzInit[xmin=0,xmax=26,
ymin=0,ymax=3328,
xstep=2,ystep=200]
\tkzAxeX[thick, poslabel=right,label=]
\tkzAxeY[thick, poslabel=above,label=]
\tkzDrawX[label={\textit{Danseurs inscrits}},below= -12pt]
\tkzDrawY[label={\textit{Prix}}, below=-10pt]
\tkzGrid
\tkzFct[domain=0:26, color=blue, very thick]{128*\x}
\tkzFct[domain=0:26, color=red, very thick]{98*\x+426}
\end{tikzpicture}
\item Pour quel nombre d'inscriptions paye-t-on le même prix quel que soit le tarif choisi?
\begin{solution}
$14.2$ % est entre 10 et 20 par contruction des paramètres
\end{solution}
\end{enumerate}
\end{exercise}
\begin{exercise}[subtitle={Parcours}]
% exo de geometrie comme au brevet blanc.
Une commune souhaite aménager des parcours de santé sur son territoire. On fait deux propositions au conseil municipale, schématisés ci-dessous:
\begin{itemize}
\item Le parcours ACDA
\item Le parcours AEFA
\end{itemize}
Ils souhaitent faire un parcours dont la longueur s'approche le plus possible de 217m.
Peux-tu les aider à choisir le parcours? Justifie
\textbf{Attention: La figure proposée au conseil municipale n'est pas à l'échelle, mais les codages et les dimension données sont correctes.}
\begin{minipage}{0.6\textwidth}
\includegraphics[scale = 0.4]{./fig/parcours}
\end{minipage}
\begin{minipage}{0.4\textwidth}
\begin{itemize}
\item $AC = 20m$
\item $CD = 99m$
\item $AE' = 26.88m$
\item $AE = 107.5m$
\item $AF = 67.9m$
\item $E'F' = 9.7m$
\item $(E'F') // (EF)$
\item L'angle $\widehat{EAF}$ vaut $30^o$
\end{itemize}
\end{minipage}
\begin{solution}
\begin{itemize}
\item Parcours ACDA:
D'après la figure, on voit que le triangle $ACD$ est rectangle en $C$ donc d'après le théorème de Pythagore, on a
\begin{align*}
AD^2 &= AC^2 + DC^2 \\
AD^2 &= 20^2 + 99^2 \\
AD^2 &= 400 + 9801 \\
AD^2 &= 10201 \\
AD &= \sqrt{10201} = 101m
\end{align*}
Donc le parcours ACDA mesure
\begin{align*}
AD + AC + CD = 101 + 20 + 99 = 220m
\end{align*}
\item Parcours AEFA:
D'après les données, on sait que $(EF) // (E'F')$. On voit aussi que $A$, $E'$ et $E$ sont alignés. Il en est de même pour les points $A$, $F'$ et $F$. Donc d'après le théorème de Thalès
\begin{tabular}{|c|c|c|c|}
\hline
Triangle AEF & AE = 107.5 & AF = 67.9 & EF \\
\hline
Triangle AE'F' & AE' = 26.88 & AF' & E'F' = 9.7 \\
\hline
\end{tabular}
est un tableau de proportionnalité. Donc on peut faire un produit en croix pour calcul $EF$.
\begin{align*}
EF = \frac{E'F' \times AE}{AE'} = \frac{9.7 \times 107.5}{26.88} = 38.8 m
\end{align*}
Donc le parcours AEFA mesure
\begin{align*}
AF + AE + EF = 67.9 + 107.5 + 38.8 = 214.2m
\end{align*}
\item Choix du parcours:
Il faudra choisir le tour $AFEA$ car sa longueur est plus proche de 217m.
\end{itemize}
\end{solution}
\end{exercise}
\begin{exercise}[subtitle={Tirages au sort}]
\renewcommand{\arraystretch}{1.5}
Dans une urne, on a placé des boules colorées indiscernables au touché. Il y a 2 boules bleu, 3 boules jaunes, 6 boules vertes et 7 boules rouges.
\begin{enumerate}
\item % Proba
\begin{enumerate}
\item Quelle est la probabilité de tirer une boule bleu?
\begin{solution}
$\dfrac{2}{18} \approx 0.11$
\end{solution}
\item Quelle est la probabilté de tirer une boule jaune ou bleu?
\begin{solution}
$\dfrac{5}{18} \approx 0.28$
\end{solution}
\item A-t-on plus de chance de tirer une boule verte ou une boule rouge?
\begin{solution}
Boules vertes: $\dfrac{6}{18} \approx 0.33$
Boules rouges: $\dfrac{7}{18} \approx 0.39$
Une boule rouge
\end{solution}
\end{enumerate}
\item % Stat
On effectue 14 tirages (avec remise) dans cette urne et on obtient les couleurs suivantes:
\begin{center}
R \hspace{0.4cm}J \hspace{0.4cm}V \hspace{0.4cm}R \hspace{0.4cm}R \hspace{0.4cm}R \hspace{0.4cm}R \hspace{0.4cm}R \hspace{0.4cm}V \hspace{0.4cm}J \hspace{0.4cm}B \hspace{0.4cm}B \hspace{0.4cm}J \hspace{0.4cm}R \\
\end{center}
\begin{enumerate}
\item Compléter le tableau des effectifs ci-dessous
\begin{tabular}{|c|*{4}{c|}}
\hline
Couleur & Bleu & Jaune & Vert & Rouge \\
\hline
Effectif & & & & \\
\hline
\end{tabular}
\begin{solution}
\begin{tabular}{|c|*{4}{c|}}
\hline
Couleur & Bleu & Jaune & Vert & Rouge \\
\hline
Effectif & 2 & 3 & 2 & 7 \\
\hline
\end{tabular}
\end{solution}
\item Calculer la fréquence des boules vertes.
\begin{solution}
Fréquence de boules vertes: $\frac{2}{14}$
\end{solution}
\end{enumerate}
\item À chaque couleur, on associe des points. Une boule bleu rapporte 10 points, une boule jaune 5 points, une boule verte 2 points et une boule rouge 0 points.
\begin{enumerate}
\item Combien de points a-t-on gagné au total?
\begin{solution}
17
\end{solution}
\item Calculer la moyenne des gains.
\begin{solution}
4.25
\end{solution}
\item Calculer la médiane des gains.
\begin{solution}
3.5
\end{solution}
\end{enumerate}
\end{enumerate}
\end{exercise}
\end{document}
%%% Local Variables:
%%% mode: latex
%%% TeX-master: "master"
%%% End:

View File

@ -0,0 +1,278 @@
\documentclass[a4paper,12pt]{article}
\usepackage{myXsim}
\usepackage{tkz-fct}
\usepackage{wrapfig}
\title{DM de Paques}
\tribe{302}
\date{Jeudi 3 mai 2018}
\sujet{22}
%\geometry{left=10mm,right=10mm, bottom= 10mm, top=10mm}
\xsimsetup{
solution/print = false
}
\begin{document}
\maketitle
\bigskip
{\Large \textbf{Nom - Prénom:}}
\begin{exercise}[subtitle={Battle of the year}]
% theme: Fonction linéaire, Fonction affine
% require: tkz-fct
Taraina veut inscrire ses 21 élèves à un entrainement pour l'évènement \textbf{Battle of the year}.
Deux tarifs lui sont proposés:
\begin{itemize}
\item Tarif Individuel: 149 \euro par danseur inscrit.
\item Tarif Groupe : Paiement d'un forfait de 489 \euro pour le groupe puis 108 \euro par danseur inscrit.
\end{itemize}
\medskip
\begin{enumerate}
\item Complète le tableau suivant :
\medskip
\begin{tabularx}{0.7\linewidth}{|c|*{3}{>{\centering \arraybackslash}X|}}\hline
Nombre d'inscriptions & 0 & 10 & 25\\
\hline
Prix au tarif Individuel en \euro & & & \\
\hline
Prix au tarif Groupe en \euro & & & \\
\hline
\end{tabularx}
\begin{solution}
\begin{tabularx}{0.7\linewidth}{|c|*{3}{>{\centering \arraybackslash}X|}}\hline
Nombre d'inscriptions & 0 & 10 & 25\\
\hline
Prix au tarif Individuel en \euro & 0 & 1490 & 3725 \\
\hline
Prix au tarif Groupe en \euro & 489 & 1569 & 3189\\
\hline
\end{tabularx}
\end{solution}
\medskip
\item Pour chacun des tarifs, exprimer le prix en fonction du nombre de danseurs inscrits.
\begin{solution}
$x$ représente ici le nombre d'élèves inscrits.
\begin{itemize}
\item Tarif Individuel: $f: x \mapsto 149x$
\item Tarif Groupe: $g: x \mapsto 108x + 489$
\end{itemize}
\end{solution}
\item Tracer sur le graphique suivant, les courbes représentants les 2 tarifs proposés.
% On force que le graphique soit légèrement plus grand que 12 cm
\begin{tikzpicture}[yscale=0.6]
\tkzInit[xmin=0,xmax=26,
ymin=0,ymax=3874,
xstep=2,ystep=200]
\tkzAxeX[thick, poslabel=right,label=]
\tkzAxeY[thick, poslabel=above,label=]
\tkzDrawX[label={\textit{Danseurs inscrits}},below= -12pt]
\tkzDrawY[label={\textit{Prix}}, below=-10pt]
\tkzGrid
\tkzFct[domain=0:26, color=blue, very thick]{149*\x}
\tkzFct[domain=0:26, color=red, very thick]{108*\x+489}
\end{tikzpicture}
\item Pour quel nombre d'inscriptions paye-t-on le même prix quel que soit le tarif choisi?
\begin{solution}
$11.926829268292684$ % est entre 10 et 20 par contruction des paramètres
\end{solution}
\end{enumerate}
\end{exercise}
\begin{exercise}[subtitle={Parcours}]
% exo de geometrie comme au brevet blanc.
Une commune souhaite aménager des parcours de santé sur son territoire. On fait deux propositions au conseil municipale, schématisés ci-dessous:
\begin{itemize}
\item Le parcours ACDA
\item Le parcours AEFA
\end{itemize}
Ils souhaitent faire un parcours dont la longueur s'approche le plus possible de 373m.
Peux-tu les aider à choisir le parcours? Justifie
\textbf{Attention: La figure proposée au conseil municipale n'est pas à l'échelle, mais les codages et les dimension données sont correctes.}
\begin{minipage}{0.6\textwidth}
\includegraphics[scale = 0.4]{./fig/parcours}
\end{minipage}
\begin{minipage}{0.4\textwidth}
\begin{itemize}
\item $AC = 180m$
\item $CD = 19m$
\item $AE' = 70.6m$
\item $AE = 141.2m$
\item $AF = 52.9m$
\item $E'F' = 89.85m$
\item $(E'F') // (EF)$
\item L'angle $\widehat{EAF}$ vaut $30^o$
\end{itemize}
\end{minipage}
\begin{solution}
\begin{itemize}
\item Parcours ACDA:
D'après la figure, on voit que le triangle $ACD$ est rectangle en $C$ donc d'après le théorème de Pythagore, on a
\begin{align*}
AD^2 &= AC^2 + DC^2 \\
AD^2 &= 180^2 + 19^2 \\
AD^2 &= 32400 + 361 \\
AD^2 &= 32761 \\
AD &= \sqrt{32761} = 181m
\end{align*}
Donc le parcours ACDA mesure
\begin{align*}
AD + AC + CD = 181 + 180 + 19 = 380m
\end{align*}
\item Parcours AEFA:
D'après les données, on sait que $(EF) // (E'F')$. On voit aussi que $A$, $E'$ et $E$ sont alignés. Il en est de même pour les points $A$, $F'$ et $F$. Donc d'après le théorème de Thalès
\begin{tabular}{|c|c|c|c|}
\hline
Triangle AEF & AE = 141.2 & AF = 52.9 & EF \\
\hline
Triangle AE'F' & AE' = 70.6 & AF' & E'F' = 89.85 \\
\hline
\end{tabular}
est un tableau de proportionnalité. Donc on peut faire un produit en croix pour calcul $EF$.
\begin{align*}
EF = \frac{E'F' \times AE}{AE'} = \frac{89.85 \times 141.2}{70.6} = 179.7 m
\end{align*}
Donc le parcours AEFA mesure
\begin{align*}
AF + AE + EF = 52.9 + 141.2 + 179.7 = 373.8m
\end{align*}
\item Choix du parcours:
Il faudra choisir le tour $AFEA$ car sa longueur est plus proche de 373m.
\end{itemize}
\end{solution}
\end{exercise}
\begin{exercise}[subtitle={Tirages au sort}]
\renewcommand{\arraystretch}{1.5}
Dans une urne, on a placé des boules colorées indiscernables au touché. Il y a 6 boules bleu, 3 boules jaunes, 8 boules vertes et 7 boules rouges.
\begin{enumerate}
\item % Proba
\begin{enumerate}
\item Quelle est la probabilité de tirer une boule bleu?
\begin{solution}
$\dfrac{6}{24} \approx 0.25$
\end{solution}
\item Quelle est la probabilté de tirer une boule jaune ou bleu?
\begin{solution}
$\dfrac{9}{24} \approx 0.38$
\end{solution}
\item A-t-on plus de chance de tirer une boule verte ou une boule rouge?
\begin{solution}
Boules vertes: $\dfrac{8}{24} \approx 0.33$
Boules rouges: $\dfrac{7}{24} \approx 0.29$
Une boule verte
\end{solution}
\end{enumerate}
\item % Stat
On effectue 14 tirages (avec remise) dans cette urne et on obtient les couleurs suivantes:
\begin{center}
V \hspace{0.4cm}B \hspace{0.4cm}V \hspace{0.4cm}V \hspace{0.4cm}J \hspace{0.4cm}R \hspace{0.4cm}R \hspace{0.4cm}V \hspace{0.4cm}V \hspace{0.4cm}V \hspace{0.4cm}R \hspace{0.4cm}B \hspace{0.4cm}B \hspace{0.4cm}V \\
\end{center}
\begin{enumerate}
\item Compléter le tableau des effectifs ci-dessous
\begin{tabular}{|c|*{4}{c|}}
\hline
Couleur & Bleu & Jaune & Vert & Rouge \\
\hline
Effectif & & & & \\
\hline
\end{tabular}
\begin{solution}
\begin{tabular}{|c|*{4}{c|}}
\hline
Couleur & Bleu & Jaune & Vert & Rouge \\
\hline
Effectif & 3 & 1 & 7 & 3 \\
\hline
\end{tabular}
\end{solution}
\item Calculer la fréquence des boules vertes.
\begin{solution}
Fréquence de boules vertes: $\frac{7}{14}$
\end{solution}
\end{enumerate}
\item À chaque couleur, on associe des points. Une boule bleu rapporte 10 points, une boule jaune 5 points, une boule verte 2 points et une boule rouge 0 points.
\begin{enumerate}
\item Combien de points a-t-on gagné au total?
\begin{solution}
17
\end{solution}
\item Calculer la moyenne des gains.
\begin{solution}
4.25
\end{solution}
\item Calculer la médiane des gains.
\begin{solution}
3.5
\end{solution}
\end{enumerate}
\end{enumerate}
\end{exercise}
\end{document}
%%% Local Variables:
%%% mode: latex
%%% TeX-master: "master"
%%% End:

View File

@ -0,0 +1,278 @@
\documentclass[a4paper,12pt]{article}
\usepackage{myXsim}
\usepackage{tkz-fct}
\usepackage{wrapfig}
\title{DM de Paques}
\tribe{302}
\date{Jeudi 3 mai 2018}
\sujet{23}
%\geometry{left=10mm,right=10mm, bottom= 10mm, top=10mm}
\xsimsetup{
solution/print = false
}
\begin{document}
\maketitle
\bigskip
{\Large \textbf{Nom - Prénom:}}
\begin{exercise}[subtitle={Battle of the year}]
% theme: Fonction linéaire, Fonction affine
% require: tkz-fct
Taraina veut inscrire ses 21 élèves à un entrainement pour l'évènement \textbf{Battle of the year}.
Deux tarifs lui sont proposés:
\begin{itemize}
\item Tarif Individuel: 173 \euro par danseur inscrit.
\item Tarif Groupe : Paiement d'un forfait de 571 \euro pour le groupe puis 137 \euro par danseur inscrit.
\end{itemize}
\medskip
\begin{enumerate}
\item Complète le tableau suivant :
\medskip
\begin{tabularx}{0.7\linewidth}{|c|*{3}{>{\centering \arraybackslash}X|}}\hline
Nombre d'inscriptions & 0 & 10 & 25\\
\hline
Prix au tarif Individuel en \euro & & & \\
\hline
Prix au tarif Groupe en \euro & & & \\
\hline
\end{tabularx}
\begin{solution}
\begin{tabularx}{0.7\linewidth}{|c|*{3}{>{\centering \arraybackslash}X|}}\hline
Nombre d'inscriptions & 0 & 10 & 25\\
\hline
Prix au tarif Individuel en \euro & 0 & 1730 & 4325 \\
\hline
Prix au tarif Groupe en \euro & 571 & 1941 & 3996\\
\hline
\end{tabularx}
\end{solution}
\medskip
\item Pour chacun des tarifs, exprimer le prix en fonction du nombre de danseurs inscrits.
\begin{solution}
$x$ représente ici le nombre d'élèves inscrits.
\begin{itemize}
\item Tarif Individuel: $f: x \mapsto 173x$
\item Tarif Groupe: $g: x \mapsto 137x + 571$
\end{itemize}
\end{solution}
\item Tracer sur le graphique suivant, les courbes représentants les 2 tarifs proposés.
% On force que le graphique soit légèrement plus grand que 12 cm
\begin{tikzpicture}[yscale=0.5]
\tkzInit[xmin=0,xmax=26,
ymin=0,ymax=4498,
xstep=2,ystep=200]
\tkzAxeX[thick, poslabel=right,label=]
\tkzAxeY[thick, poslabel=above,label=]
\tkzDrawX[label={\textit{Danseurs inscrits}},below= -12pt]
\tkzDrawY[label={\textit{Prix}}, below=-10pt]
\tkzGrid
\tkzFct[domain=0:26, color=blue, very thick]{173*\x}
\tkzFct[domain=0:26, color=red, very thick]{137*\x+571}
\end{tikzpicture}
\item Pour quel nombre d'inscriptions paye-t-on le même prix quel que soit le tarif choisi?
\begin{solution}
$15.86111111111111$ % est entre 10 et 20 par contruction des paramètres
\end{solution}
\end{enumerate}
\end{exercise}
\begin{exercise}[subtitle={Parcours}]
% exo de geometrie comme au brevet blanc.
Une commune souhaite aménager des parcours de santé sur son territoire. On fait deux propositions au conseil municipale, schématisés ci-dessous:
\begin{itemize}
\item Le parcours ACDA
\item Le parcours AEFA
\end{itemize}
Ils souhaitent faire un parcours dont la longueur s'approche le plus possible de 125m.
Peux-tu les aider à choisir le parcours? Justifie
\textbf{Attention: La figure proposée au conseil municipale n'est pas à l'échelle, mais les codages et les dimension données sont correctes.}
\begin{minipage}{0.6\textwidth}
\includegraphics[scale = 0.4]{./fig/parcours}
\end{minipage}
\begin{minipage}{0.4\textwidth}
\begin{itemize}
\item $AC = 28m$
\item $CD = 45m$
\item $AE' = 12.5m$
\item $AE = 25.0m$
\item $AF = 57.2m$
\item $E'F' = 19.4m$
\item $(E'F') // (EF)$
\item L'angle $\widehat{EAF}$ vaut $30^o$
\end{itemize}
\end{minipage}
\begin{solution}
\begin{itemize}
\item Parcours ACDA:
D'après la figure, on voit que le triangle $ACD$ est rectangle en $C$ donc d'après le théorème de Pythagore, on a
\begin{align*}
AD^2 &= AC^2 + DC^2 \\
AD^2 &= 28^2 + 45^2 \\
AD^2 &= 784 + 2025 \\
AD^2 &= 2809 \\
AD &= \sqrt{2809} = 53m
\end{align*}
Donc le parcours ACDA mesure
\begin{align*}
AD + AC + CD = 53 + 28 + 45 = 126m
\end{align*}
\item Parcours AEFA:
D'après les données, on sait que $(EF) // (E'F')$. On voit aussi que $A$, $E'$ et $E$ sont alignés. Il en est de même pour les points $A$, $F'$ et $F$. Donc d'après le théorème de Thalès
\begin{tabular}{|c|c|c|c|}
\hline
Triangle AEF & AE = 25.0 & AF = 57.2 & EF \\
\hline
Triangle AE'F' & AE' = 12.5 & AF' & E'F' = 19.4 \\
\hline
\end{tabular}
est un tableau de proportionnalité. Donc on peut faire un produit en croix pour calcul $EF$.
\begin{align*}
EF = \frac{E'F' \times AE}{AE'} = \frac{19.4 \times 25.0}{12.5} = 38.8 m
\end{align*}
Donc le parcours AEFA mesure
\begin{align*}
AF + AE + EF = 57.2 + 25.0 + 38.8 = 121.0m
\end{align*}
\item Choix du parcours:
Il faudra choisir le tour $ACDA$ car sa longueur est plus proche de 125m.
\end{itemize}
\end{solution}
\end{exercise}
\begin{exercise}[subtitle={Tirages au sort}]
\renewcommand{\arraystretch}{1.5}
Dans une urne, on a placé des boules colorées indiscernables au touché. Il y a 3 boules bleu, 9 boules jaunes, 4 boules vertes et 7 boules rouges.
\begin{enumerate}
\item % Proba
\begin{enumerate}
\item Quelle est la probabilité de tirer une boule bleu?
\begin{solution}
$\dfrac{3}{23} \approx 0.13$
\end{solution}
\item Quelle est la probabilté de tirer une boule jaune ou bleu?
\begin{solution}
$\dfrac{12}{23} \approx 0.52$
\end{solution}
\item A-t-on plus de chance de tirer une boule verte ou une boule rouge?
\begin{solution}
Boules vertes: $\dfrac{4}{23} \approx 0.17$
Boules rouges: $\dfrac{7}{23} \approx 0.3$
Une boule rouge
\end{solution}
\end{enumerate}
\item % Stat
On effectue 14 tirages (avec remise) dans cette urne et on obtient les couleurs suivantes:
\begin{center}
R \hspace{0.4cm}R \hspace{0.4cm}B \hspace{0.4cm}R \hspace{0.4cm}J \hspace{0.4cm}J \hspace{0.4cm}V \hspace{0.4cm}B \hspace{0.4cm}R \hspace{0.4cm}J \hspace{0.4cm}R \hspace{0.4cm}R \hspace{0.4cm}R \hspace{0.4cm}J \\
\end{center}
\begin{enumerate}
\item Compléter le tableau des effectifs ci-dessous
\begin{tabular}{|c|*{4}{c|}}
\hline
Couleur & Bleu & Jaune & Vert & Rouge \\
\hline
Effectif & & & & \\
\hline
\end{tabular}
\begin{solution}
\begin{tabular}{|c|*{4}{c|}}
\hline
Couleur & Bleu & Jaune & Vert & Rouge \\
\hline
Effectif & 2 & 4 & 1 & 7 \\
\hline
\end{tabular}
\end{solution}
\item Calculer la fréquence des boules vertes.
\begin{solution}
Fréquence de boules vertes: $\frac{1}{14}$
\end{solution}
\end{enumerate}
\item À chaque couleur, on associe des points. Une boule bleu rapporte 10 points, une boule jaune 5 points, une boule verte 2 points et une boule rouge 0 points.
\begin{enumerate}
\item Combien de points a-t-on gagné au total?
\begin{solution}
17
\end{solution}
\item Calculer la moyenne des gains.
\begin{solution}
4.25
\end{solution}
\item Calculer la médiane des gains.
\begin{solution}
3.5
\end{solution}
\end{enumerate}
\end{enumerate}
\end{exercise}
\end{document}
%%% Local Variables:
%%% mode: latex
%%% TeX-master: "master"
%%% End:

View File

@ -0,0 +1,278 @@
\documentclass[a4paper,12pt]{article}
\usepackage{myXsim}
\usepackage{tkz-fct}
\usepackage{wrapfig}
\title{DM de Paques}
\tribe{302}
\date{Jeudi 3 mai 2018}
\sujet{24}
%\geometry{left=10mm,right=10mm, bottom= 10mm, top=10mm}
\xsimsetup{
solution/print = false
}
\begin{document}
\maketitle
\bigskip
{\Large \textbf{Nom - Prénom:}}
\begin{exercise}[subtitle={Battle of the year}]
% theme: Fonction linéaire, Fonction affine
% require: tkz-fct
Taraina veut inscrire ses 21 élèves à un entrainement pour l'évènement \textbf{Battle of the year}.
Deux tarifs lui sont proposés:
\begin{itemize}
\item Tarif Individuel: 118 \euro par danseur inscrit.
\item Tarif Groupe : Paiement d'un forfait de 444 \euro pour le groupe puis 84 \euro par danseur inscrit.
\end{itemize}
\medskip
\begin{enumerate}
\item Complète le tableau suivant :
\medskip
\begin{tabularx}{0.7\linewidth}{|c|*{3}{>{\centering \arraybackslash}X|}}\hline
Nombre d'inscriptions & 0 & 10 & 25\\
\hline
Prix au tarif Individuel en \euro & & & \\
\hline
Prix au tarif Groupe en \euro & & & \\
\hline
\end{tabularx}
\begin{solution}
\begin{tabularx}{0.7\linewidth}{|c|*{3}{>{\centering \arraybackslash}X|}}\hline
Nombre d'inscriptions & 0 & 10 & 25\\
\hline
Prix au tarif Individuel en \euro & 0 & 1180 & 2950 \\
\hline
Prix au tarif Groupe en \euro & 444 & 1284 & 2544\\
\hline
\end{tabularx}
\end{solution}
\medskip
\item Pour chacun des tarifs, exprimer le prix en fonction du nombre de danseurs inscrits.
\begin{solution}
$x$ représente ici le nombre d'élèves inscrits.
\begin{itemize}
\item Tarif Individuel: $f: x \mapsto 118x$
\item Tarif Groupe: $g: x \mapsto 84x + 444$
\end{itemize}
\end{solution}
\item Tracer sur le graphique suivant, les courbes représentants les 2 tarifs proposés.
% On force que le graphique soit légèrement plus grand que 12 cm
\begin{tikzpicture}[yscale=0.8]
\tkzInit[xmin=0,xmax=26,
ymin=0,ymax=3068,
xstep=2,ystep=200]
\tkzAxeX[thick, poslabel=right,label=]
\tkzAxeY[thick, poslabel=above,label=]
\tkzDrawX[label={\textit{Danseurs inscrits}},below= -12pt]
\tkzDrawY[label={\textit{Prix}}, below=-10pt]
\tkzGrid
\tkzFct[domain=0:26, color=blue, very thick]{118*\x}
\tkzFct[domain=0:26, color=red, very thick]{84*\x+444}
\end{tikzpicture}
\item Pour quel nombre d'inscriptions paye-t-on le même prix quel que soit le tarif choisi?
\begin{solution}
$13.058823529411764$ % est entre 10 et 20 par contruction des paramètres
\end{solution}
\end{enumerate}
\end{exercise}
\begin{exercise}[subtitle={Parcours}]
% exo de geometrie comme au brevet blanc.
Une commune souhaite aménager des parcours de santé sur son territoire. On fait deux propositions au conseil municipale, schématisés ci-dessous:
\begin{itemize}
\item Le parcours ACDA
\item Le parcours AEFA
\end{itemize}
Ils souhaitent faire un parcours dont la longueur s'approche le plus possible de 96km.
Peux-tu les aider à choisir le parcours? Justifie
\textbf{Attention: La figure proposée au conseil municipale n'est pas à l'échelle, mais les codages et les dimension données sont correctes.}
\begin{minipage}{0.6\textwidth}
\includegraphics[scale = 0.4]{./fig/parcours}
\end{minipage}
\begin{minipage}{0.4\textwidth}
\begin{itemize}
\item $AC = 24km$
\item $CD = 32km$
\item $AE' = 6.65km$
\item $AE = 26.6km$
\item $AF = 13.7km$
\item $E'F' = 12.18km$
\item $(E'F') // (EF)$
\item L'angle $\widehat{EAF}$ vaut $30^o$
\end{itemize}
\end{minipage}
\begin{solution}
\begin{itemize}
\item Parcours ACDA:
D'après la figure, on voit que le triangle $ACD$ est rectangle en $C$ donc d'après le théorème de Pythagore, on a
\begin{align*}
AD^2 &= AC^2 + DC^2 \\
AD^2 &= 24^2 + 32^2 \\
AD^2 &= 576 + 1024 \\
AD^2 &= 1600 \\
AD &= \sqrt{1600} = 40km
\end{align*}
Donc le parcours ACDA mesure
\begin{align*}
AD + AC + CD = 40 + 24 + 32 = 96km
\end{align*}
\item Parcours AEFA:
D'après les données, on sait que $(EF) // (E'F')$. On voit aussi que $A$, $E'$ et $E$ sont alignés. Il en est de même pour les points $A$, $F'$ et $F$. Donc d'après le théorème de Thalès
\begin{tabular}{|c|c|c|c|}
\hline
Triangle AEF & AE = 26.6 & AF = 13.7 & EF \\
\hline
Triangle AE'F' & AE' = 6.65 & AF' & E'F' = 12.18 \\
\hline
\end{tabular}
est un tableau de proportionnalité. Donc on peut faire un produit en croix pour calcul $EF$.
\begin{align*}
EF = \frac{E'F' \times AE}{AE'} = \frac{12.18 \times 26.6}{6.65} = 48.7 km
\end{align*}
Donc le parcours AEFA mesure
\begin{align*}
AF + AE + EF = 13.7 + 26.6 + 48.7 = 89.0km
\end{align*}
\item Choix du parcours:
Il faudra choisir le tour $ACDA$ car sa longueur est plus proche de 96km.
\end{itemize}
\end{solution}
\end{exercise}
\begin{exercise}[subtitle={Tirages au sort}]
\renewcommand{\arraystretch}{1.5}
Dans une urne, on a placé des boules colorées indiscernables au touché. Il y a 3 boules bleu, 8 boules jaunes, 9 boules vertes et 7 boules rouges.
\begin{enumerate}
\item % Proba
\begin{enumerate}
\item Quelle est la probabilité de tirer une boule bleu?
\begin{solution}
$\dfrac{3}{27} \approx 0.11$
\end{solution}
\item Quelle est la probabilté de tirer une boule jaune ou bleu?
\begin{solution}
$\dfrac{11}{27} \approx 0.41$
\end{solution}
\item A-t-on plus de chance de tirer une boule verte ou une boule rouge?
\begin{solution}
Boules vertes: $\dfrac{9}{27} \approx 0.33$
Boules rouges: $\dfrac{7}{27} \approx 0.26$
Une boule verte
\end{solution}
\end{enumerate}
\item % Stat
On effectue 14 tirages (avec remise) dans cette urne et on obtient les couleurs suivantes:
\begin{center}
V \hspace{0.4cm}J \hspace{0.4cm}R \hspace{0.4cm}V \hspace{0.4cm}V \hspace{0.4cm}V \hspace{0.4cm}V \hspace{0.4cm}J \hspace{0.4cm}R \hspace{0.4cm}R \hspace{0.4cm}J \hspace{0.4cm}J \hspace{0.4cm}J \hspace{0.4cm}R \\
\end{center}
\begin{enumerate}
\item Compléter le tableau des effectifs ci-dessous
\begin{tabular}{|c|*{4}{c|}}
\hline
Couleur & Bleu & Jaune & Vert & Rouge \\
\hline
Effectif & & & & \\
\hline
\end{tabular}
\begin{solution}
\begin{tabular}{|c|*{4}{c|}}
\hline
Couleur & Bleu & Jaune & Vert & Rouge \\
\hline
Effectif & & 5 & 5 & 4 \\
\hline
\end{tabular}
\end{solution}
\item Calculer la fréquence des boules vertes.
\begin{solution}
Fréquence de boules vertes: $\frac{5}{14}$
\end{solution}
\end{enumerate}
\item À chaque couleur, on associe des points. Une boule bleu rapporte 10 points, une boule jaune 5 points, une boule verte 2 points et une boule rouge 0 points.
\begin{enumerate}
\item Combien de points a-t-on gagné au total?
\begin{solution}
17
\end{solution}
\item Calculer la moyenne des gains.
\begin{solution}
4.25
\end{solution}
\item Calculer la médiane des gains.
\begin{solution}
3.5
\end{solution}
\end{enumerate}
\end{enumerate}
\end{exercise}
\end{document}
%%% Local Variables:
%%% mode: latex
%%% TeX-master: "master"
%%% End:

View File

@ -0,0 +1,278 @@
\documentclass[a4paper,12pt]{article}
\usepackage{myXsim}
\usepackage{tkz-fct}
\usepackage{wrapfig}
\title{DM de Paques}
\tribe{302}
\date{Jeudi 3 mai 2018}
\sujet{25}
%\geometry{left=10mm,right=10mm, bottom= 10mm, top=10mm}
\xsimsetup{
solution/print = false
}
\begin{document}
\maketitle
\bigskip
{\Large \textbf{Nom - Prénom:}}
\begin{exercise}[subtitle={Battle of the year}]
% theme: Fonction linéaire, Fonction affine
% require: tkz-fct
Taraina veut inscrire ses 21 élèves à un entrainement pour l'évènement \textbf{Battle of the year}.
Deux tarifs lui sont proposés:
\begin{itemize}
\item Tarif Individuel: 148 \euro par danseur inscrit.
\item Tarif Groupe : Paiement d'un forfait de 589 \euro pour le groupe puis 114 \euro par danseur inscrit.
\end{itemize}
\medskip
\begin{enumerate}
\item Complète le tableau suivant :
\medskip
\begin{tabularx}{0.7\linewidth}{|c|*{3}{>{\centering \arraybackslash}X|}}\hline
Nombre d'inscriptions & 0 & 10 & 25\\
\hline
Prix au tarif Individuel en \euro & & & \\
\hline
Prix au tarif Groupe en \euro & & & \\
\hline
\end{tabularx}
\begin{solution}
\begin{tabularx}{0.7\linewidth}{|c|*{3}{>{\centering \arraybackslash}X|}}\hline
Nombre d'inscriptions & 0 & 10 & 25\\
\hline
Prix au tarif Individuel en \euro & 0 & 1480 & 3700 \\
\hline
Prix au tarif Groupe en \euro & 589 & 1729 & 3439\\
\hline
\end{tabularx}
\end{solution}
\medskip
\item Pour chacun des tarifs, exprimer le prix en fonction du nombre de danseurs inscrits.
\begin{solution}
$x$ représente ici le nombre d'élèves inscrits.
\begin{itemize}
\item Tarif Individuel: $f: x \mapsto 148x$
\item Tarif Groupe: $g: x \mapsto 114x + 589$
\end{itemize}
\end{solution}
\item Tracer sur le graphique suivant, les courbes représentants les 2 tarifs proposés.
% On force que le graphique soit légèrement plus grand que 12 cm
\begin{tikzpicture}[yscale=0.6]
\tkzInit[xmin=0,xmax=26,
ymin=0,ymax=3848,
xstep=2,ystep=200]
\tkzAxeX[thick, poslabel=right,label=]
\tkzAxeY[thick, poslabel=above,label=]
\tkzDrawX[label={\textit{Danseurs inscrits}},below= -12pt]
\tkzDrawY[label={\textit{Prix}}, below=-10pt]
\tkzGrid
\tkzFct[domain=0:26, color=blue, very thick]{148*\x}
\tkzFct[domain=0:26, color=red, very thick]{114*\x+589}
\end{tikzpicture}
\item Pour quel nombre d'inscriptions paye-t-on le même prix quel que soit le tarif choisi?
\begin{solution}
$17.323529411764707$ % est entre 10 et 20 par contruction des paramètres
\end{solution}
\end{enumerate}
\end{exercise}
\begin{exercise}[subtitle={Parcours}]
% exo de geometrie comme au brevet blanc.
Une commune souhaite aménager des parcours de santé sur son territoire. On fait deux propositions au conseil municipale, schématisés ci-dessous:
\begin{itemize}
\item Le parcours ACDA
\item Le parcours AEFA
\end{itemize}
Ils souhaitent faire un parcours dont la longueur s'approche le plus possible de 286m.
Peux-tu les aider à choisir le parcours? Justifie
\textbf{Attention: La figure proposée au conseil municipale n'est pas à l'échelle, mais les codages et les dimension données sont correctes.}
\begin{minipage}{0.6\textwidth}
\includegraphics[scale = 0.4]{./fig/parcours}
\end{minipage}
\begin{minipage}{0.4\textwidth}
\begin{itemize}
\item $AC = 126m$
\item $CD = 32m$
\item $AE' = 21.4m$
\item $AE = 42.8m$
\item $AF = 120.5m$
\item $E'F' = 59.95m$
\item $(E'F') // (EF)$
\item L'angle $\widehat{EAF}$ vaut $30^o$
\end{itemize}
\end{minipage}
\begin{solution}
\begin{itemize}
\item Parcours ACDA:
D'après la figure, on voit que le triangle $ACD$ est rectangle en $C$ donc d'après le théorème de Pythagore, on a
\begin{align*}
AD^2 &= AC^2 + DC^2 \\
AD^2 &= 126^2 + 32^2 \\
AD^2 &= 15876 + 1024 \\
AD^2 &= 16900 \\
AD &= \sqrt{16900} = 130m
\end{align*}
Donc le parcours ACDA mesure
\begin{align*}
AD + AC + CD = 130 + 126 + 32 = 288m
\end{align*}
\item Parcours AEFA:
D'après les données, on sait que $(EF) // (E'F')$. On voit aussi que $A$, $E'$ et $E$ sont alignés. Il en est de même pour les points $A$, $F'$ et $F$. Donc d'après le théorème de Thalès
\begin{tabular}{|c|c|c|c|}
\hline
Triangle AEF & AE = 42.8 & AF = 120.5 & EF \\
\hline
Triangle AE'F' & AE' = 21.4 & AF' & E'F' = 59.95 \\
\hline
\end{tabular}
est un tableau de proportionnalité. Donc on peut faire un produit en croix pour calcul $EF$.
\begin{align*}
EF = \frac{E'F' \times AE}{AE'} = \frac{59.95 \times 42.8}{21.4} = 119.9 m
\end{align*}
Donc le parcours AEFA mesure
\begin{align*}
AF + AE + EF = 120.5 + 42.8 + 119.9 = 283.2m
\end{align*}
\item Choix du parcours:
Il faudra choisir le tour $ACDA$ car sa longueur est plus proche de 286m.
\end{itemize}
\end{solution}
\end{exercise}
\begin{exercise}[subtitle={Tirages au sort}]
\renewcommand{\arraystretch}{1.5}
Dans une urne, on a placé des boules colorées indiscernables au touché. Il y a 8 boules bleu, 6 boules jaunes, 6 boules vertes et 2 boules rouges.
\begin{enumerate}
\item % Proba
\begin{enumerate}
\item Quelle est la probabilité de tirer une boule bleu?
\begin{solution}
$\dfrac{8}{22} \approx 0.36$
\end{solution}
\item Quelle est la probabilté de tirer une boule jaune ou bleu?
\begin{solution}
$\dfrac{14}{22} \approx 0.64$
\end{solution}
\item A-t-on plus de chance de tirer une boule verte ou une boule rouge?
\begin{solution}
Boules vertes: $\dfrac{6}{22} \approx 0.27$
Boules rouges: $\dfrac{2}{22} \approx 0.09$
Une boule verte
\end{solution}
\end{enumerate}
\item % Stat
On effectue 14 tirages (avec remise) dans cette urne et on obtient les couleurs suivantes:
\begin{center}
B \hspace{0.4cm}V \hspace{0.4cm}J \hspace{0.4cm}B \hspace{0.4cm}J \hspace{0.4cm}J \hspace{0.4cm}V \hspace{0.4cm}V \hspace{0.4cm}B \hspace{0.4cm}B \hspace{0.4cm}J \hspace{0.4cm}B \hspace{0.4cm}B \hspace{0.4cm}V \\
\end{center}
\begin{enumerate}
\item Compléter le tableau des effectifs ci-dessous
\begin{tabular}{|c|*{4}{c|}}
\hline
Couleur & Bleu & Jaune & Vert & Rouge \\
\hline
Effectif & & & & \\
\hline
\end{tabular}
\begin{solution}
\begin{tabular}{|c|*{4}{c|}}
\hline
Couleur & Bleu & Jaune & Vert & Rouge \\
\hline
Effectif & 6 & 4 & 4 & \\
\hline
\end{tabular}
\end{solution}
\item Calculer la fréquence des boules vertes.
\begin{solution}
Fréquence de boules vertes: $\frac{4}{14}$
\end{solution}
\end{enumerate}
\item À chaque couleur, on associe des points. Une boule bleu rapporte 10 points, une boule jaune 5 points, une boule verte 2 points et une boule rouge 0 points.
\begin{enumerate}
\item Combien de points a-t-on gagné au total?
\begin{solution}
17
\end{solution}
\item Calculer la moyenne des gains.
\begin{solution}
4.25
\end{solution}
\item Calculer la médiane des gains.
\begin{solution}
3.5
\end{solution}
\end{enumerate}
\end{enumerate}
\end{exercise}
\end{document}
%%% Local Variables:
%%% mode: latex
%%% TeX-master: "master"
%%% End:

View File

@ -0,0 +1,278 @@
\documentclass[a4paper,12pt]{article}
\usepackage{myXsim}
\usepackage{tkz-fct}
\usepackage{wrapfig}
\title{DM de Paques}
\tribe{302}
\date{Jeudi 3 mai 2018}
\sujet{26}
%\geometry{left=10mm,right=10mm, bottom= 10mm, top=10mm}
\xsimsetup{
solution/print = false
}
\begin{document}
\maketitle
\bigskip
{\Large \textbf{Nom - Prénom:}}
\begin{exercise}[subtitle={Battle of the year}]
% theme: Fonction linéaire, Fonction affine
% require: tkz-fct
Taraina veut inscrire ses 21 élèves à un entrainement pour l'évènement \textbf{Battle of the year}.
Deux tarifs lui sont proposés:
\begin{itemize}
\item Tarif Individuel: 113 \euro par danseur inscrit.
\item Tarif Groupe : Paiement d'un forfait de 449 \euro pour le groupe puis 80 \euro par danseur inscrit.
\end{itemize}
\medskip
\begin{enumerate}
\item Complète le tableau suivant :
\medskip
\begin{tabularx}{0.7\linewidth}{|c|*{3}{>{\centering \arraybackslash}X|}}\hline
Nombre d'inscriptions & 0 & 10 & 25\\
\hline
Prix au tarif Individuel en \euro & & & \\
\hline
Prix au tarif Groupe en \euro & & & \\
\hline
\end{tabularx}
\begin{solution}
\begin{tabularx}{0.7\linewidth}{|c|*{3}{>{\centering \arraybackslash}X|}}\hline
Nombre d'inscriptions & 0 & 10 & 25\\
\hline
Prix au tarif Individuel en \euro & 0 & 1130 & 2825 \\
\hline
Prix au tarif Groupe en \euro & 449 & 1249 & 2449\\
\hline
\end{tabularx}
\end{solution}
\medskip
\item Pour chacun des tarifs, exprimer le prix en fonction du nombre de danseurs inscrits.
\begin{solution}
$x$ représente ici le nombre d'élèves inscrits.
\begin{itemize}
\item Tarif Individuel: $f: x \mapsto 113x$
\item Tarif Groupe: $g: x \mapsto 80x + 449$
\end{itemize}
\end{solution}
\item Tracer sur le graphique suivant, les courbes représentants les 2 tarifs proposés.
% On force que le graphique soit légèrement plus grand que 12 cm
\begin{tikzpicture}[yscale=0.8]
\tkzInit[xmin=0,xmax=26,
ymin=0,ymax=2938,
xstep=2,ystep=200]
\tkzAxeX[thick, poslabel=right,label=]
\tkzAxeY[thick, poslabel=above,label=]
\tkzDrawX[label={\textit{Danseurs inscrits}},below= -12pt]
\tkzDrawY[label={\textit{Prix}}, below=-10pt]
\tkzGrid
\tkzFct[domain=0:26, color=blue, very thick]{113*\x}
\tkzFct[domain=0:26, color=red, very thick]{80*\x+449}
\end{tikzpicture}
\item Pour quel nombre d'inscriptions paye-t-on le même prix quel que soit le tarif choisi?
\begin{solution}
$13.606060606060606$ % est entre 10 et 20 par contruction des paramètres
\end{solution}
\end{enumerate}
\end{exercise}
\begin{exercise}[subtitle={Parcours}]
% exo de geometrie comme au brevet blanc.
Une commune souhaite aménager des parcours de santé sur son territoire. On fait deux propositions au conseil municipale, schématisés ci-dessous:
\begin{itemize}
\item Le parcours ACDA
\item Le parcours AEFA
\end{itemize}
Ils souhaitent faire un parcours dont la longueur s'approche le plus possible de 237m.
Peux-tu les aider à choisir le parcours? Justifie
\textbf{Attention: La figure proposée au conseil municipale n'est pas à l'échelle, mais les codages et les dimension données sont correctes.}
\begin{minipage}{0.6\textwidth}
\includegraphics[scale = 0.4]{./fig/parcours}
\end{minipage}
\begin{minipage}{0.4\textwidth}
\begin{itemize}
\item $AC = 40m$
\item $CD = 96m$
\item $AE' = 23.17m$
\item $AE = 69.5m$
\item $AF = 2.2m$
\item $E'F' = 54.57m$
\item $(E'F') // (EF)$
\item L'angle $\widehat{EAF}$ vaut $30^o$
\end{itemize}
\end{minipage}
\begin{solution}
\begin{itemize}
\item Parcours ACDA:
D'après la figure, on voit que le triangle $ACD$ est rectangle en $C$ donc d'après le théorème de Pythagore, on a
\begin{align*}
AD^2 &= AC^2 + DC^2 \\
AD^2 &= 40^2 + 96^2 \\
AD^2 &= 1600 + 9216 \\
AD^2 &= 10816 \\
AD &= \sqrt{10816} = 104m
\end{align*}
Donc le parcours ACDA mesure
\begin{align*}
AD + AC + CD = 104 + 40 + 96 = 240m
\end{align*}
\item Parcours AEFA:
D'après les données, on sait que $(EF) // (E'F')$. On voit aussi que $A$, $E'$ et $E$ sont alignés. Il en est de même pour les points $A$, $F'$ et $F$. Donc d'après le théorème de Thalès
\begin{tabular}{|c|c|c|c|}
\hline
Triangle AEF & AE = 69.5 & AF = 2.2 & EF \\
\hline
Triangle AE'F' & AE' = 23.17 & AF' & E'F' = 54.57 \\
\hline
\end{tabular}
est un tableau de proportionnalité. Donc on peut faire un produit en croix pour calcul $EF$.
\begin{align*}
EF = \frac{E'F' \times AE}{AE'} = \frac{54.57 \times 69.5}{23.17} = 163.7 m
\end{align*}
Donc le parcours AEFA mesure
\begin{align*}
AF + AE + EF = 2.2 + 69.5 + 163.7 = 235.4m
\end{align*}
\item Choix du parcours:
Il faudra choisir le tour $AFEA$ car sa longueur est plus proche de 237m.
\end{itemize}
\end{solution}
\end{exercise}
\begin{exercise}[subtitle={Tirages au sort}]
\renewcommand{\arraystretch}{1.5}
Dans une urne, on a placé des boules colorées indiscernables au touché. Il y a 2 boules bleu, 5 boules jaunes, 3 boules vertes et 6 boules rouges.
\begin{enumerate}
\item % Proba
\begin{enumerate}
\item Quelle est la probabilité de tirer une boule bleu?
\begin{solution}
$\dfrac{2}{16} \approx 0.12$
\end{solution}
\item Quelle est la probabilté de tirer une boule jaune ou bleu?
\begin{solution}
$\dfrac{7}{16} \approx 0.44$
\end{solution}
\item A-t-on plus de chance de tirer une boule verte ou une boule rouge?
\begin{solution}
Boules vertes: $\dfrac{3}{16} \approx 0.19$
Boules rouges: $\dfrac{6}{16} \approx 0.38$
Une boule rouge
\end{solution}
\end{enumerate}
\item % Stat
On effectue 14 tirages (avec remise) dans cette urne et on obtient les couleurs suivantes:
\begin{center}
B \hspace{0.4cm}J \hspace{0.4cm}R \hspace{0.4cm}J \hspace{0.4cm}B \hspace{0.4cm}V \hspace{0.4cm}J \hspace{0.4cm}V \hspace{0.4cm}J \hspace{0.4cm}J \hspace{0.4cm}V \hspace{0.4cm}J \hspace{0.4cm}J \hspace{0.4cm}R \\
\end{center}
\begin{enumerate}
\item Compléter le tableau des effectifs ci-dessous
\begin{tabular}{|c|*{4}{c|}}
\hline
Couleur & Bleu & Jaune & Vert & Rouge \\
\hline
Effectif & & & & \\
\hline
\end{tabular}
\begin{solution}
\begin{tabular}{|c|*{4}{c|}}
\hline
Couleur & Bleu & Jaune & Vert & Rouge \\
\hline
Effectif & 2 & 7 & 3 & 2 \\
\hline
\end{tabular}
\end{solution}
\item Calculer la fréquence des boules vertes.
\begin{solution}
Fréquence de boules vertes: $\frac{3}{14}$
\end{solution}
\end{enumerate}
\item À chaque couleur, on associe des points. Une boule bleu rapporte 10 points, une boule jaune 5 points, une boule verte 2 points et une boule rouge 0 points.
\begin{enumerate}
\item Combien de points a-t-on gagné au total?
\begin{solution}
17
\end{solution}
\item Calculer la moyenne des gains.
\begin{solution}
4.25
\end{solution}
\item Calculer la médiane des gains.
\begin{solution}
3.5
\end{solution}
\end{enumerate}
\end{enumerate}
\end{exercise}
\end{document}
%%% Local Variables:
%%% mode: latex
%%% TeX-master: "master"
%%% End:

View File

@ -0,0 +1,278 @@
\documentclass[a4paper,12pt]{article}
\usepackage{myXsim}
\usepackage{tkz-fct}
\usepackage{wrapfig}
\title{DM de Paques}
\tribe{302}
\date{Jeudi 3 mai 2018}
\sujet{27}
%\geometry{left=10mm,right=10mm, bottom= 10mm, top=10mm}
\xsimsetup{
solution/print = false
}
\begin{document}
\maketitle
\bigskip
{\Large \textbf{Nom - Prénom:}}
\begin{exercise}[subtitle={Battle of the year}]
% theme: Fonction linéaire, Fonction affine
% require: tkz-fct
Taraina veut inscrire ses 21 élèves à un entrainement pour l'évènement \textbf{Battle of the year}.
Deux tarifs lui sont proposés:
\begin{itemize}
\item Tarif Individuel: 146 \euro par danseur inscrit.
\item Tarif Groupe : Paiement d'un forfait de 494 \euro pour le groupe puis 111 \euro par danseur inscrit.
\end{itemize}
\medskip
\begin{enumerate}
\item Complète le tableau suivant :
\medskip
\begin{tabularx}{0.7\linewidth}{|c|*{3}{>{\centering \arraybackslash}X|}}\hline
Nombre d'inscriptions & 0 & 10 & 25\\
\hline
Prix au tarif Individuel en \euro & & & \\
\hline
Prix au tarif Groupe en \euro & & & \\
\hline
\end{tabularx}
\begin{solution}
\begin{tabularx}{0.7\linewidth}{|c|*{3}{>{\centering \arraybackslash}X|}}\hline
Nombre d'inscriptions & 0 & 10 & 25\\
\hline
Prix au tarif Individuel en \euro & 0 & 1460 & 3650 \\
\hline
Prix au tarif Groupe en \euro & 494 & 1604 & 3269\\
\hline
\end{tabularx}
\end{solution}
\medskip
\item Pour chacun des tarifs, exprimer le prix en fonction du nombre de danseurs inscrits.
\begin{solution}
$x$ représente ici le nombre d'élèves inscrits.
\begin{itemize}
\item Tarif Individuel: $f: x \mapsto 146x$
\item Tarif Groupe: $g: x \mapsto 111x + 494$
\end{itemize}
\end{solution}
\item Tracer sur le graphique suivant, les courbes représentants les 2 tarifs proposés.
% On force que le graphique soit légèrement plus grand que 12 cm
\begin{tikzpicture}[yscale=0.6]
\tkzInit[xmin=0,xmax=26,
ymin=0,ymax=3796,
xstep=2,ystep=200]
\tkzAxeX[thick, poslabel=right,label=]
\tkzAxeY[thick, poslabel=above,label=]
\tkzDrawX[label={\textit{Danseurs inscrits}},below= -12pt]
\tkzDrawY[label={\textit{Prix}}, below=-10pt]
\tkzGrid
\tkzFct[domain=0:26, color=blue, very thick]{146*\x}
\tkzFct[domain=0:26, color=red, very thick]{111*\x+494}
\end{tikzpicture}
\item Pour quel nombre d'inscriptions paye-t-on le même prix quel que soit le tarif choisi?
\begin{solution}
$14.114285714285714$ % est entre 10 et 20 par contruction des paramètres
\end{solution}
\end{enumerate}
\end{exercise}
\begin{exercise}[subtitle={Parcours}]
% exo de geometrie comme au brevet blanc.
Une commune souhaite aménager des parcours de santé sur son territoire. On fait deux propositions au conseil municipale, schématisés ci-dessous:
\begin{itemize}
\item Le parcours ACDA
\item Le parcours AEFA
\end{itemize}
Ils souhaitent faire un parcours dont la longueur s'approche le plus possible de 62km.
Peux-tu les aider à choisir le parcours? Justifie
\textbf{Attention: La figure proposée au conseil municipale n'est pas à l'échelle, mais les codages et les dimension données sont correctes.}
\begin{minipage}{0.6\textwidth}
\includegraphics[scale = 0.4]{./fig/parcours}
\end{minipage}
\begin{minipage}{0.4\textwidth}
\begin{itemize}
\item $AC = 20km$
\item $CD = 21km$
\item $AE' = 1.2km$
\item $AE = 3.6km$
\item $AF = 8.3km$
\item $E'F' = 16.77km$
\item $(E'F') // (EF)$
\item L'angle $\widehat{EAF}$ vaut $30^o$
\end{itemize}
\end{minipage}
\begin{solution}
\begin{itemize}
\item Parcours ACDA:
D'après la figure, on voit que le triangle $ACD$ est rectangle en $C$ donc d'après le théorème de Pythagore, on a
\begin{align*}
AD^2 &= AC^2 + DC^2 \\
AD^2 &= 20^2 + 21^2 \\
AD^2 &= 400 + 441 \\
AD^2 &= 841 \\
AD &= \sqrt{841} = 29km
\end{align*}
Donc le parcours ACDA mesure
\begin{align*}
AD + AC + CD = 29 + 20 + 21 = 70km
\end{align*}
\item Parcours AEFA:
D'après les données, on sait que $(EF) // (E'F')$. On voit aussi que $A$, $E'$ et $E$ sont alignés. Il en est de même pour les points $A$, $F'$ et $F$. Donc d'après le théorème de Thalès
\begin{tabular}{|c|c|c|c|}
\hline
Triangle AEF & AE = 3.6 & AF = 8.3 & EF \\
\hline
Triangle AE'F' & AE' = 1.2 & AF' & E'F' = 16.77 \\
\hline
\end{tabular}
est un tableau de proportionnalité. Donc on peut faire un produit en croix pour calcul $EF$.
\begin{align*}
EF = \frac{E'F' \times AE}{AE'} = \frac{16.77 \times 3.6}{1.2} = 50.3 km
\end{align*}
Donc le parcours AEFA mesure
\begin{align*}
AF + AE + EF = 8.3 + 3.6 + 50.3 = 62.2km
\end{align*}
\item Choix du parcours:
Il faudra choisir le tour $AFEA$ car sa longueur est plus proche de 62km.
\end{itemize}
\end{solution}
\end{exercise}
\begin{exercise}[subtitle={Tirages au sort}]
\renewcommand{\arraystretch}{1.5}
Dans une urne, on a placé des boules colorées indiscernables au touché. Il y a 4 boules bleu, 6 boules jaunes, 7 boules vertes et 9 boules rouges.
\begin{enumerate}
\item % Proba
\begin{enumerate}
\item Quelle est la probabilité de tirer une boule bleu?
\begin{solution}
$\dfrac{4}{26} \approx 0.15$
\end{solution}
\item Quelle est la probabilté de tirer une boule jaune ou bleu?
\begin{solution}
$\dfrac{10}{26} \approx 0.38$
\end{solution}
\item A-t-on plus de chance de tirer une boule verte ou une boule rouge?
\begin{solution}
Boules vertes: $\dfrac{7}{26} \approx 0.27$
Boules rouges: $\dfrac{9}{26} \approx 0.35$
Une boule rouge
\end{solution}
\end{enumerate}
\item % Stat
On effectue 14 tirages (avec remise) dans cette urne et on obtient les couleurs suivantes:
\begin{center}
B \hspace{0.4cm}R \hspace{0.4cm}V \hspace{0.4cm}R \hspace{0.4cm}R \hspace{0.4cm}J \hspace{0.4cm}J \hspace{0.4cm}V \hspace{0.4cm}V \hspace{0.4cm}J \hspace{0.4cm}R \hspace{0.4cm}R \hspace{0.4cm}R \hspace{0.4cm}R \\
\end{center}
\begin{enumerate}
\item Compléter le tableau des effectifs ci-dessous
\begin{tabular}{|c|*{4}{c|}}
\hline
Couleur & Bleu & Jaune & Vert & Rouge \\
\hline
Effectif & & & & \\
\hline
\end{tabular}
\begin{solution}
\begin{tabular}{|c|*{4}{c|}}
\hline
Couleur & Bleu & Jaune & Vert & Rouge \\
\hline
Effectif & 1 & 3 & 3 & 7 \\
\hline
\end{tabular}
\end{solution}
\item Calculer la fréquence des boules vertes.
\begin{solution}
Fréquence de boules vertes: $\frac{3}{14}$
\end{solution}
\end{enumerate}
\item À chaque couleur, on associe des points. Une boule bleu rapporte 10 points, une boule jaune 5 points, une boule verte 2 points et une boule rouge 0 points.
\begin{enumerate}
\item Combien de points a-t-on gagné au total?
\begin{solution}
17
\end{solution}
\item Calculer la moyenne des gains.
\begin{solution}
4.25
\end{solution}
\item Calculer la médiane des gains.
\begin{solution}
3.5
\end{solution}
\end{enumerate}
\end{enumerate}
\end{exercise}
\end{document}
%%% Local Variables:
%%% mode: latex
%%% TeX-master: "master"
%%% End:

View File

@ -0,0 +1,100 @@
\renewcommand{\arraystretch}{1.5}
\Block{set bBleu = randint(2,10)}
\Block{set bJaune = randint(2,10)}
\Block{set bVerte = randint(2,10)}
\Block{set bRouge = randint(2,10)}
\Block{set nbrTot = bBleu + bJaune + bVerte + bRouge}
Dans une urne, on a placé des boules colorées indiscernables au touché. Il y a \Var{bBleu} boules bleu, \Var{bJaune} boules jaunes, \Var{bVerte} boules vertes et \Var{bRouge} boules rouges.
\begin{enumerate}
\item % Proba
\begin{enumerate}
\item Quelle est la probabilité de tirer une boule bleu?
\begin{solution}
$\dfrac{\Var{bBleu}}{\Var{nbrTot}} \approx \Var{(bBleu / nbrTot) |round(2)}$
\end{solution}
\item Quelle est la probabilté de tirer une boule jaune ou bleu?
\begin{solution}
$\dfrac{\Var{bJaune + bBleu}}{\Var{nbrTot}} \approx \Var{((bJaune + bBleu)/ nbrTot) | round(2)}$
\end{solution}
\item A-t-on plus de chance de tirer une boule verte ou une boule rouge?
\begin{solution}
Boules vertes: $\dfrac{\Var{bVerte}}{\Var{nbrTot}} \approx \Var{(bVerte / nbrTot) |round(2)}$
Boules rouges: $\dfrac{\Var{bRouge}}{\Var{nbrTot}} \approx \Var{(bRouge / nbrTot) |round(2)}$
\Block{if bVerte > bRouge}
Une boule verte
\Block{else}
Une boule rouge
\Block{endif}
\end{solution}
\end{enumerate}
\item % Stat
On effectue 14 tirages (avec remise) dans cette urne et on obtient les couleurs suivantes:
\Block{set urne = ['B']*bBleu +
['J']*bJaune +
['V']*bVerte +
['R']*bRouge
}
\Block{set sample = Dataset.random(14, distrib="choice", rd_args=[urne], nbr_format=str)}
\begin{center}
\Var{sample | join(" \hspace{0.4cm}")} \\
\end{center}
\begin{enumerate}
\item Compléter le tableau des effectifs ci-dessous
\Block{set wsample = WeightedDataset(sample)}
\begin{tabular}{|c|*{4}{c|}}
\hline
Couleur & Bleu & Jaune & Vert & Rouge \\
\hline
Effectif & & & & \\
\hline
\end{tabular}
\begin{solution}
\begin{tabular}{|c|*{4}{c|}}
\hline
Couleur & Bleu & Jaune & Vert & Rouge \\
\hline
Effectif & \Var{wsample['B']} & \Var{wsample['J']} & \Var{wsample['V']} & \Var{wsample['R']} \\
\hline
\end{tabular}
\end{solution}
\item Calculer la fréquence des boules vertes.
\begin{solution}
Fréquence de boules vertes: $\frac{\Var{sample.count('V')}}{14}$
\end{solution}
\end{enumerate}
\item À chaque couleur, on associe des points. Une boule bleu rapporte 10 points, une boule jaune 5 points, une boule verte 2 points et une boule rouge 0 points.
\begin{enumerate}
\Block{set wPts = WeightedDataset([10, 5, 2, 0], [wsample['B'], wsample['J'], wsample['V'], wsample['R']])}
\item Combien de points a-t-on gagné au total?
\begin{solution}
\Var{wPts.sum()}
\end{solution}
\item Calculer la moyenne des gains.
\begin{solution}
\Var{wPts.mean()}
\end{solution}
\item Calculer la médiane des gains.
\begin{solution}
\Var{wPts.quartile(2)}
\end{solution}
\end{enumerate}
\end{enumerate}

Binary file not shown.

Binary file not shown.

Binary file not shown.

View File

@ -0,0 +1,201 @@
<?xml version="1.0" encoding="UTF-8" standalone="no"?>
<!-- Created with Inkscape (http://www.inkscape.org/) -->
<svg
xmlns:dc="http://purl.org/dc/elements/1.1/"
xmlns:cc="http://creativecommons.org/ns#"
xmlns:rdf="http://www.w3.org/1999/02/22-rdf-syntax-ns#"
xmlns:svg="http://www.w3.org/2000/svg"
xmlns="http://www.w3.org/2000/svg"
xmlns:sodipodi="http://sodipodi.sourceforge.net/DTD/sodipodi-0.dtd"
xmlns:inkscape="http://www.inkscape.org/namespaces/inkscape"
width="150.23199mm"
height="136.30997mm"
viewBox="0 0 532.31809 482.98807"
id="svg2"
version="1.1"
inkscape:version="0.91 r13725"
sodipodi:docname="parcours.svg">
<defs
id="defs4">
<marker
inkscape:stockid="Arrow1Mend"
orient="auto"
refY="0"
refX="0"
id="Arrow1Mend"
style="overflow:visible"
inkscape:isstock="true">
<path
id="path4193"
d="M 0,0 5,-5 -12.5,0 5,5 0,0 Z"
style="fill:#000000;fill-opacity:1;fill-rule:evenodd;stroke:#000000;stroke-width:1pt;stroke-opacity:1"
transform="matrix(-0.4,0,0,-0.4,-4,0)"
inkscape:connector-curvature="0" />
</marker>
</defs>
<sodipodi:namedview
id="base"
pagecolor="#ffffff"
bordercolor="#666666"
borderopacity="1.0"
inkscape:pageopacity="0.0"
inkscape:pageshadow="2"
inkscape:zoom="0.5083801"
inkscape:cx="343.2008"
inkscape:cy="320.01025"
inkscape:document-units="px"
inkscape:current-layer="layer1"
showgrid="false"
inkscape:object-paths="true"
inkscape:snap-intersection-paths="true"
inkscape:object-nodes="true"
inkscape:snap-smooth-nodes="true"
inkscape:snap-nodes="false"
fit-margin-top="0"
fit-margin-left="0"
fit-margin-right="0"
fit-margin-bottom="0"
inkscape:window-width="1366"
inkscape:window-height="711"
inkscape:window-x="0"
inkscape:window-y="27"
inkscape:window-maximized="1" />
<metadata
id="metadata7">
<rdf:RDF>
<cc:Work
rdf:about="">
<dc:format>image/svg+xml</dc:format>
<dc:type
rdf:resource="http://purl.org/dc/dcmitype/StillImage" />
<dc:title></dc:title>
</cc:Work>
</rdf:RDF>
</metadata>
<g
inkscape:label="Calque 1"
inkscape:groupmode="layer"
id="layer1"
transform="translate(-135.94006,-328.23181)">
<g
id="g5252">
<path
inkscape:connector-curvature="0"
id="path4136"
d="m 173.09883,723.86783 0,-363.90095 182.93399,0 z"
style="fill:none;fill-rule:evenodd;stroke:#000000;stroke-width:3.75;stroke-linecap:butt;stroke-linejoin:miter;stroke-miterlimit:4;stroke-dasharray:none;stroke-opacity:1" />
<path
inkscape:connector-curvature="0"
id="path4138"
d="M 173.09883,723.86783 527.16462,574.37339 641.80979,772.94464 Z"
style="fill:none;fill-rule:evenodd;stroke:#000000;stroke-width:3.75;stroke-linecap:butt;stroke-linejoin:miter;stroke-miterlimit:4;stroke-dasharray:none;stroke-opacity:1" />
<path
inkscape:connector-curvature="0"
id="path4140"
d="M 367.43234,641.81591 430.3568,750.80426"
style="fill:none;fill-rule:evenodd;stroke:#000000;stroke-width:3.75;stroke-linecap:butt;stroke-linejoin:miter;stroke-miterlimit:4;stroke-dasharray:none;stroke-opacity:1" />
<rect
y="359.96689"
x="173.09883"
height="14.260983"
width="14.260983"
id="rect4142"
style="fill:none;fill-opacity:1;stroke:#000000;stroke-width:3.75;stroke-miterlimit:4;stroke-dasharray:none;stroke-opacity:1" />
<text
sodipodi:linespacing="125%"
id="text4146"
y="732.71948"
x="135.72522"
style="font-style:normal;font-weight:normal;font-size:27.5px;line-height:125%;font-family:sans-serif;letter-spacing:0px;word-spacing:0px;fill:#000000;fill-opacity:1;stroke:none;stroke-width:1px;stroke-linecap:butt;stroke-linejoin:miter;stroke-opacity:1"
xml:space="preserve"><tspan
y="732.71948"
x="135.72522"
id="tspan4148"
sodipodi:role="line">A</tspan></text>
<text
sodipodi:linespacing="125%"
id="text4150"
y="348.64197"
x="155.39554"
style="font-style:normal;font-weight:normal;font-size:27.5px;line-height:125%;font-family:sans-serif;letter-spacing:0px;word-spacing:0px;fill:#000000;fill-opacity:1;stroke:none;stroke-width:1px;stroke-linecap:butt;stroke-linejoin:miter;stroke-opacity:1"
xml:space="preserve"><tspan
y="348.64197"
x="155.39554"
id="tspan4152"
sodipodi:role="line">C</tspan></text>
<text
sodipodi:linespacing="125%"
id="text4154"
y="351.11523"
x="362.91745"
style="font-style:normal;font-weight:normal;font-size:27.5px;line-height:125%;font-family:sans-serif;letter-spacing:0px;word-spacing:0px;fill:#000000;fill-opacity:1;stroke:none;stroke-width:1px;stroke-linecap:butt;stroke-linejoin:miter;stroke-opacity:1"
xml:space="preserve"><tspan
y="351.11523"
x="362.91745"
id="tspan4156"
sodipodi:role="line">D</tspan></text>
<text
sodipodi:linespacing="125%"
id="text4158"
y="562.57117"
x="530.11517"
style="font-style:normal;font-weight:normal;font-size:27.5px;line-height:125%;font-family:sans-serif;letter-spacing:0px;word-spacing:0px;fill:#000000;fill-opacity:1;stroke:none;stroke-width:1px;stroke-linecap:butt;stroke-linejoin:miter;stroke-opacity:1"
xml:space="preserve"><tspan
y="562.57117"
x="530.11517"
id="tspan4160"
sodipodi:role="line">E</tspan></text>
<text
sodipodi:linespacing="125%"
id="text4162"
y="785.82935"
x="654.03821"
style="font-style:normal;font-weight:normal;font-size:27.5px;line-height:125%;font-family:sans-serif;letter-spacing:0px;word-spacing:0px;fill:#000000;fill-opacity:1;stroke:none;stroke-width:1px;stroke-linecap:butt;stroke-linejoin:miter;stroke-opacity:1"
xml:space="preserve"><tspan
y="785.82935"
x="654.03821"
id="tspan4164"
sodipodi:role="line">F</tspan></text>
<text
sodipodi:linespacing="125%"
id="text4166"
y="779.01465"
x="411.03983"
style="font-style:normal;font-weight:normal;font-size:27.5px;line-height:125%;font-family:sans-serif;letter-spacing:0px;word-spacing:0px;fill:#000000;fill-opacity:1;stroke:none;stroke-width:1px;stroke-linecap:butt;stroke-linejoin:miter;stroke-opacity:1"
xml:space="preserve"><tspan
y="779.01465"
x="411.03983"
id="tspan4168"
sodipodi:role="line">F'</tspan></text>
<text
sodipodi:linespacing="125%"
id="text4170"
y="627.48328"
x="347.18118"
style="font-style:normal;font-weight:normal;font-size:27.5px;line-height:125%;font-family:sans-serif;letter-spacing:0px;word-spacing:0px;fill:#000000;fill-opacity:1;stroke:none;stroke-width:1px;stroke-linecap:butt;stroke-linejoin:miter;stroke-opacity:1"
xml:space="preserve"><tspan
y="627.48328"
x="347.18118"
id="tspan4172"
sodipodi:role="line">E'</tspan></text>
<text
sodipodi:linespacing="125%"
id="text4174"
y="805.49969"
x="216.37354"
style="font-style:normal;font-weight:normal;font-size:27.5px;line-height:125%;font-family:sans-serif;letter-spacing:0px;word-spacing:0px;fill:#000000;fill-opacity:1;stroke:none;stroke-width:1px;stroke-linecap:butt;stroke-linejoin:miter;stroke-opacity:1"
xml:space="preserve"><tspan
y="805.49969"
x="216.37354"
id="tspan4176"
sodipodi:role="line">Départ et arrivée</tspan></text>
<path
sodipodi:nodetypes="cc"
inkscape:connector-curvature="0"
id="path4178"
d="m 204.57134,793.69748 c -89.99172,0.98351 -36.88185,-60.48624 -36.88185,-60.48624"
style="fill:none;fill-rule:evenodd;stroke:#000000;stroke-width:3.75;stroke-linecap:butt;stroke-linejoin:miter;stroke-miterlimit:4;stroke-dasharray:none;stroke-opacity:1;marker-end:url(#Arrow1Mend)" />
</g>
</g>
</svg>

After

Width:  |  Height:  |  Size: 8.3 KiB

View File

@ -0,0 +1,91 @@
% exo de geometrie comme au brevet blanc.
%- set AD, AC, DC = random_pythagore()
%- set tourACDA = AC+AD+DC
%- set AE, AF = round(tourACDA/2*random(), 1), round(tourACDA/2*random(), 1)
%- set EF = round(tourACDA - AE - AF - randint(20,40)*0.2, 1)
%- set tourAEFA = round(AE+EF+AF, 1)
%- set rapport = randint(2,5)
%- set AE1, AF1, EF1 = round(AE/rapport,2) , round(AF/rapport,2), round(EF/rapport,2)
%- set objectif = randint(floor(tourAEFA), tourACDA)
%- if objectif > 100
%- set unit = "m"
%- else
%- set unit = "km"
%- endif
Une commune souhaite aménager des parcours de santé sur son territoire. On fait deux propositions au conseil municipale, schématisés ci-dessous:
\begin{itemize}
\item Le parcours ACDA
\item Le parcours AEFA
\end{itemize}
Ils souhaitent faire un parcours dont la longueur s'approche le plus possible de \Var{objectif}\Var{unit}.
Peux-tu les aider à choisir le parcours? Justifie
\textbf{Attention: La figure proposée au conseil municipale n'est pas à l'échelle, mais les codages et les dimension données sont correctes.}
\begin{minipage}{0.6\textwidth}
\includegraphics[scale = 0.4]{./fig/parcours}
\end{minipage}
\begin{minipage}{0.4\textwidth}
\begin{itemize}
\item $AC = \Var{AC}\Var{unit}$
\item $CD = \Var{DC}\Var{unit}$
\item $AE' = \Var{AE1}\Var{unit}$
\item $AE = \Var{AE}\Var{unit}$
\item $AF = \Var{AF}\Var{unit}$
\item $E'F' = \Var{EF1}\Var{unit}$
\item $(E'F') // (EF)$
\item L'angle $\widehat{EAF}$ vaut $30^o$
\end{itemize}
\end{minipage}
\begin{solution}
\begin{itemize}
\item Parcours ACDA:
D'après la figure, on voit que le triangle $ACD$ est rectangle en $C$ donc d'après le théorème de Pythagore, on a
\begin{align*}
AD^2 &= AC^2 + DC^2 \\
AD^2 &= \Var{AC}^2 + \Var{DC}^2 \\
AD^2 &= \Var{AC**2} + \Var{DC**2} \\
AD^2 &= \Var{AC**2 + DC**2} \\
AD &= \sqrt{\Var{AC**2 + DC**2}} = \Var{AD}\Var{unit}
\end{align*}
Donc le parcours ACDA mesure
\begin{align*}
AD + AC + CD = \Var{AD} + \Var{AC} + \Var{DC} = \Var{tourACDA}\Var{unit}
\end{align*}
\item Parcours AEFA:
D'après les données, on sait que $(EF) // (E'F')$. On voit aussi que $A$, $E'$ et $E$ sont alignés. Il en est de même pour les points $A$, $F'$ et $F$. Donc d'après le théorème de Thalès
\begin{tabular}{|c|c|c|c|}
\hline
Triangle AEF & AE = \Var{AE} & AF = \Var{AF} & EF \\
\hline
Triangle AE'F' & AE' = \Var{AE1} & AF' & E'F' = \Var{EF1} \\
\hline
\end{tabular}
est un tableau de proportionnalité. Donc on peut faire un produit en croix pour calcul $EF$.
\begin{align*}
EF = \frac{E'F' \times AE}{AE'} = \frac{\Var{EF1} \times \Var{AE}}{\Var{AE1}} = \Var{EF} \Var{unit}
\end{align*}
Donc le parcours AEFA mesure
\begin{align*}
AF + AE + EF = \Var{AF} + \Var{AE} + \Var{EF} = \Var{tourAEFA}\Var{unit}
\end{align*}
\item Choix du parcours:
%- if abs(tourACDA - objectif) < abs(tourAEFA - objectif)
Il faudra choisir le tour $ACDA$ car sa longueur est plus proche de \Var{objectif}\Var{unit}.
%- else
Il faudra choisir le tour $AFEA$ car sa longueur est plus proche de \Var{objectif}\Var{unit}.
%- endif
\end{itemize}
\end{solution}

View File

@ -0,0 +1,82 @@
% theme: Fonction linéaire, Fonction affine
% require: tkz-fct
%- set I_a = randint(100, 200)
%- set G_b =randint(3*I_a, 4*I_a)
%- set G_a = randint(I_a*14//20, I_a*16//20)
Taraina veut inscrire ses 21 élèves à un entrainement pour l'évènement \textbf{Battle of the year}.
Deux tarifs lui sont proposés:
\begin{itemize}
\item Tarif Individuel: \Var{I_a} \euro par danseur inscrit.
\item Tarif Groupe : Paiement d'un forfait de \Var{G_b} \euro pour le groupe puis \Var{G_a} \euro par danseur inscrit.
\end{itemize}
\medskip
\begin{enumerate}
\item Complète le tableau suivant :
\medskip
\begin{tabularx}{0.7\linewidth}{|c|*{3}{>{\centering \arraybackslash}X|}}\hline
Nombre d'inscriptions & 0 & 10 & 25\\
\hline
Prix au tarif Individuel en \euro & & & \\
\hline
Prix au tarif Groupe en \euro & & & \\
\hline
\end{tabularx}
\begin{solution}
\begin{tabularx}{0.7\linewidth}{|c|*{3}{>{\centering \arraybackslash}X|}}\hline
Nombre d'inscriptions & 0 & 10 & 25\\
\hline
Prix au tarif Individuel en \euro & 0 & \Var{10*I_a} & \Var{25*I_a} \\
\hline
Prix au tarif Groupe en \euro & \Var{G_b} & \Var{G_b + 10*G_a} & \Var{G_b + 25*G_a}\\
\hline
\end{tabularx}
\end{solution}
\medskip
\item Pour chacun des tarifs, exprimer le prix en fonction du nombre de danseurs inscrits.
\begin{solution}
$x$ représente ici le nombre d'élèves inscrits.
\begin{itemize}
\item Tarif Individuel: $f: x \mapsto \Var{I_a}x$
\item Tarif Groupe: $g: x \mapsto \Var{G_a}x + \Var{G_b}$
\end{itemize}
\end{solution}
\item Tracer sur le graphique suivant, les courbes représentants les 2 tarifs proposés.
%- set xmax = 26
%- set ymax = I_a*26
% On force que le graphique soit légèrement plus grand que 12 cm
%- set yscale = round(12*200/ymax, 1)
\begin{tikzpicture}[yscale=\Var{yscale}]
\tkzInit[xmin=0,xmax=\Var{xmax},
ymin=0,ymax=\Var{ymax},
xstep=2,ystep=200]
\tkzAxeX[thick, poslabel=right,label=]
\tkzAxeY[thick, poslabel=above,label=]
\tkzDrawX[label={\textit{Danseurs inscrits}},below= -12pt]
\tkzDrawY[label={\textit{Prix}}, below=-10pt]
\tkzGrid
\tkzFct[domain=0:\Var{xmax}, color=blue, very thick]{\Var{I_a}*\x}
\tkzFct[domain=0:\Var{xmax}, color=red, very thick]{\Var{G_a}*\x+\Var{G_b}}
\end{tikzpicture}
\item Pour quel nombre d'inscriptions paye-t-on le même prix quel que soit le tarif choisi?
\begin{solution}
$\Var{G_b/(I_a - G_a)}$ % est entre 10 et 20 par contruction des paramètres
\end{solution}
\end{enumerate}

View File

@ -0,0 +1,42 @@
\documentclass[a4paper,12pt]{article}
\usepackage{myXsim}
\usepackage{tkz-fct}
\usepackage{wrapfig}
\title{DM de Paques}
\tribe{302}
\date{Jeudi 3 mai 2018}
\sujet{\Var{infos.num}}
%\geometry{left=10mm,right=10mm, bottom= 10mm, top=10mm}
\xsimsetup{
solution/print = false
}
\begin{document}
\maketitle
\bigskip
{\Large \textbf{Nom - Prénom:}}
\begin{exercise}[subtitle={Battle of the year}]
\Block{include "./tarif_danseurs.tex"}
\end{exercise}
\begin{exercise}[subtitle={Parcours}]
\Block{include "./parcours.tex"}
\end{exercise}
\begin{exercise}[subtitle={Tirages au sort}]
\Block{include "./Proba_stat.tex"}
\end{exercise}
\end{document}
%%% Local Variables:
%%% mode: latex
%%% TeX-master: "master"
%%% End: